Вы находитесь на странице: 1из 285

First published in 2017 by Gloucester Publishers Limited, London

Copyright © 2017 Cyrus Lakdawala

The right of Cyrus Lakdawala to be identified as the author of this work has been
asserted in accordance with the Copyrights, Designs and Patents Act 1988.

All rights reserved. No part of this publication may be reproduced, stored in a


retrieval system or transmitted in any form or by any means, electronic,
electrostatic, magnetic tape, photocopying, recording or otherwise,
without prior permission of the publisher.

British Library Cataloguing-in-Publication Data


A catalogue record for this book is available from the British Library.

ISBN Kindle: 978 1 78194 411 0


ISBN Ebook: 978 1 78194 412 7

Distributed in North America by National Book Network,


15200 NBN Way, Blue Ridge Summit, PA 17214. Ph: 717.794.3800.

Distributed in Europe by Central Books Ltd.,


99 Wallis Road, London E9 5LN. Ph 44(0)845 458 9911.

All other sales enquiries should be directed to Everyman Chess.


email: info@everymanchess.com; website: www.everymanchess.com

Everyman is the registered trade mark of Random House Inc. and is used in this work
under licence from Random House Inc.

Everyman Chess Series


Chief advisor: Byron Jacobs

Typeset and edited by First Rank Publishing, Brighton.


Cover design by Horatio Monteverde.
Printed by TJ International Limited, Padstow, Cornwall.
About the Author
Cyrus Lakdawala is an International Master, a former National Open and American Open
Champion, and a six-time State Champion. He has been teaching chess for over 30 years,
and coaches some of the top junior players in the U.S.

Also by the Author:


Play the London System
A Ferocious Opening Repertoire
The Slav: Move by Move
1 ... d6: Move by Move
The Caro-Kann: Move by Move
The Four Knights: Move by Move
Capablanca: Move by Move
The Modern Defence: Move by Move
Kramnik: Move by Move
The Colle: Move by Move
The Scandinavian: Move by Move
Botvinnik: Move by Move
The Nimzo-Larsen Attack: Move by Move
Korchnoi: Move by Move
The Alekhine Defence: Move by Move
The Trompowsky Attack: Move by Move
Carlsen: Move by Move
The Classical French: Move by Move
Larsen: Move by Move
1 ... b6: Move by Move
Bird’s Opening: Move by Move
Petroff Defence: Move by Move
Fischer: Move by Move
Anti-Sicilians: Move by Move
Contents
About the Author
Bibliography
Introduction

1 The Pseudo-Dragon and Pseudo-Lopez


2 The Classical Variation
3 The Jump to Nowhere
4 The Bc4 Cro-Magnon Lines
5 Fianchetto Lines
6 The Coward’s Variation
7 The Austrian Attack
8 The Dzindzi-Indian
9 The Averbakh Variation
10 Other d4 and c4 Lines
11 English Opening Set-ups
12 Anti-Queen’s Pawn Lines and Everything Else

Index of Complete Games


Bibliography
Dangerous Weapons: The Pirc and Modern, Richard Palliser, Colin McNab & James Vigus
(Everyman Chess 2009)
Modern Defence, Jon Speelman & Neil McDonald (Everyman Chess 2000)
Starting Out: The Modern, Nigel Davies (Everyman Chess 2008)
Starting Out: The Pirc/Modern, Joe Gallagher (Everyman Chess 2003)
The Fianchetto Solution, Emmanuel Nieman & Samy Shoker (New in Chess 2016)
The Hippopotamus Rises, Andrew Martin (Batsford 2005)
The Modern Defence: Move by Move, Cyrus Lakdawala (Everyman Chess 2012)
The Modern Tiger, Tiger Hillarp Persson (Quality Chess 2014)
Tiger’s Modern, Tiger Hillarp Persson (Quality Chess 2005)

Electronic/Online
ChessBase 13 with the Komodo 9 engine
ChessPublishing.com
ChessToday.net
Introduction
If your writer is the Dr. Jane Goodall of boring openings, then did I write a book on the
Modern Defence, an exception to the norm? I’m the guy who writes books on Slav, Caro-
Kann, Colle and London Systems, all openings designed to insulate the strategic-minded
player from a tactical opponent’s volatility.
I’m not normally a fan of the embrace-your-weakness school, where you play an
opening which is almost anti-matter to your normal style, since when we play a line
which fails to suit our natural style, a part of our mind screams betrayal. The Modern
Defence is one of the most complex lines in chess (completely opposite to my style), yet I
have played it since my youth and for some bizarre reason, always scored well with it,
unlike other experiments with complex lines like the Dragon and Najdorf.
When we abandon the openings of our youth, we become the 18-year-old who leaves
his economically depressed small town for the big city, vowing to never return. Yet in the
case of the Modern, I return, again and again. When I was I kid I cherished the delusion
that deep inside me slept a dormant, attacking version of me, which today I realize just
doesn’t exist. Somehow the Modern fits, because I’m essentially a counter-puncher, not
an attacker, and in the Modern, you had better prepare to get attacked – a lot!
The Modern has become one of my forbidden pleasures. One’s viewpoint depends
upon just where the observer is standing. From my perspective, playing the Modern is the
same as agreeing to a lucrative business deal with a mob boss. You may make a lot of
money, but on the other hand, he may put a contract on your life if he feels cheated. So if
you prefer to forego the baroque splendor of tangled complications, and like to play it
simple and safe, then the Modern Defence is definitely not for you (I’m sorry, but there is
a strict no refund policy on this book).
There is a distressing divide between our study and our application of what we
learned over the board. At first the Modern feels utterly confusing, but with practice, our
understanding gels with our study. I dislike chess books where the advice given feels like
it is given by a well meaning yet impractical academician. So in this one I mainly
concentrate on the central aspects of my own Modern Defence repertoire, with
explanations which hopefully are clear and easy to understand.

A High Risk Venture: Is Our Opening Sound?


When we opt for a tempting, but risky opening like the Modern, it sometimes feels as if
we voluntarily place ourselves on trial. Our argument’s effectiveness or failure will be the
judge, who either preserves and enriches, or ruins us. By refusing to seize central space,
are we not offering our opponent too much latitude? To bait our opponent to seize the
centre and then either squeeze us, or attack us, opens us to grave risk/opportunity.
One thing you will rarely get in the Modern is a dull game, and we must weave in an
increased error-factor into our computations and assessments, since the positions we
reach tend to be both strategically gnarled and at the same time, tactically tense. The
variations themselves tend not to be of a straight line nature, but, instead, a tangle of
branches. So it’s important for us to factor in a higher number of errors from both sides
when compared to more sedate opening lines, due to the increased complexity levels.
The positions we reach exude an alien, other-worldly quality of a reality out of synch with
the human world.
Essentially, in the Modern we are graded on a curve, which means we can commit 10
errors and still win, if our opponent commits 11. There are few opening lines which
arouse such passions, concerning their soundness or lack of it (I have been told so often
that the Modern is busted, that my ears feel like they seep blood every time I hear this
false claim at the San Diego Chess Club). Draws are less likely in the Modern, since when
attack and defence clash to this degree, there is generally only room for one at the top.
Just realize that because we tend to be on the defensive in many Modern lines, this
doesn’t necessarily translate to inferiority, since White’s dynamism may easily morph in to
harmful exaggeration which overestimates attacking chances, while underestimating
structural and material cost. Despite intense theoretical scrutiny, our opening remains
perfectly viable for Black.

Why Play 1 ... g6?


To a chess writer, advice is our sole export product. So let me tell you why I love the
Modern: one great advantage of 1 ... g6 is that it’s a mono-opening which can be played
on virtually any white first move. Our philosophy in the opening stage is to strive for
exclusion and become masters of the not-so-narrow structures, which hopefully are
familiar to us, but not so much for our opponents. Adaption is evolution’s single most
survival tool, so be on high alert for opportunities/dangers of structural shifts, which
happen with alarming frequency in the Modern.
Let’s take a look at some of our key battlegrounds:
For now the future remains an amorphous entity, still waiting to take shape. White’s
Pseudo-Dragon position in the above diagram will probably be the structure you face the
most, since White’s intended attack with f3, g4 and h5 is fun, mechanical and easy to
understand, and therefore very popular at club level. We have multiple methods of
thwarting this intent. We can play ... h5, or we can play the sneaky ... h6, intending to
meet White’s coming h5 with the bypassing ... g5. Yes, our opponent attacks us, but
remember these points:
1. An attack is an entity which by its nature, tends to consume, rather than produce.
This means that White often hands over material or agrees to structural concessions in
the process.
2. Against the Pseudo-Dragon we rely upon a battle plan where
our shifty king often remains in the centre, for extended periods
and sometimes the entire game. The purpose is to avoid
offering our opponent the necessary data of where to attack us.
If our opponent attacks kingside, our king may castle long. If
they press on the queenside, we can castle short. The point is,
by stalling castling, our opponent may be tempted into pouring
wealth and resources into the invasion of an empty lot.
A king or queen may rule in one of three ways:
1. By the will of the people.
2. Out of tradition and precedent.
3. By force and fear.
It’s pretty obvious that White goes for number three on the list with his last move, 4
f4. In the Austrian Attack our opponent renounces all ties to security, and for some
Modern Defence players like me, entry to the scary Austrian Attack represents a brief,
panic-attack-inducing moment. Don’t underestimate Black’s chances, since the Austrian
Attack folk are people who make $3,000 a month, and tend to spend $3,500. Often we
are the recipients of White’s sacrificial generosity. If we avoid mate (which I concede is a
big ‘if’), we often overextend White.
This incongruous mix is the Neanderthal with a cell phone. The Dzindzi-Indian is the
intellectual property of my friend GM Roman Dzindzichashvili, where our mongrelized
entity mixes ideas from the Nimzo-Indian and Leningrad Dutch. In the opening phase
many of us quickly bang out the first 10 or 15 moves with the ease of long familiarity,
where obsessive memorization becomes a substitute for creativity. Not so here, since our
Dzindzi-Indian may catch most of your club level opponents off guard, since it received
only scant theoretical attention.
Our idea is to hand over our precious dark-squared bishop with ... Bxc3+ (which
seems to be a recurring theme in this book), and follow it with ... f5, which inhibits but
doesn’t prevent White’s e4. Our hope is that the structural damage we inflict overcomes
White’s bishop-pair and enhanced dark-square control. To hand over our dark-squared
bishop in the Modern may feel like the equivalent of a baseball team losing their best
hitter to an injury. Is our blasphemous premise sound? Well, to me the question is an
unanswerable as the Mona Lisa’s smile.
In the above position White’s most dangerous line is to play 6 e4! and after 6 ... fxe4
proceed in Staunton Gambit fashion with 7 f3!. Our best is to decline with either ... d6
and ... e5!, or ... Nf6!, intending ... Nbd7 and then take on doubled isolated e-pawns with
... Ne5.
If the Dzindzi-Indian feels too radical, then you have the option of bypassing it entirely
with 3 ... d6 4 e4 e5 which leads to the Averbakh line, rather than 3 ... c5, which enters
the Dzindzi-Indian.
It’s easy to spot the structural landmarks of a King’s Indian, but our Modern versus
White’s queen’s pawn lines differ in the following ways:
1. With our last move ... e5, we allow the game to jump from the opening phase to
the ending in an instant, without the in-between middlegame period. We lose the right to
castle, should White exchange on e5. This is okay, since we are in an ending and our king
plans to help out, rather than hide.
2. If White opts for a d5 blocked formation, we can play for an early ... f5. How does
this differ from King’s Indian? In the King’s Indian Black first plays ... Nf6, then moves the
knight and only then plays ... f5. This means that in our Modern version, we can play ... f5
and then follow with ... Nf6, where we gain a full move over the King’s Indian.
One word of warning though: it’s not so simple, since in many lines White can play
exf5 and the position alters into something distinctly alien. When we correctly recapture
with our g-pawn, White’s queen can give check on h5, as in the So-Norwood game from
the book. This results in a crisis/opportunity situation for our side far faster than in any
King’s Indian.
Here White’s position is Fred Reinfeld’s dream. After all, White dominates the centre
while Black’s forces squirm uncomfortably in their cramped quarters. Our strength once
again lies in our attack-the-vacant-lot theory. White finds it incredibly difficult to locate a
target in our position, or open the position. If they play e5, we shut them down with ...
d5; if they play d5, we close with ... e5. The point is we are the only one with viable
pawn breaks. Our chameleon intent is barely discernible, since it blends into the position’s
natural landscape.
In 1968 I fell under the hypnotic spell of the Modern Defence games of Duncan
Suttles, Canada’s first grandmaster. I waited impatiently for my Canadian Chess Chat
magazine, which inevitably featured Suttles’ bizarre games/interpretations of the
opening. Almost five decades later, I still vaguely remembered the following one, which
allows me to happily time-travel back to 1967.

Game 1
L.Barczay-D.Suttles
Sousse Interzonal 1967

I played the following game over when I was seven years old, and today, 50 years later,
still vaguely remembered the diagrammed position on move 14, where Suttles
annihilated the base of White’s queenside.
1 e4 g6
For Canadian chess players in the 1960’s, the bizarre Modern
Defence games of Duncan Suttles, then our country’s only GM,
were akin to the faithful, reading scripture.
2 d4 Bg7 3 Nf3 d6 4 Bc4
Carlsen has played this move many times in blitz games. This is the Cro-Magnon line,
where White, in the most unsubtle fashion possible eyes the vulnerable f7-square.
4 ... c6

Note: In the old days Modern players set up with ... c6 and ... b5, which today has
been replaced with a preference for the more efficient ... a6 and ... b5 set-up.

Why is the latter formation more efficient? The answer is because in the ... c6 version,
Black often later plays for ... Bb7, ... Nbd7 and ... c5. If Black’s c-pawn is destined to go to
c5, then why waste a tempo with the redundant ... c6? I generally meet Cro-Magnon lines
with entry to the Hippopotamus formation with 4 ... e6.
5 Nc3
5 Qe2 Nf6 6 e5 Nd5 is also okay for Black.
5 ... b5!?
It’s hard not to interpret such a gesture as an affront. The move violates the principle:
avoid confrontation when lagging in development. 5 ... Nf6 is a lot safer.
6 Bb3
The natural reaction. GM Barczay avoids the temptation to punish his opponent with 6
Nxb5!?. A move like this is played with the thought: any discourtesy must be immediately
rebuffed. Attempting to decipher such coming complications at the board turns us into the
witch doctor who tosses the bones and then believes he accurately interprets their
sprawl.
After 6 ... d5! (6 ... cxb5?? 7 Bd5 White wins material) 7 Bb3 dxe4 two white pieces
are simultaneously attacked, but this is far from the end of the story: 8 Ng5! cxb5 9 Nxf7
Qxd4 10 Qxd4 Bxd4 11 c3 Bg7 12 Nxh8 e6 13 Nxg6 hxg6. Black is probably okay here
with two pieces for a rook, pawn, plus weak black pawns.
6 ... b4?!

The main move, but I feel it’s not great, since it weakens without receiving benefit in
return.

Tip: Today, Modern players tend to play ... b4 only when White inserts a4 first.
I prefer the sober 6 ... Nf6.

7 Ne2 a5 8 c3
Somewhat stronger is 8 a3! Na6 9 axb4 axb4 10 Nf4 Bd7 (10 ... e5?? loses to 11 dxe5
dxe5 12 Bxf7+!) 11 Qe2! (attacking the a6-knight while menacing e5) 11 ... Qc8 12 e5 d5
13 0-0 and Black was left in a strategically miserable situation, A.Rombaldoni-G.Laco,
Trieste 2012.
8 ... Nf6 9 e5
After 9 Ng3 bxc3 10 bxc3 0-0 11 0-0 Qc7 Black looks fine.
9 ... dxe5 10 Nxe5
Avoiding 10 dxe5?! Qxd1+ 11 Bxd1 Nd5 when White’s extended e-pawn is a weakness
more than a strength in the ending.
10 ... 0-0 11 0-0 Nfd7
Challenging White’s e5 outpost.
12 Ng4
He hopes to make use of his control over h6.
12 ... a4! 13 Bc2
13 Bxa4? is met with 13 ... Nb6!, double-attacking a4 and g4. Following 14 Nh6+ Bxh6
15 Bxh6 Nxa4 16 Bxf8 Kxf8 Black’s two minor pieces are worth more than White’s rook
and pawn.
13 ... c5?
This move, although strategically desirable, is tactically unfeasible. Correct was 13 ...
bxc3 14 bxc3 Nb6 with approximately even chances.
14 Bh6?
Of all chess achievements, the most unstable may be possession of the initiative,
since our asset can be snatched away by a single indifferent move. This is the case of the
right square, but the wrong piece. White gains a serious advantage with 14 Nh6+! Kh8 15
dxc5! when Black is unable to recapture on c5, due to the knight fork on f7.
14 ... a3!

This diagram is the poster child for the hypermodern’s desire to destroy the
opponent’s centre from the wings. Suttles’ undermining action remains deeply imprinted
in my memory, despite the passage of a half century from when I originally played over
the game. Should we be thankful to comps, or despise them? When I first played over
this game, I was convinced that Black was winning here, but today’s comps say this
assessment just isn’t so, despite the dire look to White’s collapsing queenside.
15 Bxg7
15 bxa3 bxc3 16 Bxg7 Kxg7 17 Nxc3 Qa5 looks about even.
15 ... axb2 16 Rb1
16 Bxf8 bxa1R (I always perversely underpromote if given the opportunity) 17 Qxa1
Nxf8 18 Ne3 Ba6 19 c4 Nbd7 still looks about even.
16 ... Kxg7 17 cxb4 Nb6!
Suttles does a wonderful job of completely confusing the issue.
18 Ne5
White could also capture:
a) 18 dxc5? Bxg4 19 cxb6 Qxd1 20 Bxd1 Rxa2 21 Nc3 Bf5! 22 Bc2 Ra3 23 Nb5 Bxc2 24
Rxb2! Rb3! 25 Rxc2 Rxb4 26 Nc7 Rxb6 and Black comes out a pawn ahead.
b) 18 bxc5! Bxg4 19 cxb6 Rxa2 20 b7 Qb6 21 Be4 Rd8 when I prefer Black, but White’s
position remains fully playable.
18 ... cxd4

Exercise (critical decision): To play it safe or take action? Act prematurely and
we enter an uneven battle; refuse to act when we should, and opportunity may slip
away. White can play the safe 19 Nxd4, which eventually gives up his a2-pawn,
with an even game. The alternative is to play for a win with the riskier 19 Bb3,
after which it becomes a battle of White’s wing passers versus Black’s central passers?
Which line would you play?

19 Bb3?!
Answer: Black’s central passers are the more valuable in this line. Correct was to
enter the equal variation 19 Nxd4! Qd6! (19 ... Rxa2?? loses material to the clever shot
20 Nxf7! – attraction/overloaded defender – 20 ... Kxf7 21 Bb3+) 20 Re1 f6 21 Nef3 Rxa2
22 Bb3 Ra1 23 Rxa1 bxa1Q 24 Qxa1 which is about even. Black should avoid the greedy
24 ... Qxb4? which allows 25 Ng5! with a strong initiative.
19 ... f6 20 Nd3 e5
It’s ironic that the hypermodern player ended up with the centre, while the classical
side resolved with the wing pawns. In this case Black’s central pawns prove to be more
potent than White’s queenside majority.
21 Rxb2 Nc6 22 a4?
This is overly ambitious. White should proceed more cautiously with 22 Qd2, after
which Black stands only slightly better.
22 ... Qd6
Even stronger was 22 ... Bf5! 23 Nec1 e4 24 Nc5 d3 25 g4 Bc8 26 Re1 (26 Nxe4? Qd4
heavily favours Black) 26 ... f5 where it becomes obvious that Black’s central passers
clearly outweigh White’s on the a- and b-files.
23 Qc2?!
He had to try 23 a5 Nd5 24 Bxd5 Qxd5.
23 ... Bf5!
Stronger than the queen sacrifice 23 ... Nxb4 24 Nxb4 Qxb4 25 Bd5 Qxb2! 26 Qxb2
Nxd5, although even there Black has all the winning chances.
24 Ng3
We sense an escalation in White’s desperation. 24 Qc5 Qxc5 25 Nxc5 Nxb4 just leaves
him down a pawn for nothing.
24 ... Bxd3 25 Qxd3 Nxb4 26 Qb5 Rfb8
The side who is winning generally desires stability, rather than embraces
complications. Therefore 26 ... f5! cutting off Ne4, is a strong consideration.
27 Ne4 Qe7 28 a5 Nd7 29 Qc4
If 29 Qa4 f5 30 Nd2 Nc5 31 Qa3 Qc7 and the cruelty of natural selection singles out
the a-pawn as the herd’s straggler.
29 ... f5
There is nothing wrong with the simple 29 ... Rxa5.
30 Ng5!
This is a clever attempt to confuse matters.
30 ... Qxg5!?
A less adventurous player would consider the safer 30 ... Rxa5 31 Ne6+ Kh8 32 Rfb1
Nd5 when Black consolidates.
31 Qf7+ Kh6 32 Qxd7 Nd3 33 Rbb1 Nc5 34 Qd5 Rxa5!
White’s queenside passers are no more and e5 is safe due to the threat on b3.
35 Bc4 Rxb1 36 Rxb1 Qe7 37 Bf1 Ra7 38 Re1 Rd7!
Principle: place your rook behind your passed pawn.
39 Qg8
Alternatively, 39 Qxe5 Qxe5 40 Rxe5 d3 41 Re1 d2 42 Rd1 Ne4 and there is no
remedy to the coming ... Nc3, and 39 Rxe5 Rxd5 40 Rxe7 d3 41 Re1 d2 42 Rd1 Ne4 is the
same thing, with ... Nc3 next.
39 ... e4 40 Bc4 0-1
40 ... Rd8! forces a queen swap, rendering further resistance moot.

Summary
The inherent flexibility of the Modern Defence offers our imagination great scope of
stylistic interpretation.

Acknowledgements
Many thanks to Byron Jacobs for the edit and to Richard Palliser for the final edit, and
also thanks to chief proofreader Nancy. May the dark byways of the Modern lead you to
many happy adventures.

Cyrus Lakdawala,
San Diego,
May 2017
Chapter One
The Pseudo-Dragon and Pseudo-Lopez
1 e4 g6 2 d4 Bg7 3 Nc3 d6 4 Be3

In the Pseudo-Dragon part of the chapter White acts as if we played the Sicilian
Dragon, planning to attack our king in Yugoslav Dragon fashion with f3, 0-0-0, g4 and h4,
as here with 4 ... a6 5 Qd2 b5 6 h4.
However, there are three critical differences between the Sicilian Dragon and the
Modern Pseudo-Dragon, which I believe fall in our favour:
1. Unlike the Dragon, where Black’s king castles short early, in the Pseudo-Dragon,
our king remains uncommitted. We can indeed castle short, but we can also keep our
king in the middle, or even castle queenside. White’s dilemma: where to direct attacking
energy? Our opponent can only guess where our king will eventually end up.
2. In the Modern Pseudo-Dragon, our pawns are far more flexible than in a Sicilian
Dragon. This means that we can meet White’s kingside f3, g4, h4 pawn storm with either
... h5, or even ... h6, where we intend to bypass White’s h5 push with ... g5.
3. Our piece placement options are also higher in the Pseudo-Dragon. For example,
our g8-knight may later play to f6 in Dragon fashion, but we also have options of playing
the knight to e7, after making room with ... e6 or ... e5. Our b8-knight usually moves to
d7, and from there it can shift to b6 and sometimes even c5 if we exchanged our c-pawn
for White’s d-pawn, or maybe it can play to e5, and then c4, just as in the Sicilian Dragon.
These unpredictable shifts may induce a confusing effect on our opponent, since our side
represents a disorientingly moving target.
In the Pseudo-Lopez section, White’s play mimics a Ruy Lopez plan of transferring his
b1-knight to g3 to build up for a kingside attack. We have three key resources which
differentiate it from the Ruy Lopez, all I believe in our favour:
1. In most closed Lopez positions, Black fianchettos with the laboured manoeuvre ...
Be7, ... 0-0, ... Re8, ... Bf8, ... g6 and finally ... Bg7. In our Modern version, we skip the
unnecessary middlemen and just fianchetto straight away, saving multiple tempi.
2. In the Closed Lopez, Black already committed to castling kingside. In our Modern
version, we haven’t, giving us access to annoying ... h5 and ... h4 ideas, chasing our
opponent’s g3-knight.
3. We in some cases have access to a manoeuvre with ... Nf6, attacking e4, and when
White protects with Bd3, we play ... Ng4, chasing down White’s important dark-squared
bishop for our knight.

Game 2
N.Arutyunov-C.Lakdawala
San Diego (rapid) 2010

1 e4 g6
Ich bin ein Moderner. For a boring Slav/Caro-Kann guy like me to willingly enter the
unclear complications of the Modern feels the same as the double amputee signing up for
tennis lessons, but for some weird reason, I score well with the Modern, whereas I get
clobbered in other tangled/complex lines like the Dragon, Najdorf, et al. Maybe it’s
because the Modern is so heavily reliant on structural decisions and defensive ideas,
which are both within the competency realm of the strategist, rather than the tactician.
2 d4 Bg7 3 Nc3 d6 4 Be3
Practice has also regularly seen 4 f3 c6.

Note: The ... c6 version is the old school Modern. In this book we relegate these lines
to the notes and concentrate on the ... a6 ideas.

Here after 5 Be3 b5 6 Qd2 Nd7 7 h4 h5 8 Nh3 Nb6 9 Nf2 a5 10 Bd3 Rb8 11 b3 Bd7 12
Ne2 a4 13 Rd1 Nh6 14 0-0 Qc7 15 c3 Rb7 16 Bc2 Kf8!? 17 Nd3 Kg8 18 Ng3 Kh7 (man,
that is a laboured form of castling!) 19 Nf2 axb3 20 axb3 c5 21 Nh3 Bxh3 22 gxh3 b4! my
childhood hero managed to dismantle White’s entire pawn chain and went on to win,
L.Myagmarsuren-D.Suttles, Sousse Interzonal 1967.
4 ... a6
The thematic idea is to expand on the queenside. Instead, 4 ... Nf6 transposes to the
Pirc Defence and 4 ... c6, as mentioned in the above note, is the old school Modern.
5 Qd2
White enters the combative Pseudo-Dragon line, where he intends to play in Yugoslav
Dragon fashion, with f3, 0-0-0, g4, and h4.

Note: One feature which distinguishes the Modern from the Pirc is flexibility. In
the Modern, Black holds back on the development of the g8-knight, while in the
Pirc the knight is brought out quickly, with the move order often being
1 e4 d6 2 d4 Nf6 3 Nc3 g6.

5 ... Nd7
There is nothing wrong with the immediate 5 ... b5, and if 6 a4 b4 7 Nd1 Nf6 8 f3 a5 9
c3 bxc3 10 bxc3 with a complex position not unfavourable to Black.
6 f3
Later in the chapter we look at ideas involving:
a) 6 Nf3 b5 7 a4 b4 8 Ne2 Ngf6! 9 Ng3 Ng4! hits White’s key bishop.
b) 6 a4 is an attempt to thwart Black’s queenside aspirations.
Tip: When White suppresses our ... b5 break with a4, go for a Hippopotamus set-up
with ... b6. The price White paid for playing a4 is that he all but ensured that he won’t
be able to play 0-0-0, since Black can easily pry open the position with a ... b5
sacrifice.

Here 6 ... b6 7 Nf3 Bb7 8 Bd3 e6 9 0-0 h6 10 h3 Ne7 11 Rfe1 is a typical


Hippopotamus set-up, which we examine in greater detail later in the book.
c) 6 0-0-0 b5 7 h4 h6 8 f4 (a reminder: don’t assume the game won’t transpose to an
Austrian Attack-style game just because White didn’t play f4 on move 4; factor it in on
every move) 8 ... h5! 9 Nf3 Nh6! and White’s space is compensated by Black’s control
over the g4 hole.
d) 6 h4 h6 7 f4. There it is again! White pulls the switcharoo to the Austrian Attack.
Once again, Black can go for the plan 7 ... h5! 8 Nf3 Nh6!.
6 ... b5 7 g4
Alternatively:
a) 7 h4 should be met with 7 ... h5 8 Nh3 Bb7 9 Ng5 Ngf6 10 0-0-0 and now I like the
flexible 10 ... e6, keeping White in the dark as to where we will castle.
b) 7 a4 doesn’t fit well with White’s Yugoslav Attack game plan since it opens the
queenside, a place normally reserved for White’s king. Following 7 ... b4 8 Nd1 Rb8 9 c3
bxc3 10 bxc3 c5 11 Bd3 Qa5 12 Ne2 Ngf6 Black stands no worse.
7 ... Bb7 8 g5!?
It isn’t easy to strike the perfect balance between functionality and experimentation.
This non-standard move is made with the thought: when we blindly follow trends, the
mob mentality makes our opening decisions for us. The idea is to play h4 and h5, without
allowing Black bypass options of ... h6 and ... g5. White may also believe he is hampering
the development of my g8-knight, but this isn’t so, since the e7-square is still available
after ... e6.
Normal is 8 h4. Here Black can play 8 ... h6 keeping the position fluid and meeting 9
h5 with the bypassing 9 ... g5!.
8 ... e6

A new move in the position.

Tip: At any point you deem favourable, Hippopotamize your Modern by developing
your g8-knight to e7, after playing ... e6. The Hippopotamus set-up is normally ... g6,
... e6, ... d6 and ... b6, with the black knights on d7 and e7 and both bishops
fianchettoed.

8 ... h6! also looks fine for Black after 9 gxh6 Bxh6! 10 Bxh6 Rxh6. Black stands no
worse, since White’s kingside structure is slightly damaged and Black can either leave his
king in the centre, or castle queenside.
9 a3
I’m not so sure he needs this move, which wastes time and also provides Black with a
target to pry open queenside lines with a future ... b4 pawn break. 9 h4 can be met with
9 ... h5!.
9 ... h6!
This move breaks up White’s kingside structure.
10 h4
Avoiding 10 gxh6? Qh4+! 11 Qf2 Qxf2+ 12 Kxf2 Bxh6 13 Bxh6 Rxh6 and Black earned
the better chances in the ending, due to his superior structure and the open h-file.
10 ... hxg5 11 Bxg5 Ne7

12 d5?
This appears to be one of those lawless, ‘We-don’t-need-no-stinking badges’
moments. My opponent reasons that the goddess of attack always demands a blood
sacrifice, but my question is: what attack?
This impatient pawn sacrifice is one of those sleazy debt restructuring company
scams, where interest payments far exceed the original debt. It’s designed to open lines,
but all it really does is doom White’s far flung d-pawn, without obtaining full
compensation. Correct is 12 0-0-0 Nb6 13 Kb1 Qd7, intending to follow suit and castle
long, with balanced chances.
12 ... exd5 13 exd5 Nb6
White’s d5-pawn is virtually a cadaver, ready for dissection.
14 0-0-0 f6!
The hostile environment of the Modern Defence requires in us a survivalist’s
unflinching pragmatism, by sometimes choosing externally ugly moves. By breaking the
pin Black dooms White’s far flung d-pawn. I weigh this with the slight decrease in Black’s
king safety, which for now looks well within tolerable limits.
15 Bf4
I thought he might try the unsound 15 Nh3? fxg5 16 Nxg5 Nbxd5!, allowing White his
idea: 17 Ne6. Then Black has access to the resource 17 ... Bh6!, retaining the extra
material, with a winning position.
15 ... Nbxd5
Here I rubbed my hands together and licked my lips in the attitude of Ebenezer
Scrooge, about to count his money after a hard day at the office. After the game my
opponent felt he had compensation for the pawn, while I thought the complete opposite.
16 Bg3??
He had to try 16 Nxd5 Nxd5 17 Re1+ Kd7! 18 Bh3+ f5 19 Bg5 Qf8 20 Ne2 Re8 when
White is down a pawn and also nurses a pair of kingside isolanis on f3 and h4. Black’s
king looks safe to me and I don’t see a speck of compensation for White.

Exercise: (combination alert): White just blundered. How can Black pick up
heavy material?

Answer: Step 1: Pin White’s queen to his king, forcing f4.


16 ... Bh6! 17 f4 Nxc3
Step 2: Clear the h1-a8 diagonal for my light-squared bishop.
18 Qxc3 Bxh1
Step 3: Bon appetit. Black picks up a rook and White can resign.
19 Qxf6
Hey, I distinctly remember saying: “and White can resign”! I’m sorry Scarlett, but your
team lost the war and the new anti-bellum reality is to dig for turnips for dinner. I
suppose we all secretly believe in miracles, even in situations this wretched.
19 ... Rf8 20 Qc3 Nf5 21 Nh3
21 ... Nxg3?!
Now you may rightfully ask why I exchanged off a good knight for his rather lethargic
bishop. Did I fall prey to the positional player’s disease of oversimplification? Perhaps I’m
fear mongering, taking such a crappy bishop with a good knight. I made the decision, not
because I saw immediate danger, but more from a kind of self-preservatory habit. In my
defence, I cite the following principle: swap pieces when up on material. In such
situations the temptation is to opt for a Stalinesque purge of perceived enemies around
our king, whether they are dangerous or not!
With hindsight, far stronger is to first toss in 21 ... Qf6! and if he attempts to dodge
the swap with 22 Qb3 now is the right time to swap/simplify with 22 ... Nxg3! 23 Qxg3
Qxf4+. Of course, taking with the bishop also wins. Black will be a rook up in the ending.
22 Qxg3 Qf6 23 Bd3 Bd5
There is no reason to make the rather cowardly concession 23 ... Qxf4+?! 24 Nxf4
Bxf4+ 25 Qxf4 Rxf4 26 Bxg6+ Kf8 27 Rxh1. Sure, it’s a won ending for Black, but there
really was no reason to return any of my extra material.
24 Bxg6+ Bf7 25 Bxf7+
25 Bd3 is met with 25 ... 0-0-0.
25 ... Qxf7 26 Qg4 Qf5 0-1

Summary
Unlike Dragon lines, in the Pseudo-Dragon Black’s pieces and pawns represent a
constantly shifting target, which may confuse a belligerent opponent.

Game 3
A.Naiditsch-P.Svidler
Khanty-Mansiysk (rapid) 2009

1 e4 g6
When we play the Modern, we better have a taste for adventure in our lives.
2 d4 Bg7 3 Nc3 a6!?
3 ... d6 is the normal move order.
4 Be3 b5
This is a bit of a weird move order, but we soon transpose. Normally Black plays ...
d6, before ... a6 and ... b5.
5 f3 d6 6 Qd2 Nd7 7 h4

Tip: When White prepares the Yugoslav Attack pawn storm, intending h5 next,
we should take swift measures against it. We can either play ... Nf6, ... h5, or the
flexible ... h6, as played in the game. Don’t allow White to play h5 and g4
unchallenged,
or we may allow White’s attack/initiative to grow out of control.

7 ... h6
The most flexible move in the position. The idea is to meet h5 with the bypassing ...
g5!.
8 0-0-0
As mentioned above, meet 8 h5 with the bypassing 8 ... g5!.
8 ... Nb6
Note: Black moves one knight from d7 to make room for a future ... Nf6 and ... Nfd7
to evade a coming g4 and g5 from White.

9 Nh3
9 Nge2?! allows Black to pick off the bishop-pair with 9 ... Nc4 10 Qd3 Nxe3 11 Qxe3. I
prefer Black, whose power on the dark squares enhanced dramatically with the exchange
of knight for bishop.
9 ... Nf6 10 Nf4
White intends to bang a knight down on d5. Instead, after 10 Bd3 c6! (this move
discourages d5 from White) 11 g4 (11 e5 fails to bother Black after 11 ... Nfd5) 11 ... Qc7
12 g5 hxg5 13 Nxg5 Nh5 the chances are even, J.Suto-U.Joppich, correspondence 2007.
10 ... Nfd7!
In this way White’s e5 ideas are negated. It is superior to the 10 ... b4?! of Y.Habu-
G.Kasparov, Tokyo (rapid) 2014, when White should play 11 Ncd5 Nfxd5 12 Nxd5 Nxd5 13
exd5 a5 14 Re1. Advantage White, whose d5-pawn exerts a cramping influence on Black’s
position.
10 ... Bb7 is also possible, since it denies White his intention to play a knight to d5
and 11 e5 can be met with 11 ... b4! 12 Nce2 Nfd5 13 Nxd5 Nxd5 14 e6 fxe6 15 Nf4 Nxf4
16 Bxf4 Rb8 with a messy position typical of Modern Pseudo-Dragon lines.
11 Ncd5
This move shouldn’t scare us, since swaps benefit the more cramped side, which in
this case is Black.
11 ... Nxd5
11 ... c6 is also possible. Black looks no worse after 12 Nxb6 Nxb6.
12 exd5!?
Naiditsch hopes to cramp Black with his d5-pawn. The problem is his pawn is not all
that secure.
12 ... Nb6 13 Bd3 Bb7?!
Correct was 13 ... Qd7! 14 Rhe1 Bb7. White’s d5-pawn soon falls and I’m not so sure
the open e-file fully compensates.

Exercise (combination alert): Svidler inverts the correct move order, allowing
White a combination which exposes Black’s king. Do you see it?

Answer: Deflection/knight fork. White will regain his piece with a favourable position.
14 Bxg6! fxg6
Forced. 14 ... Nxd5?? fails to the pin/double attack shot 15 Ne6!.
15 Ne6 Qd7?
Svidler was probably in shock from missing his opponent’s idea, and blunders again.
He had to play 15 ... Qb8! 16 Nxg7+ Kd7 17 Bxh6 Qg8! when d5 falls and Black is still in
the game. In this case the damage inflicted by missing White’s combination bruises
Black’s dignity more than his actual position.
16 Qd3!!
Irreversible purpose is often the mark of a fanatic, who can be our most dangerous
enemy, since a fanatic always refuses to negotiate. Naiditsch, a natural tactician,
interprets chaos as opportunity, so he discards a safe plus and offers material instead.
The stink of the weak g6-square just won’t wash off Black’s position, as he is threatened
with mate on the move.
Naiditsch correctly concludes that Black’s king is a prize worthy of the risk of sacrifice.
This piece offer is even stronger than capturing on g7. A more practical player would play
the simpler 16 Nxg7+ Kd8 17 Qd3 Bxd5 18 Qxg6 with a winning position for White.
16 ... Kf7 17 h5!
This move enables the entry of White’s queen.
17 ... gxh5 18 Qf5+ Kg8
18 ... Ke8?? walks into 19 Qg6 mate.
19 Qg6 Rh7

Exercise: (critical decision): White has two ways to continue his attack. He can
play the clever 20 Bxh6, which is possible due to Black’s overloaded rook, or he can
continue 20 Rxh5, feeding another piece into the attack. Which one would you play?

20 Bxh6?
Answer: It isn’t easy to harness attacking energy without being singed by it in the
process. In this case the flashy way is the wrong way. White’s attack grows out of control
after 20 Rxh5! Qe8 21 Qg4 Qf7 (21 ... Kh8 is met with the crushing 22 Rdh1) 22 Rxh6!
when Black won’t survive the assault.
20 ... Qe8!
The goal of insurgency is to harass the ruling authority and keep the spirit of
disobedience alive. Svidler finds the only move to stay alive. Suddenly Black is presented
with a clean slate upon which a new future will be written.
21 Qg5 Qf7
Just in time.
22 Bxg7
22 Rxh5 can be met with 22 ... Qxh5! 23 Qxh5 Bxh6+ 24 Kb1 Bg7 25 Qg4 Nxd5. Two
pieces and a rook is probably too much for just one queen, so White must bail out with
the line 26 Ng5 Rh6 27 Ne6 Rh7 28 Ng5 with a forced repetition draw.
22 ... Rxg7 23 Nxg7 Qxg7 24 Qxh5 Nxd5
Black’s king is safe and he has two minor pieces for a rook and pawn. It is White who
now fights for a draw. The Modern is a very slippery opening, so factor in that your games
will be erratically up and down.
25 Rh4 Nf6 26 Qf5 Bc8
Not Black’s best. Correct was 26 ... Kf7! 27 Rdh1 e6 28 Qf4 Ke7 29 Rh6 Rg8. Black
unravelled smoothly and holds a clear edge.
27 Qf4 Bd7?!
27 ... Kf7! keeps the game even.
28 Rdh1 Rf8 29 g4
The advantage just swung back to White!
29 ... c6 30 Rh6 Nh5
A cute trick, but White still stands considerably better.
31 Qd2 Nf4 32 Qh2
Threatening a check on h8, followed by Rh7, winning Black’s queen.
32 ... Qg5 33 Kb1?!
The evaluation keeps swinging back and forth. White remains in control after 33
Rh8+! Kf7 34 Rxf8+ Kxf8 35 Qh8+ Kf7 36 Kd1! when Black’s king is in danger.
33 ... Kf7! 34 Re1
Now Rh7+ and Rxe7 is in the air.
34 ... Nd5?
After 34 ... Ng6! it’s anybody’s game.
Exercise (combination alert): Black’s last move allows White a combination
which wins a pawn. How?

35 Rh5?
This is the fairy tale hero who climbs the maiden’s hair to rescue her from
imprisonment in the tower, only to discover that a crone is smiling up at him (apparently
she used hair colouring to mask the white), when he reaches the top.
Answer: White missed 35 Rh7+! Ke8 (or 35 ... Kg8? 36 Rh1 Nf4 37 Rh8+ Kf7 38
Rxf8+ Kxf8 39 Qh8+ Kf7 – if Black blocks with his queen, then 40 Qh6+ picks up the
loose f4-knight – 40 Rh7+ Ke6 41 Rg7!, which wins the queen, since 41 ... Qf6?? allows
mate in two moves after 42 Qg8+ Qf7 43 Qxf7; the king resigns himself to his Cersei
Lannister-like sister’s passionate groping) 36 Qxd6, picking up a key pawn when Black’s
game begins to crack in rebellion against overburdened defensive demands.
35 ... Qf6 36 g5!? Qxd4?
Time pressure has this awful way of invalidating all our theories, and neither player is
able to correctly assimilate the data. If two super-GMs blunder this often in the Modern
Defence, then it’s pretty much assured that we will too in our games! So actually factor in
a mutually high blunder ratio when you dare to play 1 ... g6.
Exercise: (combination alert): The d4-pawn was poison for Black. White to play
and win.

37 g6+?
When we lose a game we should have won, we console ourselves with the thought:
without the painful memory of our losses, we have no way to contrast the joy of our wins.
This game is one of those crash landings where the plane spins like crazy and the
terrified passengers aboard are no longer able to differentiate between sky and earth.
White goes from winning to losing.
Answer: Winning was 37 Rh4! and if Black wants to hang on to his queen and play 37
... Qc5 (37 ... Qg7 is met with the killing pin 38 Rh7), White wins with 38 Rf4+! Ke8 (or
38 ... Nxf4 39 Qh7+ Ke8 40 Qxe7 mate) 39 Qh5+! picking up a full rook.
37 ... Ke8
Now Black’s king skips away to safety like a truant child on a sunny day, attempting to
evade the drudgery of school.
38 Rh7
The attempted combination 38 Qxd6?? fails miserably to the discovery 38 ... Nc3+ 39
bxc3 Qxd6.
38 ... Qf6
Everything is covered and White’s f- and g-pawns are chronically weak.
39 Qg1 c5 40 a3 Bf5
Svidler restores his monopoly on the light squares.
41 Rh6 Rg8 42 g7
A time pressure blunder in a lost position, but 42 Qh1 Rxg6 43 Rh2 Kd7 is hopelessly
lost for White.
42 ... Qxh6 0-1

Summary
My suggestion is to meet most h4 ideas with ... h6!. The idea is to keep the structure
flexible and meet White’s h5 ideas with the bypass ... g5!.

Game 4
D.Aldama-C.Lakdawala
San Diego (rapid) 2017

1 e4 g6
Very few of us lower our performance expectations in tournaments as we grow older.
Even in my dotage I continue to play my childhood favourite. I played the Modern for two
reasons:
1. My opponent IM Aldama leads by a half point and I was desperate for a win, so the
unbalancing effect of the Modern is perfect.
2. Sometimes it’s possible to use an opponent’s strength against him. Aldama tends to
really go crazy against any line where Black cedes the centre early on, so I pretty much
guaranteed that there would be a decisive result in the game.
2 d4 Bg7 3 Nc3 d6 4 Be3 a6 5 Qd2 b5 6 h4
White tosses in h4 before playing f3.
6 ... Nf6
Back to the Pirc, which is so closely tied to Modern that sometimes we weave from
one to the other at a moment’s notice. 6 ... h6 is also possible, but I don’t like Black after
7 f4 since White didn’t waste a tempo with a previous f3 and gets f4 in one go. Still,
Black’s game is considered playable after 7 ... Nf6 taking advantage of the g4 hole.
7 f3

Tip: Don’t be afraid of 7 Bh6 ideas when Black hasn’t yet castled kingside. Our
solution is to just take it and then play to castle queenside, or just leave the king in
the centre with a plan like 7 ... Bxh6! 8 Qxh6 Nbd7 9 f3 b4 10 Nce2 c5
11 0-0-0 Qa5 12 Kb1 c4. Black’s chances look no worse than White’s.

7 ... h5 8 Nh3
The idea behind 8 0-0-0 is to wait until Black plays ... Bb7, and only then play Nh3,
without worry of ... Bxh3. I like Tiger’s suggestion of 8 ... c6! 9 Kb1 Qc7 when Black’s c8-
bishop continues to keep a bead on the h3-square, planning ... Bxh3 if White’s knight
hops to the square. After 10 Bd3 (10 e5 Nd5 11 exd6 Qxd6 12 Ne4 Qc7 looks fine for
Black) 10 ... e5 11 dxe5 dxe5 12 Nh3 (at long last) 12 ... Bxh3 13 Rxh3 Nbd7 14 Rhh1
Bf8!? (to be considered is 14 ... Nf8, intending to reroute to e6) 15 Qf2 (E.Berg-
I.Andreasson, Lidkoping 2013), I would play 15 ... Bh6! ridding White of his bishop-pair
and sticking him with a slightly bad bishop.
8 ... Bxh3! 9 Rxh3

Tip: We can consider handing White the bishop-pair in such situations. What do
we obtain for it? We gain time, since we misplace White’s rook on h3, and we also
avoid a white knight roosting on g5 later on.

The move I played scores better than 8 ... Nbd7 9 Ng5. Black’s king will never rest
easy with a white knight sitting on g5.
9 ... c6!
I threw this move in so that White was unable to weaken my queenside light squares
with a4 and d5 ideas.
10 0-0-0 Nbd7 11 Qe1
Chess is full of frustrations, yet its study can be a creative pleasure, especially when
we put our own stamp on it, rather than mimicking others. White played this instantly,
indicating home prep. This looks like a dangerous new move in the position, the idea of
which is to play e5.

11 ... Qc7

Warning: Be alert for White’s e5 disruption tricks. 11 ... 0-0? is forcefully met with
12 e5! with a big advantage for White.

12 Kb1 c5!?
This thematic-looking move may possibly be premature, since it violates the principle:
avoid opening the position when your opponent owns the bishop-pair. When we
deliberately repeat a past negative pattern – in my case, underestimating the opponent’s
attacking potential – just before we make our questionable decision, we think to
ourselves: ‘This time things will turn out differently!’.
I sent this game to IM Jeremy Silman, and he actually liked my principle-violating
move choice, and vehemently advocated for it. His analysis follows later in the game.
Black also has:
a) 12 ... 0-0? (when we castle early, we allow White a Dragon-style prying sacrifice)
13 g4! hxg4 14 fxg4 Nxg4 15 h5 with a strong attack for White.
b) 12 ... Nb6?! 13 e5! Nfd5 14 Nxd5 Nxd5 15 e6! with advantage to White.
c) Best may be 12 ... Rc8 13 g3 0-0. We reason that if White plays g4, he lost a
tempo. Now if 14 g4!? hxg4 15 fxg4 b4! 16 g5 bxc3 17 gxf6 Nxf6 18 h5! Nxh5 19 Qxc3 c5
20 Bxa6 cxd4 21 Qxc7 Rxc7 22 Bxd4 Bxd4 23 Rxd4 Nf4 24 Rf3 g5 I don’t think Black
stands worse in this ending, which will probably end with some crazy mutual promotion
race.
13 dxc5! Nxc5

14 Bg5
After 14 Bd4 I claimed an advantage for White. Jeremy disagreed, writing: “Since I
like to save lost positions (in life and chess), let me challenge your assessment”. In the
position after 14 ... Rc8!? (allowing White’s threat) 15 Bxf6 Bxf6 16 Nd5 Qd8 17 Nxf6+
exf6 18 Rh2 0-0 19 g3 Qe7 20 Rhd2 Rc6 the computer assesses as a clear advantage for
White, while Jeremy told me he preferred Black here since White can’t win the not-so-
weak d6-pawn, and Black can build for an attack on the queenside. After having looked at
it for a long time, my revised assessment is that Black’s position is fully playable and
Jeremy out-assessed the comp!
14 ... Ncd7
Maybe I should try 14 ... Qb7! which both side-steps White’s Bxf6 and Nd5 ideas, as
well as suppresses g4. 15 e5 isn’t really a concern due to 15 ... dxe5 16 Qxe5 Ne6.
15 g4!?
Some players’ warped psyches are just greedy for danger. This is typical Aldama, who
goes all out, even when a draw wins him the tournament! Here I suggested 15 Nd5 which
the comp liked and assessed as better for White. Once again Jeremy challenged the
comp’s assessment and claimed that Black was better after 15 ... Nxd5 16 exd5 Nb6 17
c3 Na4. I think Jeremy is once again out-assessing the comp (and your writer by my
misguided association with the comp!). Having looked at this position for a long time, I
think Black has good chances of building up for an assault down the b-file, mixed with
sacrificial possibilities on b2 and c3.
15 ... Rc8
Threat: ... b4 and ... Qxc2+.
16 Bd3 Qa5
I also considered 16 ... e6 to remove Nd5 ideas, but I was nervous about entering the
complications associated with 16 ... hxg4!? 17 fxg4 Nxg4!? 18 Nd5 Qc5 19 Nxe7 and Black
is unable to play 19 ... Nf2??, since after 20 Be3! and it is White who wins.
17 gxh5
17 Bd2 is met with 17 ... b4 18 Nd5 Nxd5 19 exd5 Bc3! and I prefer Black’s game.
17 ... Rxh5!?
Perhaps the wrong recapture. Far safer is 17 ... Nxh5 18 Bd2 Qb4 19 a3 Qc5 when
Black stands no worse.
18 Bd2
He reinforces c3. On 18 f4!, right or wrong, I planned the exchange sacrifice 18 ...
Rxc3!?.
18 ... Qc7 19 f4?!
Now his centre gets loose. He should repeat the position with 19 Bg5.
19 ... Nc5
Better was 19 ... b4! 20 Na4 (20 Ne2 Nc5! is also troublesome for White) 20 ... Ra5 21
b3 Rxa4!, which is a highly promising exchange sacrifice.
20 e5!?
Note: An overtly aggressive action may in reality be masquerading as a defensive
mechanism, the way a fat kid (i.e. your writer) might make a fat joke to immunize
himself from future insults from the school bullies.

IM Aldama, sensing that he has been outplayed, is acutely aware that your innocent
writer is a man easily hoodwinked by a treacherous opponent, so he indulges in his rather
shady just-say-yes-to-everything strategy to complicate. He didn’t want to go passive
with 20 Rc1 b4 21 Nd1 Nxd3 22 cxd3 Rc5 with advantage to Black.
20 ... dxe5?!
I wish I could tell you that I play every line of the Modern Defence the way Glenn
Gould plays Bach, but this just wouldn’t be true. My move is okay, but much stronger was
20 ... Nxd3! which was more accurate.
21 Be2! b4!?
One warning/request about going over my games: please lower your expectations. 21
... Rf5! leaves White without compensation for his missing pawn.
22 Bxh5 bxc3 23 Rxc3 Nfe4!?
When we get low on time, we lack data and are obliged to play by blind feel.
Difficulties obstructing us can either discourage, or they can harden us to our purpose.
Here I indicate to my opponent that my initiative isn’t for sale. I refuse his material offer
by rejecting the line 23 ... gxh5 24 Rc4 e6 25 Bb4 (25 b4?! doesn’t work since Black
evades material loss with the simple 25 ... Qb7) 25 ... Nfd7 with an irrationally complex
position.
24 Rc4 Nd6!?
24 ... gxh5 25 Rxe4 Nxe4 26 Qxe4 Qc4 is about even.
25 Ba5! Qc6 26 Rxd6! exd6 27 Bg4
After 27 fxe5! Bxe5 (not 27 ... gxh5?? 28 exd6+ Kf8 29 Qe7+ Kg8 30 d7 and White
wins) 28 Be2 White’s bishop-pair should offer him decent chances in the open position.
27 ... f5 28 Be2?!
We are both low on the clock and my opponent begins to lose his way. 28 fxe5! Bxe5
(forced) 29 Bd1 Kd7 is balanced.
28 ... d5!
Black seizes the initiative. Note that Black never castled in the entire game, which,
oddly enough, is considered normal in Modern/Pirc situations.
29 Rb4??
When we miss a combination, we reach that awful place where we become the stage
actor who forgets his lines and notices the uncomfortable stirring within the audience.
White had to try 29 Rc3! e4 and now maybe he can try to muck it up with 30 h5! Bxc3 31
Qxc3 with dangerous compensation for the exchange, although the materialistic comp
still prefers Black.

Exercise (combination alert): Let’s take a look at the surveillance video. Black to
play and win.

Answer: Double attack.


29 ... Nb7!
Game over. Now a5 hangs, as well as the mating threat on c2. Somehow my position
reminded me of Moses’ doubt, trial, persecution, exodus, and finally redemption.
30 Rxb7
Hey I distinctly said, “as well as the mating threat on c2”! Miscalculation tends to be
either born of depression or exhaustion. This last blunder fails to alter the game’s
outcome, and it merely represents a shift from one helplessness to another. If 30 Bd1
Nxa5 31 fxe5 Nc4 and Black is up a piece with the initiative and a safe king.
30 ... Qxc2+ 0-1
Oh no, no, no, thank you. Hold your applause and please sit down. Thank you. He
gets backranked. In such situations, memories of my own backrankgate arise: I was in
7th grade and about to win a junior tournament where I was up heavy material versus
my undeserving opponent, who pulled a similar back-rank cheapo on me.

Summary
In situations where we committed to ... h5 and the opponent plays Nh3, then consider
giving up the bishop-pair with ... Bxh3!. For it, we obtain the following:
1. We deprive White of a powerful attacking knight on g5.
2. We misplace the opponent’s rook on h3.

Game 5
P.Darini-R.Mamedov
World Rapid Championship, Doha 2016

1 d4 g6 2 e4 Bg7 3 Nc3 a6 4 Be3 d6 5 Qd2

Note: When White sets up with his queen to d2 in conjunction with Nf3, Black
is sometimes given a future opportunity to chase down White’s bishop with a
future ... Nf6 and ... Ng4 manoeuvre, as seen in the game.

If White wants to preserve his dark-squared bishop, he can opt for:


a) 5 a4 b6! where we enter the Hippopotamus formation.
b) 5 f4 b5 6 Nf3 Bb7 7 Bd3 transposes to the Austrian Attack.
c) 5 Nf3 b5 6 Bd3 Bb7 7 0-0 Nd7 8 a4 b4 9 Ne2 Ngf6 10 Ng3 (10 e5!? is unplayed,
after which 10 ... Nd5 11 e6 fxe6 12 Ng5 Nf8 looks unclear; White may have full
compensation for the pawn) 10 ... c5 11 c3 bxc3 12 bxc3 0-0 and Black equalized in
H.Stevic-I.Saric, Bol 2015. Following 13 d5 Qc7 14 Nd2 e6 15 c4 a5 16 Rc1 Ba6 17 Nb3
exd5 18 exd5 Black’s pressure on c4 and a4, coupled with ideas of a future ... Rb8 and ...
Rb4, look like they outweigh White’s central space.
5 ... Nd7
5 ... b5 can also be played here.
6 Nf3
You might wonder why isn’t this game in the next chapter on the Classical set-up
versus the Modern. The difference is that in this version White posted his queen on d2
and bishop on e3, which is the guidepost for this chapter. This is the starting position of
the Pseudo-Lopez set-up for White. He plans to transfer his c3-knight to g3, via e2. The
problem with this strategy is that it allows Black a chance to pick up the bishop-pair and
equalize fully.
6 ... b5

7 a4
White seeks to shake up Black’s queenside structure.
Note: Castling queenside, in my opinion, is not a good fit for White when Nf3 has
been played, since he is unable to reinforce the e4-pawn with f3.

For example, 7 0-0-0?! Bb7 8 Bd3 c5 9 dxc5 (9 d5? allows Black a nasty attack after 9
... Qa5 10 Kb1 c4 11 Bf1 b4 12 Ne2 c3) 9 ... Nxc5 10 Bxc5 (10 Kb1 Rc8 looks like a very
pleasant Sicilian Dragon-like set-up for Black) 10 ... dxc5 11 Qe2?! (threat: Bxb5+,
winning Black’s queen) 11 ... Qa5! (dual threats: ... c4, trapping White’s bishop, and also
... b4, followed by ... Qxa2) 12 e5 b4 13 Ne4 Qxa2 with a clear advantage for Black,
A.Boruchovsky-E.Romanov, Aix-les-Bains 2011.
7 ... b4 8 Ne2
White follows his Ruy Lopez outlook, transferring the knight to g3.
8 ... Ngf6 9 Ng3
Now White’s queen threatens Black’s b-pawn ...
9 ... Ng4!

... which Black ignores. This is a promising pawn sacrifice for Black.
10 Bf4!

Tip: Sometimes our strongest plan is to take no action at all.

I think White is wise to decline. 10 Qxb4?! allows Black the initiative, a development
lead, the bishop-pair and power on the dark squares, all just for one measly pawn after
10 ... Nxe3 11 fxe3 Rb8. I would take Black any day of the week, even if for now his
threats are implied, more than actually stated.
10 ... e5
The goal is to eventually chase down White’s dark-squared bishop.
11 Bg5 f6 12 Be3
Nobody has tried 12 Bh4!? which leads to a bit of a traffic jam for White’s pieces on
the kingside.
12 ... Nxe3 13 Qxe3
13 fxe3 is annoyingly met with 13 ... Bh6.
13 ... 0-0 14 Bc4+ Kh8 15 0-0 Nb6 16 Bb3 a5 17 Rfd1?!
Correct was 17 dxe5 fxe5 when even here, I slightly prefer Black, due to the bishop-
pair and open f-file.
17 ... Qe7
Black missed 17 ... Bg4! 18 dxe5 (not 18 h3? Bxf3 19 Qxf3 exd4 and White is unable
to recapture d4, due to ... f5!) 18 ... fxe5 when ... Bxf3 is coming, with a serious disability
for White.
18 h4?!
Once again White should release the tension with 18 dxe5.

Exercise (planning): How can Black take the initiative by following a strategic
principle?

18 ... h5?!
Answer: Black should play on the principle: open the position when you own the
bishop-pair, with 18 ... exd4!. This move rouses Black’s slumbering pieces into full
consciousness: 19 Nxd4 f5! (once again following the principle) 20 exf5 Qxe3 21 fxe3 c5!
(and again!) 22 Nf3 c4 23 Rxd6 cxb3 24 cxb3 (after 24 Rxb6?? Bxb2 25 Rf1 bxc2 26 Rc6
b3 Black’s two connected passed pawns win easily) 24 ... Nd7 25 fxg6 Ne5. On paper,
White has four pawns for the piece. In reality, his pawns are sitting-duck targets which
soon fall.
19 dxe5 dxe5 20 Nd2 Bg4
Inducing f3, which slightly weakens White’s dark squares.
21 f3 Be6 22 Bxe6 Qxe6 23 Ngf1
White begins to lose the initiative after this move. He should challenge Black’s
queenside pawns with 23 c3.
23 ... Rfd8
Black stands better after 23 ... Qc6! 24 b3 f5.
24 Qb3
Once again he should play 24 c3!.
24 ... Qe7 25 Ne3 Qc5 26 Ndf1 Bh6
Black begins to seize a dark-square initiative.
27 Kf2 Kg7 28 Ke2
White’s king is magically drawn to the middle, this time to break the pin on his e3-
knight.
28 ... Rab8
Played to discourage c3.
29 Rd3 Rd4!
Principle: operate on the colour of your strength, in this case the dark squares.
30 Rad1 Rbd8 31 c3! Rxd3 32 Rxd3 Rxd3 33 Kxd3 Qd6+ 34 Ke2 bxc3
Exercise (critical decision): White escapes his difficulties, on condition he finds
the correct recapture. Should he play 35 bxc3 or should he recapture with his queen?

35 bxc3?
Answer: The enfeeblement of White’s queenside structure is the tipping point. He
should hold the game with 35 Qxc3! Nxa4 36 Qxa5.
35 ... Qc6!
This move overloads White’s queen and wins a pawn.
36 Qa3
White puts up greater resistance with 36 Qb5 Qxc3 37 Qe8 Qb2+ 38 Nd2 Qb4.
36 ... Nxa4
Not only did Black win a pawn, but he also acquired a passed a-pawn.
37 Qe7+
37 Kf2 offered better chances to hold the game.
37 ... Kg8 38 c4 Bf8 39 Qd8 Kf7 40 Nd2
40 Qd5+ Qxd5 41 cxd5 Nb6 is a lost ending for White.
40 ... Qd6!
Principle: force swaps when ahead on material.
41 Qc8
The position’s logic may argue for cautious restraint, while our ambition whispers
reckless schemes into our ear. White absolutely had to trade queens at this point, since
Black’s last move was designed to compel, rather than gently persuade. 41 Qxd6 lasts
longer, but should lose in the end as well.
Exercise (combination alert): White’s king is the guy with the red laser trained on
his forehead, about to get whacked by the mafia hitman (in this case, hitwoman).
It’s not even really a combination, more a forcing pair of moves. Black to play and
win:

Answer: Mating net/material loss.


41 ... Nc3+! 42 Ke1 Qd3!
Now the white king is threatened with a beating, followed by the theft of his lunch
money. White must hand over both knights if he wants to avoid mate.
43 Qxc7+
A spite check is in actuality a coping mechanism to keep our bruised self-esteem
salved and bandaged, if only temporarily.
43 ... Kg8 0-1
White’s rebuttal falls short, since he just ran out of checks.

Summary
When White’s queen and bishop line up on d2 and e3, always be on the lookout for the ...
Ng4! idea, which bags the bishop-pair and dark squares.
Chapter Two
The Classical Variation
1 e4 g6 2 d4 Bg7 3 Nc3 d6 4 Nf3 a6

In this chapter we look at the old school Classical line, where White follows the
principle: develop knights before bishops. So White brings his knights out early to c3 and
f3, while remaining fluid with the rest of his or her piece placement. We once again go
with our ... a6, ... b5 set-up and should reach a perfectly acceptable position, since
White’s set-up is too mild to worry us much.

Game 6
H.Stevic-I.Nepomniachtchi
European Individual Championship, Aix-les-Bains 2011

1 e4 g6 2 d4 Bg7
Here is an example of the ... c6 version versus the Classical: 2 ... d6 3 Nc3 c6 4 a4
Bg7 5 Be3 Nf6 6 h3 Nbd7 7 Nf3 e5 8 Be2 0-0 9 Qd2 Qe7 10 dxe5 dxe5 11 Nh2?! (this
decentralizing/attacking move doesn’t work out well for White; 11 Rd1 is better). 11 ...
Nc5 12 Bf3 Ne6 13 Ne2 Rd8 14 Qc1 h5 (preventing Ng4; White’s souring position gets
worse after his next move) 15 g4?! h4! 16 Bg2 (Black’s h-pawn is safe since 16 Bg5? Nxg5
17 Qxg5 Qb4+ is awful for White) 16 ... Nh7 (seizing control over g5) 17 0-0 b6!. Now ...
Ba6 is coming and Black achieved a strategically winning position, R.Bruno-C.Lakdawala,
San Diego (rapid) 2005.
3 Nc3
Another typical set-up is seen after 3 Nf3 d6 4 h3 a6 5 a4 b6 6 Nc3 Bb7 7 Be3 Nd7 8
Qd2 e6 9 Be2 h6 10 0-0-0!?.

Note: As I mentioned before in the book, I feel that White’s a4 and queenside
castling is an incompatible mix, since his king feels unsafe and the e4-pawn lacks
the normal f3 reinforcement.

Indeed, following 10 ... Ne7 11 h4 b5! 12 Bd3 (12 axb5 axb5 13 Bxb5? Ra1+ 14 Nb1
Bxe4 is heavily in Black’s favour) 12 ... bxa4 (I want to open the b-file) 13 Nxa4 Nf6 14
d5!? (just as in the second game of the book, White sacrifices a pawn to open lines; I am
sceptical of his full compensation) 14 ... exd5 15 exd5 Nexd5 16 Rde1 Kf8! White didn’t
have enough for his pawn, B.Stamper-C.Lakdawala, San Diego (rapid) 2013.
3 ... d6 4 Nf3 a6
Once again, there is no law against the older ... c6 plan: 4 ... c6 5 a4 Nf6 6 Be2 0-0 7
0-0 a5 8 h3 Na6.
Tip: In the ... c6 version, when White plays a4, we can respond with ... a5, ... Na6
and ... Nb4, when it is annoyingly difficult for White to eject Black’s knight.

After 9 Be3 Nb4 10 Qd2 e5 11 Rfd1 exd4 (this is an important idea borrowed from the
King’s Indian Defence; don’t be afraid to hand White central control with ... exd4 ideas,
since our enhanced piece play compensates White’s space) 12 Nxd4 Re8 13 Bf3 Nd7 14
Rac1 Qe7 Black’s active pieces and pressure on e4 fully compensate the potential
weakness of d6, F.Thornally-C.Lakdawala, San Francisco 2002.
5 Be2
GM Gawain Jones calls this White’s most critical move. Next game we look at 5 a4
suppressing ... b5.
Instead, 5 Be3 b5 6 Bd3 Bb7 7 0-0 Nd7 8 a4 b4 9 Ne2 c5 10 c3 bxc3 11 bxc3 Ngf6 12
Ng3 (here we are again with the Pseudo-Lopez strategy, but note that here Black doesn’t
have the ... Ng4 manoeuvre from last chapter, mainly since White hasn’t hemmed in his
dark-squared bishop with Qd2) 12 ... 0-0 is once again Stevic-Saric from note ‘c’ to
White’s 5th move in Game 5.
5 ... b5
5 ... Nd7?! is a slightly inaccurate move order, which my opponent failed to exploit: 6
0-0 b5 7 Be3?! (Tiger Hillarp suggests 7 d5! intending Nd4, to punish Black for the c6
hole) 7 ... Bb7 (now all is well with Black’s position again) 8 d5 Ngf6 (threat: ... b4 and ...
Nxe4) 9 a3 c6! (Black eliminates the c6 hole before White has time for Nd4) 10 dxc6 Bxc6
11 Nd4! Bb7 (11 ... Bxe4?! 12 Nxe4 Nxe4 13 c4 bxc4 14 Bxc4 offers White huge
compensation for the pawn) 12 f3 0-0 13 Qd2 Rc8 14 Rad1 Qc7 15 Bh6.
Tip: When White castles kingside, don’t live in fear of Bh6, even if castled short,
since if White’s attempted attack comes to nothing, then all our opponent has done
is to hand him or herself a bad remaining bishop, as seen in this note.

After 15 ... Rfd8 16 Bxg7 Kxg7 17 Kh1 Nb6 18 Qg5 e6 19 Nb3 h6 20 Qg3 Nh5 21 Qh4
e5 22 Rd2 Nf4 23 Bd3 d5 Black achieved a favourable Najdorf structure, with a thematic
... d5 central break, B.Baker-C.Lakdawala, San Diego (rapid) 2016.
6 a4
This disruption is White’s best shot at an edge, according to GM Jones. 6 0-0 is too
mild to earn an edge after 6 ... Bb7 7 Re1 Nd7 8 Bf1 c5 with a pleasant position for Black.
6 ... b4 7 Nd5 a5
Also fine is 7 ... Bb7, intending to meet 8 Nxb4!? with 8 ... Bxe4. It feels to me like
Black got the better of the deal, having swapped off his lesser b-pawn for White’s more
valuable e-pawn, H.Stevic-V.Bukal, Zadar 2010.
8 c3!
Principle: create confrontation when leading in development.
8 ... e6 9 Ne3 Ne7
A new move. 9 ... Bb7 was played previously.
10 0-0 Bb7 11 e5!?
If the colours we see are not perceived as the same by our dogs, then where lies
reality? Stevic decides he will not waste a moment on the usual opening pleasantries,
and instead opts for immediate confrontation. A move like this wouldn’t even be
considered by a safety-first positional player like me, since I interpret it as a temporary
sugar high, at the cost of long-term weakening. However, to an optimist/aggressive
player, it feels like a reasonable gamble.
This attempt to gain space is tricky, since White’s far flung e-pawn is now at risk. If
the move doesn’t quite count as the pure drug of sacrifice, then it certainly qualifies as a
derivative. It basically boils down to the question of just what price you are willing to pay
to get your wishes. White can play a bit calmer with 11 Bd3, the less committal
alternative.
11 ... dxe5 12 dxe5!?
The thematic continuation of his previous move. Eventually Stevic is unable to outrun
the chain of consequences stemming from his 12th move decision. Safer is 12 Nxe5, but
Stevic didn’t play his risky 11th move just to back down with a safe option on move 12.
12 ... Qxd1
The swap is logical, since Black can try and exploit the potential weakness of White’s
advanced e-pawn in an ending.
13 Rxd1 Nd5?!
I’m not sure why Nepo rejected the straightforward continuation 13 ... bxc3! 14 bxc3
Nd7 15 Bb5 Bc6. Now if White attempts to protect his weak e-pawn with 16 Nc4 Bxb5 17
axb5 Nd5 18 Bd2 a4 he reaches an unpleasant ending, since he is tied down both by
Black’s passed a-pawn and his need to defend e5.
14 Bb5+
Maybe White should consider 14 Nxd5 Bxd5 15 cxb4. This move gains White a passed
a-pawn, but not necessarily the advantage.
14 ... c6 15 Bf1
15 Nxd5 exd5 creates an unbalanced ending with opposing wing majorities.
15 ... Nd7
15 ... bxc3 once again looks promising for Black. White must either hand over a pawn,
or face 16 Nxd5 cxd5 17 Bb5+ Nd7 18 bxc3 Ba6 when I like Black’s structure in the
ending.
16 Nc4
Not only protecting e5, but also threatening a fork on d6.
16 ... 0-0 17 Nd6?!
17 cxb4 axb4 18 a5 Ba6 is approximately even.

Exercise (combination alert): White threatens Black’s b7-bishop, yet


Nepomniachtchi didn’t move it. What did he play to seize the advantage?

Answer: Zwischenzug. Black can leave his bishop hanging.


17 ... bxc3! 18 bxc3!
An unfortunate necessity. Forced moves can still be good ones. White must avoid 18
Nxb7? Rfb8 regaining the lost piece, with a clear strategic advantage, since after 19 bxc3
Rxb7 White’s a-, c- and e-pawns are all targets, and at least one of them soon falls.
18 ... Ba6
Hoping to swap away his bad bishop.
19 Bxa6!?
White should try 19 c4.

Exercise (combination alert): Black once again has access to a mini


combination which helps his position. What should he play?

19 ... Rxa6?!
Black’s intent feels like it zigs and zags, rather than moves in a straight line. He still
retains an edge after this move, but missed the stronger continuation.
Answer: Zwischenzug (again!): 19 ... Nxc3! 20 Bb7 Nxd1 21 Bd2 Nb2 22 Bxa8 Rxa8
and once again, Black is up a pawn.
20 Nc4 Rb8 21 Bd2 Nc5
Reminding White that he is tied down to his weakness on a4.
22 Rdb1
Ah, yes, the wished for the end is justified by our lowbrow means. White sets up a
cheapo.
22 ... Rxb1+ 23 Rxb1 Ra8
The hasty 23 ... Nxa4?? is horribly punished with 24 Rb8+ Bf8 25 Bh6 with mate
coming. When our opponent falls for such a whopping cheapo, we become the lucky
smuggler, who against all the odds successfully sneaks drugs hidden in body cavities past
the looming airport sniffer dog, who suffers from a cold, a runny nose, and is just having
an off olfactory day.
24 g3?
The flaw with this move centres around proportionality. It’s made with the thought:
with each concession we grant, we become victims of blackmail, where each instalment is
merely a down payment on an unpayable total. Yet the move remains in violation of the
principle: pawns which may be sacrificed in a complex middlegame, should be preserved
in an ending, since in this portion of the game, they greatly increase in value. White had
to play the distastefully passive 24 Ra1. The rook resigns himself to the depressing
question: ‘Am I destined for more in life than mere servitude and obsequiousness?’. The
answer unfortunately is: ‘No’.
24 ... Bf8!?
I wouldn’t give White a second chance to protect the a-pawn, and would play 24 ...
Nxa4 25 Nxa5 (playing on Black’s weak back rank) 25 ... Naxc3 26 Rb2 c5 with an extra
pawn for Black.
25 Nd4
This time 25 Ra1 loses a pawn to 25 ... Nb3 which removes the defender of c3.
25 ... Nxa4
At last.
26 Ra1 Ndb6!
This messy ending is the Winter garden maze from The Shining, where both sides in
crazy Jack Nicholson fashion seem to be continually retracing their steps, desperate to
locate the exit before they freeze to death. 26 ... Naxc3 is also possible: 27 Bxc3 Nxc3 28
Rxa5 Rc8 29 Ra6 c5 30 Nc6 Nd5 31 Nd6 Nb4! 32 Nxc8 Nxa6 33 Nb6 and Black is a pawn
up, but his position is far from easy to convert.
27 Nb3?!
White misses his best shot to draw with 27 Nxb6! Nxb6 28 Nxc6 Nc4 29 Bc1 Rc8! 30
Nd4 Nxe5 31 Rxa5 Rxc3. White has chances to hold the game, since Black’s remaining
pawns are on the same side of the board.
27 ... Nxc4 28 Rxa4 Nxe5 29 Kg2 Rb8!
Each of White’s attempts to reach a drawn ending is thwarted/duplicated by a clever
defensive parry, in perfect synchronicity.

Tip: Remember this geometric theme, where Black forces his opponent to capture
a5 with his knight, rather than rook, leading to an unpleasant pin.

30 Nxa5
Perhaps he should try the desperate 30 Nd4!?.
30 ... Ra8!
Now White must wiggle to escape the pin.
31 c4 Bc5
Missing 31 ... c5! 32 Bc3 Bg7 and if White attempts 33 Ra1 then 33 ... Nxc4! wins a
second pawn.
32 Bc3 Nd3
Targeting f2.
33 Ra1
33 f4 Bf8! 34 Ra1 Bg7! 35 Nb3 Rxa1 36 Bxa1 Bxa1 37 Nxa1 leaves White down a
pawn in a lost knight ending.
33 ... Nxf2 34 Nxc6 Rxa1 35 Bxa1 Ng4!
Threat: ... Ne3+ and ... Nxc4. The ending is a straightforward win for Black since
White’s passed pawn is blockaded, while Black eventually generates two connected
passed pawns on the kingside.
36 Ne5
Such a move is shorthand for desperation, driven by a combination of compulsion and
disgust, since simplification equals concession to the material-down side.
36 ... Nxe5 37 Bxe5 f5 38 Kf3 Kf7 39 h4
Technically this is unwise, since it allows Black to fix his h-pawn on the same colour as
his remaining bishop. Still, avoiding the move wouldn’t have saved him.
39 ... Ke7 40 g4 Bd6
Breaking White’s blockade.
41 Bc3 h5!

Principle: fix your opponent’s pawns on the same colour as your opponent’s remaining
bishop.
42 gxh5 gxh5 43 Be1 e5 44 Ke3 Kd7 45 Bc3 Ke6 46 Be1 Be7
Reminding White that he must now eternally watch over his weak h-pawn.
47 Kf3 Bf6 48 Ke3 Kd6 49 Kd3 Bd8 50 Bb4+ Ke6 51 Be1 Bb6 52 Ke2 f4 0-1
Black’s e- and f-pawns are the Kenyan marathoners who finish way ahead of the pack
and don’t even look tired. ... Kf5 is coming, after which White’s two passers roll.

Note: At a GM level this position is an easy win for Black, but at club level Black can
still make the terrible mistake of allowing White to sacrifice his bishop for both
black passed pawns, after which it is a technical draw, since Black is unable to win
with the wrong-coloured rook pawn and his bishop, which doesn’t cover the
queening square. Here is Black’s winning technique: 53 Kf3 Kf5 54 Bb4 e4+
55 Ke2 f3+ 56 Kd2 e3+ 57 Kd3 e2 58 Kd2 (58 c5 is met with the overload
trick 58 ... Bxc5!) 58 ... Bf2 59 Kd3 e1Q.

Summary
In my opinion the Classical is too mild a line to worry Black in the Modern.

Game 7
T.Toh-I.Rogers
Singapore 1997

1 d4 g6 2 e4 Bg7 3 Nc3 d6 4 Nf3 a6


We have already examined the permutations of 4 ... c6 in the notes to previous
games.
5 a4

White suppresses our natural ... b5 expansion idea. In this case I suggest we go with
the Hippopotamus formation.

Did you know? The Hippopotamus is a shifty formation, which consists of a double
fianchetto and also black knights on e7 and d7, with ... e6 and ... d6 played.
Essentially
we refuse (for now) to move any pawns past the third rank! Its hidden power lies in
its
flexibility, coupled with its ability to frustrate White’s attempts to open the game.

5 ... b6
Instead, 5 ... Nf6 transposes to Pirc, while 5 ... Bg4 is also a reliable but boring path, if
you are okay with a drawish ending after 6 Be2 Nc6 7 Be3 e5 8 dxe5 (8 d5 Bxf3 9 Bxf3
Nd4 is okay for Black) 8 ... dxe5 9 Qxd8+ Rxd8 10 Nd5 Rd7. Maybe White can claim a
microbe of an edge, but Black’s chances to draw far outweigh White’s winning chances.
6 Be2
With 6 Bc4 e6 as always, we meet any c4 posting from White’s bishop with our
blunting ... e6: for example, 7 0-0 Ne7 8 Qe2 h6 9 d5 (and of course here we bypass) 9 ...
e5 10 Ne1 0-0 11 Nd3 Nd7 12 Nb4 (White attacks a6, while hoping to exploit the c6 hole)
12 ... Nc5 13 Nc6!? Qe8 (to be considered was 13 ... Nxc6 14 dxc6 Ne6) 14 b4!? Nxc6 15
dxc6 Ne6 16 Nd5 Nd4 17 Qd3 Qd8 (the chances look balanced until White’s hasty next
move) 18 b5? (White gains decent compensation for the pawn after 18 Be3 Nxc6 19 b5)
18 ... axb5 19 Bxb5 Nxb5 20 Qxb5 Ba6 and Black picked up a full exchange, which White’s
powerful d5-knight was unable to offset, Lou Yiping-V.Malakhov, Chinese League 2013.
6 ... Bb7 7 Be3 Nd7 8 0-0 e6
Continuing the Hippo path.
9 h3 Ne7 10 Qd2 h6

Tip: In Hippo positions where White lines up with Be3 and Qd2, play ... h6 to
prevent White’s intended Bh6. Also a second tip, free of charge: be warned that
the comps all favour White’s game, giving it between a plus equal and a plus over
minus. My advice is to ignore their Reinfeldian viewpoint, that White stands clearly
better due to the extra central space.

In non-forcing positions, it’s a mistake to turn our thinking process over to the
machines, which in the end, enslave us by making all our decisions for us. In Frank
Herbert’s Dune, the scriptures wisely state: “Thou shalt not make a machine to
counterfeit a human mind.” If you trust your human writer over the comps, then I say
Black has excellent dynamic possibilities, if not full equality.
11 Bd3 c5!?
This confrontational central break looks a little early to me, since Black remains
behind in development. I would be more inclined to try 11 ... g5!, intending ... Ng6.
12 dxc5
I would maintain the tension with 12 Rad1.
12 ... dxc5
Also playable is the Open Sicilian-like capture with 12 ... Nxc5 13 Rfd1 Rc8!. Black
achieved dynamic equality, but he must avoid 13 ... Qc7? 14 Bb5+!.
13 Rad1 g5!

Black finally reveals his intent, transitioning from covert to overt. Note how from here
on Rogers begins to seize control over the central dark squares.
14 Qe2 Ng6?!
14 ... Qc7! is more accurate.
15 Nd2?!
White misses a chance to break up Black’s queenside pawns with 15 a5!. Black is
unable to bypass with 15 ... b5? since White has the promising sacrifice 16 Nxb5!.
15 ... Qc7
Now all is well in Black’s camp.
16 Nc4
Now after 16 a5? b5 either sacrifice on b5 is unsound for White.
16 ... 0-0 17 Bc1 Rfd8
Note how Rogers retains his a8-rook on its original square, in order to reinforce his a6-
pawn.
18 Qh5
This fails to intimidate Black, whose king remains relatively secure, mainly due to his
dark-square power.
18 ... Nde5
Challenging White’s best piece, his c4-knight.
19 Nxe5 Qxe5
Now Black holds an iron grip on the central dark squares, which adds to White’s
growing list of strategic insecurities.
20 g3!?
This is an idea suggestive of makeshift improvisation, since normal paths look
unpleasant. He toys with f4 ideas, at the peril of his soon-to-be-exposed king. 20 Be3 Rd7
allows Black to build quietly.
20 ... Qc7 21 Rfe1
White may have planned 21 f4? and then realized that Black has 21 ... c4! 22 Be2 b5!
(Black’s b-pawn is tactically protected due to a queen check on either b6 or c5) 23 axb5
axb5 24 Rxd8+ Rxd8. The undermining ... b4 is coming and White’s position collapses.
21 ... Rd7 22 Bf1
22 h4? harms White more than it helps, after 22 ... gxh4 23 gxh4 c4 24 Bf1 Rxd1 25
Rxd1 Kh7. White’s king has been exposed far more than Black’s.
22 ... Rad8 23 Rxd7 Rxd7 24 Bg2 Bc6!

Tip: Be satisfied with incremental progress created by easily achievable goals,


rather than wishes you would normally ask of a genie who appeared from a lamp.
In this case GM Rogers prepares to activate and disrupt on the queenside with ... b5
next.
25 Bf1 Bb7?!
There was no need for this. He can play 25 ... b5! after all: 26 axb5 axb5 when 27
Nxb5?! is met with 27 ... Qa5! with a double attack on b5 and e1. After 28 Nc3 Bxc3! 29
bxc3 Qxc3 30 Rd1 Qxc2 31 Rxd7 Bxd7 32 Be3 Bc6! Black is up a clean pawn and should
consolidate.
26 Bg2 Qc6 27 Rd1
He should try 27 Bf1 to suppress Black’s ... b5 intent.
27 ... Rxd1+ 28 Qxd1 Bd4

Rogers plays it safe. In my opinion, most Modern Defence players are far too in love
with their dark-squared bishops. I would try the radical 28 ... Bxc3! 29 bxc3 Qxa4. Black
wins a pawn and inflicted structural damage. I just don’t believe that Black will be
checkmated due to the absence of his protector of the dark squares.
29 Qh5 Kg7 30 h4 f6 31 Bd2?! Qd6!
Threatening both ... Bxf2+ and also ... Qxg3.
32 Qf3 Bc6! 33 hxg5 hxg5 34 Qd3!
White avoids a clever trap. 34 Bf1? suppressing ... b5 is met with 34 ... Ne5 35 Qe2
and now Black has the startling shot 35 ... Nc4! threatening ... Qxg3+. After 36 Kh2 Bxf2!
(this second discovered attack shot ends resistance) 37 Qxf2 Qxd2 38 Qxd2 Nxd2 White is
busted.
34 ... Ne5
Stronger was to disrupt White’s queen with 34 ... c4!.
35 Qe2 c4 36 Be3 Bxe3 37 Qxe3 Qb4!
Rogers continues to hack away at freezer ice with a screwdriver, targeting b2 and a4
simultaneously.
38 Qc1
This is an awkward contrivance and this act of contrition isn’t going to right past
strategic wrongs. Compared to Black’s queen, White’s queen is the manager of Burger
King, who hopes to network with investment bankers and senators. Equally discouraging
is 38 f4!? Ng4 39 Qe2 Qc5+ 40 Kh1 Ne3 41 Bf3 Qd4 42 fxg5 fxg5 when Black remains in
control.
38 ... Bxa4 39 Nxa4 Qxa4 40 f4
White makes a desperate fling at Black’s semi-exposed king, hoping to generate
enough play to perhaps deliver perpetual check.
40 ... Nf7
Also strong is 40 ... gxf4 41 gxf4 Ng6.
41 e5 fxe5! 42 fxg5 Qb4 43 Kh2 Qe7
Dooming White’s extended g5-pawn. Now it’s just a matter of Black being careful and
avoiding perpetual check.
44 Qe3 b5 45 Qb6 Qxg5 46 Qxa6

Exercise (combination alert): How did Black pick up a second pawn?

Answer: Attraction/double attack.


46 ... Qh6+!
Step 1: Force White’s king to g1.
47 Kg1
After 47 Bh3?? Ng5 48 Qb7+ Kg6 49 Qg2 e4 Black soon reaches a winning king and
pawn ending, but now comes Step 2: Double attack.
47 ... Qc1+ 48 Kh2 Qxb2
The queen conveniently protects b5.
49 Qxe6 Qxc2
Step 3: Pick up White’s c-pawn as well. When we take a head count, we notice that
White is short by two clean pawns, with no perpetual check ideas in sight.
50 Qd7 c3 51 Kg1 Qc1+ 52 Kf2 c2 0-1
The c-pawn nearly reaches the finish line on c1. 53 Qg4+ is met with the block 53 ... Qg5.

Summary
Note the Hippopotamus plan displayed in this game of ... h6, ... g5!, ... Ng6, seizing more
and more control over the central dark squares.
Chapter Three
The Jump to Nowhere
1 e4 g6 2 d4 Bg7 3 Nc3 d6 4 Bg5

On the fourth move, White plays Bg5, attacking thin air. The idea is to pin our e-pawn,
while leaving open the possibility of a future Qd2, or even f4. The potential flaw within
White’s idea is that it weakens control over d4, which we hit straight away.
In this chapter, I advocate the blasphemous plan of giving away our dark-squared
bishop, the darling only child of openings like the Modern and Dragon. I realize that to
most of us this opening mutation is a bit Island-of-Dr-Moreau creepy to our offended
sensibilities. Why do we hand White this almost sacred piece for the c3-knight?
The reason is that we inflict structural damage by handing White doubled isolated c-
pawns and an isolated a-pawn. This means that if White’s attack on our weakened
kingside dark squares fails, he will be certainly left with a busted ending. Maybe I’m
weighing strategic factors too heavily, but in my experience I haven’t yet been mated and
White’s queenside pawns keep falling.

Game 8
J.Humphrey-C.Lakdawala
San Diego (rapid) 2012
1 e4 g6 2 d4 Bg7 3 Nc3 d6 4 Bg5

Hey, what exactly are you doing there on g5?

Tip: The weakness behind White’s aggressive last move is that it fails to reinforce d4,
since White’s dark-squared bishop is nearly always posted on e3 in such Modern
situations. So how do we exploit this? By striking at White’s centre with ... c5 at once.

4 ... c5
4 ... c6 and 4 ... a6 are Black’s main lines, but over the past few years I have come to
the conclusion that the best method of exploiting White’s ‘Jump to Nowhere’ line is to
immediately challenge d4, since White’s dark-squared bishop is conveniently missing from
e3.
5 Bb5+
The idea is to develop rapidly, much like the Moscow variation of the Sicilian, followed
by Nge2. Alternatives:
a) 5 d5 (as a Trompowsky player, I have had trouble as White with this transposition
to a Schmidt Benoni, since White’s dark-squared bishop may once again be misplaced on
g5) 5 ... Nf6 6 Nf3 0-0 7 a4 h6 8 Bf4 g5!? 9 Bd2?! (correct was 9 Bg3 not fearing the loss
of the bishop-pair after 9 ... Nh5!? 10 Nd2 Nxg3 11 hxg3; White’s enhanced control over
the light squares, coupled with Black’s slightly loose king, make up for White’s weakness
on the dark squares) 9 ... e6 10 Be2 exd5 11 exd5 Na6 12 0-0 Nb4 13 Bc4?! (correct is 13
h4) 13 ... Bf5 14 Ne1 Re8 15 Nd3 a6 16 Nxb4?! (16 a5 should be played) 16 ... cxb4 17
Ne2 Rc8 18 Bb3 was C.Lakdawala-D.Aldama, San Diego (rapid) 2014. Black seizes a
fearsome initiative after 18 ... Ne4!.
b) 5 Nf3!?.

Note: This move transposes the game to a line of the Sicilian Dragon which has
great similarities to the Modern.

Following 5 ... cxd4 6 Nxd4 Nc6 7 Nb3 Nf6 8 Qd2!? (8 Be2 0-0 9 0-0 is a line of the
Sicilian Dragon, considered not too threatening for Black), as in K.Trujillo-C.Lakdawala,
San Diego (rapid) 2015, Black has even chances in the opposite-wing attack position with
8 ... 0-0 9 0-0-0.
c) Next game we look at 5 dxc5.
5 ... Bd7 6 Bxd7+
6 dxc5?? hangs a piece to the undermining/zwischenzug 6 ... Bxc3+ 7 bxc3 Bxb5.
6 ... Nxd7 7 Nge2
White insists on playing the position like an Open Sicilian versus Dragon set-up.
There’s also 7 d5 Bxc3+! 8 bxc3 Qa5.
Tip: In this chapter I often suggest we chop our opponent’s c3-knight, if in doing so
we inflict structural damage upon our opponent.

After 9 Ne2 h6 10 Be3 Ngf6 11 f3 Ne5 12 0-0 Nc4 13 Bc1 b5 I prefer Black’s superior
structure and fantastic c4-knight over White’s theoretical chances to attack much later on.
7 ... cxd4 8 Nxd4 Qb6! 9 Nb3?!
This innocent-looking move is the woman who toys with her suitor’s emotions, just for
the malicious fun of it, and then she unexpectedly falls for him. The move is strategically
suspect, due to my response. White also has:
a) After 9 Nd5? Qxd4 10 Qxd4 Bxd4 11 Nc7+ Kd8 12 Nxa8 White may be up the
exchange, but this is a temporary state, since the a8-knight is eventually lost: for
example, 12 ... Ngf6 13 Bxf6 Bxf6 14 0-0-0 Nc5 15 Rhe1 Be5 16 g3 Kd7 17 f4 Bg7 18 e5
Rxa8. Black picked up two pieces for the rook and is winning.
b) Relatively best is 9 Ndb5 (threat: Nd5!) 9 ... Rc8 10 Be3 Bxc3+ 11 Nxc3 Qxb2 12
Bd4 Ngf6! 13 0-0 Qa3 14 Nd5 b6 when I don’t believe in White’s full compensation for the
pawn.
9 ... Bxc3+!
Some sins are to be heartily embraced, if in their commission they lead to the greater
good. At first this idea feels like a hazy truth, mingled with a large dose of fantasy. Your
writer has a penchant for such ‘unauthorized’ lines, which fall just outside respectable
theory’s sphere. To my thinking, handing over my precious dark-squared bishop is no
strategic contradiction.

Note: No, this is not a peyote-induced decision. I’m not one of those Modern
players who coddles my dark-squared bishop, as if it were a beloved only child.
In this case I encourage you to commit Modern Defence/Dragon Sicilian blasphemy
by swapping it away for White’s knight, which inflicts horrific damage upon
White’s structure. In essence, we taunt White with the thought: ‘You had better
mate me. Because if you don’t, I will win any endgame due to your awful structure!’.

10 bxc3 Ngf6 11 0-0!?


White falls back to his greatest strength, which is his development lead, so he
logically offers his e-pawn to open lines. 11 Qd3 Rc8 12 0-0 0-0 13 Rfe1 Qc6 is also
favourable for Black, whose king remains a long way from mate, while weakling White’s
queenside pawns are ripe for the plucking.
11 ... Nxe4!
In this chapter I encourage you to view White’s pawns as an all-you-can-eat buffet.
Intuitively, I felt I would survive White’s attempted assault and abscond with a pawn or
two.
12 Bh6
White prevents kingside castling.
12 ... f5!
Intending to artificially castle with ... Kf7.
13 Be3
After 13 c4 Rc8 14 Nd2 Qc5 15 Nxe4 fxe4 16 Re1 Qxc4 I don’t see enough
compensation for White.
13 ... Qc6 14 Nd4 Qc4!

Principle: operate on the colour of your strength, which in this case is the light
squares. The c3 target isn’t running away. Black’s queen exerts power over the light
squares and also keeps watch over potential e6 knight invasion attempts from White.
Instead, 14 ... Qxc3? takes greed one step too far. White now has access to the trick 15
Nxf5! when Black is unable to play 15 ... gxf5? due to 16 Bd4! skewering black queen and
rook.
15 Ne2 0-0
So I got to castle after all.
16 f3 Nxc3!?
Another pawn falls and White’s game continues to pauperize. I could also proceed
with a ‘c3-isn’t-running-away’ philosophy, with the safer 16 ... Nef6.
17 Nxc3 Qxc3 18 Qd5+ Rf7 19 Bd4
19 Rfe1 is met with 19 ... Qc6 as well.
19 ... Qc6 20 Qe6 Nf8
Principle: a queen is the worst possible blockader, since an attack by any opposing
lesser piece sends her fleeing.
21 Qb3 e5
Black’s centre begins to roll, and White’s position squeaks and wheezes like an
untuned violin played by a tone deaf seven-year-old.
22 Bf2 b6
Now b7 is no longer a target.
23 Rad1
23 a4, intending to disrupt with a5, is too slow after 23 ... Rc8 24 Rfc1 Qc4.
23 ... Re8 24 Rd5
This secondary attempt to blockade d5 meets with only transient success.
24 ... Re6 25 Rfd1 f4!

Principle: fix your pawns on the same colour as your opponent’s remaining bishop.
26 Qd3 Rc7
Reminding White that he has pawn weaknesses as well.
27 Rd2 Qc3
Principle: induce swaps when ahead materially.
28 Qxc3
The most miserable moment of a chess game we lose is the exact instant when we
lost hope. I have a feeling my opponent realized he wasn’t going to save a two-pawns-
down ending, with no discernible counterplay. 28 Qe2 Qa1+ is also hopeless for White:
29 Be1 Qxa2 and now White is unable to capture d6, due to 30 Rxd6? Rxd6 31 Rxd6 Rxc2
with a completely winning position for Black.
28 ... Rxc3
The ending is an easy win for Black, with two extra pawns and the superior structure.
29 Kf1
After 29 Rxd6 Rxd6 30 Rxd6 Rxc2 White can resign, since Black threatens both the a2-
pawn and ... Rc1+.
29 ... Rc6 30 Ke2 Nd7 31 Kd1 g5 32 g3 Nf6
The blockade of d5 is easily broken.
33 Rb5 Rc3 34 Rb3

34 ... Rxb3!?
I know, I know. You ask: ‘Why straighten out White’s pawns?’ I’ve noticed a strange
paradox: when the mind orders the body to do something, it obeys; when the mind
orders the mind to let go of a harmful habit, it meets with fierce resistance. Here it is
again: my overwillingness to simplify. Shockingly, it is also Komodo’s top choice, which
happily validates my cowardly decision.
35 cxb3 Kf7
Principle: put your king to work in an ending, since he doesn’t require as much
protection.
36 Rc2 Nd5
Cutting off c7 entry.
37 Rc8 Re8
Oh, no you don’t.
38 Rc4 Ke6 39 Kd2 Rf8
Threat: ... fxg3 and ... Rxf3.
40 g4 h5! 41 h3 Rh8 42 Ke2 hxg4 43 hxg4 Kd7
Depriving his rook of the c8-square, which allows my rook access into his position.
44 Rc1 Nf6
The final step is to get my knight out of the way in order to push my d- and e-pawns.
45 a4 d5 46 a5 d4
Now White’s activity level reaches a state close to catatonic immobility.
47 axb6 axb6 0-1

Summary
Be aware that in our version of the Modern versus the Jump to Nowhere line, we can
easily transpose to Sicilian Dragon lines considered non-threatening to Black’s side. It
would be in your best interest to research these lines and be thoroughly familiar with
them.

Game 9
Y.Salaun-J.Seret
French Championship, Epinal 1989

1 e4 g6 2 d4 Bg7 3 Nc3 d6 4 Bg5 c5 5 dxc5

White hopes to trade away the d4 problem.


5 ... Qa5
I would be inclined to consider the immediate 5 ... Bxc3+! 6 bxc3 Qa5 7 Qd4, as in
J.Hodgson-I.Lipnowski, Winnipeg 1994. Here Black can play 7 ... Nf6! and now if 8 Bxf6 (8
Qb4 Qxb4 9 cxb4 dxc5 also looks fine for Black) 8 ... dxc5 9 Qe3 exf6 I prefer Black’s
structure.
6 Qd2
From my experience, most of our opponents want us to play ... Bxc3, so very few of
your opponents will go passive with 6 Bd2 Qxc5 when Black reached a comfortable
Dragon equality.
6 ... Bxc3!
As we all understand, bishop-takes-c3-aphobia runs rampant in the chess world, but
not for us Modern folk, who do as we please. Speaking of please, please keep in mind
that many normal chess laws are rendered inoperable in Modernworld. I once again
recommend the unbalancing barter of our dark squares, for the opponent’s structure. Be
warned though this time our c3 capture comes with conflicting opinions on the aftermath.
Instead, 6 ... Qxc5 7 Nd5! leads to an edge for White, since it’s difficult to remove that
annoying d5-knight. Black must avoid the suicidal 7 ... Bxb2?? 8 Rb1 Bg7 9 Be3 and the
trouble is Black finds it impossible to defend c7, since 9 ... Qc6?? is met by a killing
bishop’s pin on b5, while after 9 ... Qa3 10 Rb3 Qxa2 11 Nc7+ Black is busted.
7 bxc3

Note: Most of your opponents are likely to recapture with the pawn, as it’s logical
for White to retain queens on the board, since his dark-squared bishop represents
attacking power.

The paradoxical 7 Qxc3! may yield White a tiny endgame edge after 7 ... Qxc3+ 8
bxc3 dxc5 9 a4!, A.Motylev-E.Inarkiev, Tomsk 2004. The idea is to meet a future ... b6
with a5. Here Black should continue 9 ... Nc6 10 Bb5 Bd7 11 Nf3 Nf6 12 Nd2!?. Komodo
prefers White, while I wouldn’t be stressed out taking on Black’s side.
7 ... Qxc5 8 Bd3
8 Qd4 can be met with the cool response 8 ... Nf6! 9 Bxf6 exf6 10 Qxf6 0-0! with both
a development lead and structural superiority for the pawn.
8 ... Nd7
In this way he can play ... Ngf6 without fear of damage to his structure from Bxf6.
9 Ne2 Ngf6 10 0-0 0-0 11 Kh1
White prepares to attack with f4 next.
11 ... b6
Black completes development by fianchettoing on the queenside.
12 Rae1

Principle: mass your pieces first, before attempting to launch an attack against the
opponent’s king.
12 ... Bb7
Tickling White’s e4-pawn, which now requires protection.
13 Ng3!?
Well, don’t economists constantly tell us that consumer spending is a sure way to
jump-start a sluggish economy? He wants his knight to participate in the attack and by
doing so, he abandons defence of his forward c-pawn. I prefer White’s aggressive choice
to the passive alternative 13 f3. This is the kind of move where you resign yourself to life
in a minimum security prison, thinking: ‘Well, at least it’s better than getting locked up
with the murderers in the maximum security prison.’ After 13 ... Ne5 I like Black’s
chances.
13 ... Rac8 14 c4 Ba6
Bye, bye c4-pawn. White must deliver mate, or he will be ground down structurally
and materially.
15 f4
Here they come.
15 ... Bxc4 16 Bxc4!
The meaning of White’s last move is painfully explicit: ‘I’m playing for mate and damn
the cost!’. This all-or-nothing move is necessary to induce Black’s queen away from the
e5-square, so that White can push next move. Komodo assesses the chances at dead
even, which is a bit weird, since it looks to me like someone is likely to win or lose here.
16 ... Qxc4 17 e5
This thematic disruption should offer White just enough compensation for his material
and structural deficits.
17 ... dxe5 18 fxe5 Nd5

Exercise (critical decision): White maintains equality with only one continuation.
Is 19 Bh6 White’s best move?

19 Bh6?
Answer: It isn’t. Black can defend and consolidate after the move played. White
should play the tricky 19 Bxe7! Nxe7 20 Qxd7 Qe6 21 Qd2 Rc4 22 Ne4 Nf5 when attack
and defence feel like they balance each other out.
19 ... Rfd8 20 Re4
The single most important item on the devil’s to-do list is to swing pieces near Black’s
king and then pray to God for a combination or, failing that, a swindle. 20 Qf2 is met with
20 ... Nc3! which not only covers f7, but also prevents White’s possibilities of Ne4 and
Re4.
20 ... Qxa2
Black’s king is okay, as long as f7 is adequately covered. Stronger though is 20 ... Qc6!
which retains all strategic plusses, while preventing White’s next move knight hop.
21 Nf5!
The knight acts as a catalyst for his previously stalled attack.
21 ... Nc5?!
Black begins to go astray under White’s intense pressure. 21 ... Re8! is correct, but 21
... gxf5?? walks into the simple 22 Qg5+ Kh8 23 Qg7 mate.
22 Rd4?!
White is in danger of becoming a victim of his own attempted con game. White
obtains decent practical chances after 22 Rg4! Rd7 23 Qf2 f6 (23 ... Nc7?? loses instantly
to 24 Nxe7+! Rxe7 25 Qf6) 24 Qg3 (threatening a rook sacrifice on g6) 24 ... Kf7 25 c4!
gxf5 26 Qh3! Ke6! with head-spinning complications.
22 ... Ne6
Now all is well for Black.
23 Qf2
After 23 Rxd5 gxf5! 24 Rxd8+ Rxd8 25 Qf2 Nf8! Black will survive and consolidate his
extra material.
23 ... Nxd4 24 Nd6
24 Nxe7+ fails miserably to 24 ... Nxe7 when f7 is sneakily protected by Black’s
queen, who hides on a2.
24 ... f6
24 ... f5! is pretty much the same, but slightly more accurate, since it saves Black a
tempo.
25 Nxc8 Rxc8 26 Qxd4 Qc4 27 Qa1 f5
Black closes avenues of attack and consolidates.
28 e6
For now White decides to keep his future suicidal plans to himself.
28 ... Nf6
This sealant makes Black’s fortress airtight. White’s attack is at an end and he is just
down a truckload of pawns.

Tip: Remain awake when we are winning!

How often have club players, due to a mixture of fatigue and overconfidence, played a
move as boneheaded as 28 ... Qxc2?? which allows 29 Qg7 mate. Black’s king sits in the
corner shivering, which is odd, since the weather report said 84 degrees and sunny. It’s
times like these where we vow to give up chess and take up checkers.
Who among us hasn’t thrown away countless won games in our collective chess lives?
Whenever we do so we become the gullible person who eagerly wires money after
getting the email from the kindly Nigerian prince, who promises to give us 10 million
dollars in the future, if only we first wire him $3,000 (plus our social security number) for
his immediate/urgent needs.
29 g4?
Hey, I just said, “This sealant makes Black’s fortress airtight”! I suppose quick, pain-
free euthanasia is sometimes the preferable option to a painfully drawn-out death
process for the terminally ill. 29 Rd1 a5 wouldn’t have saved White.
29 ... Qe4+ 0-1
Black forces mate after 30 Kg1 Qxg4+ 31 Kh1 Rxc2.

Summary
Meet White’s 5 dxc5 with 5 ... Qa5 (or even the immediate 5 ... Bxc3+!) and 6 ... Bxc3!,
where we once again give up our dark-squared bishop in exchange for structural damage.
Chapter Four
The Bc4 Cro-Magnon Lines
1 e4 g6 2 d4 Bg7 3 Nf3 d6 4 Bc4

When we were children who just started to play chess, didn’t we all delight in our
favourite trap? After 1 e4 e5 2 Bc4! Bc5 3 Qh5!! we all prayed for 3 ... Nf6??. Then the
spectators would shower the board with gold coins when we brilliantly banged out 4 Qxf7
mate! Well, in this chapter the Cro-Magnon guys hope to do the same against our
Modern, as they play an early Bc4, greedily eyeing our f7-square. My suggestion is to
dampen their enthusiasm by switching to the Hippopotamus formation, where our early
... e6 blunts White’s c4-bishop, who now stares at a wall.

Game 10
L.Sussman-C.Lakdawala
San Diego (rapid) 2012

1 d4

Note: Don’t assume you will enter queen’s pawn lines of the Modern if White opens
with 1 d4 or 1 Nf3, since either one can easily transpose back to the e4 versions, as
with 1 d4 g6 2 e4 Bg7 3 d4 d6 4 Nc3 or 4 c3 we return to the king’s pawn versions.
1 ... g6 2 e4 Bg7 3 Nc3
3 Nf3 d6 4 c3 leads to Coward’s variation, which we cover in Chapter 6.
3 ... d6
Sometimes I do play the ... c6 version, which as I mentioned earlier, we cover far less
in this book due to space reasons. I show this game since it’s an offshoot of the Cro-
Magnon lines: 3 ... c6 4 Bc4 d6 5 Qf3 (this is the Monkey’s Bum variation – don’t blame
me, I didn’t make up the ridiculous name! – where White introduces a not-so-subtle
threat to our f7-square) 5 ... e6 6 Nge2 Nd7 7 0-0 Ngf6 8 Qg3 Qe7 9 f4 d5!.

Tip: Rigid structures help the side behind in development, in this case Black.

Following 10 exd5 cxd5 11 Bd3 0-0 12 a4 a6 13 Kh1 Re8 14 b3 Nh5! 15 Qf3 f5! (if
given time, Black will play ... Nhf6 and ... Ne4 with a monster knight sitting pretty on e4)
16 g4!? (this move disrupts Black’s plan at the high cost of weakening White’s king) 16 ...
fxg4 17 Qxg4 Qf7 18 Qf3 Nb8! (re-routing to c6, where it intensifies pressure on d4, while
freeing my c8-bishop) 19 Nd1 Nc6 20 c3 Bd7 21 Nf2? (21 Ne3 was necessary to prevent
Black’s next move) 21 ... e5! White’s king became fatally exposed in C.Carreto-
C.Lakdawala, San Diego (rapid) 2006.
4 Bc4
We returned to that atavistic place of our ancestors, where emotions ran high and
brows tended to run low. White takes aim at our f7-square. I suggest we immediately
Hippopotamize with ... e6 to blunt White’s attacking joy.
4 ... e6
Alternatively, 4 ... Nf6 transposes to the Pirc and 4 ... c6 5 Qf3 to the Monkey’s Bum,
shown in the above note of my game against Carreto.
5 Nf3 Ne7 6 0-0 a6
Here comes the Hippo.
7 a4 b6 8 Be3 Bb7 9 Qd2 h6 10 Rfe1 Nd7
Note: In this chapter the parties operate in parallel universes of thought.
White achieved Fred Reinfeld’s dream. He owns the centre and dominates in the
space department. Even Komodo, a huge Reinfeld fan, assesses heavily in White’s
favour. In reality, theory considers this position as perfectly playable for Black.

Why? Because White lacks access to any viable pawn breaks. For instance, if White
plays a5, we immediately bypass with ... b5!. If White plays e5, we bypass with ... d5!. If
White plays d5, then comes ... e5!. So Black’s trump in the position is that our side is the
one who decides when and where to break. This compensates for White’s space and
Reinfeldianly handsome pieces.
11 d5
The following game is a good example of how to play it if White refuses to play either
d5 or e5: 11 h3 g5! 12 Nh2 Nf6 13 f3 Ng6 14 Rad1 0-0 15 Ng4 Nh5 16 Bf2 d5!? 17 exd5
exd5 18 Bd3 Nhf4 19 Bxg6 fxg6 20 Ne5 Qd6 21 Nd3 Rae8 22 Ne2 Nxd3 23 Qxd3 Re6 24
Bg3 Qd7 25 Nc3 Rfe8 26 Rxe6 Rxe6 27 Re1 c5 28 dxc5?! (don’t unnecessarily open the
game for your opponent’s bishop-pair; correct was 28 Rxe6 Qxe6 – Black is unable to play
the zwischenzug of 28 ... Bxd4+, since White would then slide his rook back to e3 – 29
Ne2) 28 ... bxc5 29 Rxe6 Qxe6 and Black eventually won, G.Gjokaj-I.Glek, Bad Zwesten
1999.
11 ... e5
We follow the principle: the side behind in development should strive to keep the
game closed.
12 Qe2 0-0 13 Rad1
The rook doesn’t do anything on the clogged d-file. He is better off preparing for
Black’s coming ... f5 break with 13 Nd2 f5 14 f3.
13 ... f5
Black’s kingside attack is on the move, while White’s queenside hopes remain clogged
for now.
14 exf5!
Principle: open the centre when assaulted on the wing.
14 ... gxf5

Tip: In such structures I find that it’s usually better to recapture with my g-pawn,
rather than the e7-knight.

The reason is three-fold:


1. By recapturing with the g-pawn, Black increases central space.
2. By recapturing with the g-pawn, Black seizes control over the crucial e4-square.
3. By recapturing with the g-pawn, Black opens the g-file for a potential attack on
White’s king with a future ... Kh8 and ... Rg8.
15 Bc1
He gets out of the way of a future ... f4 push.
15 ... Ng6 16 g3?!
A move made with the thought: delaying an unpleasant but necessary action can be
as fatal as taking incorrect action. The trouble is his move is not yet necessary. A trail of
weak light squares around his king spreads in the wake of this unforced move, which
agrees to carry a heavy defensive burden. He should keep cool with 16 Nd2 when
Komodo calls it even, while I prefer Black.
16 ... Nf6 17 Nd2 Bc8
Re-routing the bishop to take aim at White’s king.
18 Nf1 Kh8
This wise precaution gets away from the glare of White’s c4-bishop and also prepares
to slide my rook to the g-file with ... Rg8 soon.
19 f4!?
He decides he must take a stand to halt Black’s kingside build-up, but it also offers
Black targets of opportunity to open the kingside. The meek 19 f3 may be a better try.
19 ... Rg8
Worth considering is 19 ... exf4 20 gxf4 to further expose White’s king.
20 Kh1 Bb7

Hi, I’m back! The white king’s position on h1 prompts the bishop back to the diagonal.
21 fxe5!?
This opens the centre further, but also mobilizes Black’s central/kingside pawns.
21 ... dxe5 22 Kg1?!
Natural, but inaccurate. White doesn’t like that b7-bishop staring at his king, so he
gets off its long diagonal, but the trouble is he moves his king to the g-file which is
vulnerable due to Black’s g8-rook. Komodo found 22 Ne3! Qd7 23 Rf1 f4 24 Qd3! Bf8 25
Ng2 when White’s power on the central light squares offsets Black’s kingside attacking
chances.
22 ... Qd7 23 Ne3 f4
It is when my attackers congeal around my opponent’s king like hiving wasps that my
evil wannabe-Tal doppelganger arises and insinuates himself, when I often wish he would
go away and leave me in peace. The good news is that my last move is strong and I have
a powerful attack, despite the incompetent attacker conducting the black pieces.
24 Ng2!?
Now he risks getting mated. 24 Bd3 allows 24 ... e4! 25 Nxe4 fxe3 26 Nxf6 Bxf6 27
Qxe3 Qg7 28 Qxh6+ Qxh6 29 Bxh6 Bxd5 30 Bxg6 Rxg6 31 Rxd5 Rxh6. Two pawns are not
enough for the piece in the ending.
24 ... Qh3!
Threat: ... Ng4.
25 Rd3!
A clever lateral defence, designed to menace Black’s queen.
25 ... Ng4 26 Nh4
Forced.
26 ... Nxh4 27 gxh4!?
No better was 27 gxf4 Nf3+ 28 Rxf3 Qh4 29 h3 exf4! with a winning attack for Black,
since 30 Rxf4 is met with 30 ... Bd4+! (clearance) 31 Rxd4 Qg3+ 32 Qg2 (or 32 Kh1
Nf2+) 32 ... Qxe1+ 33 Bf1 Ne3 winning White’s queen.
27 ... Qxh4 28 Ne4 Bf8
Threatening nasty discoveries with my knight.
29 Kh1
White’s king remains alive, although the word ‘alive’ feels like an exaggeration. At this
point I expected the onlookers to burst into joyous applause and song, delighted by your
writer’s sparkling attacking play. Unfortunately the applause never came, due to my next
move.
Exercise (critical decision): Black can play the combination 29 ... Nxh2,
overloading White’s queen. Does it work?

29 ... Nxh2?!
This inaccuracy allows White to force queens off the board. As mentioned above, your
writer struggles with a few unresolved competency issues when attacking, to the point
where I’m a kind of eunuch and that beautiful attack can be sitting in my lap, whispering
alluring promises into my ear, and I still don’t have a clue what to do with her.

Tip: Not every combination should be played, even when we see it, since sometimes
a quiet continuation is far stronger, as in this case.

Answer : 29 ... Bc8!! has the disarmingly passive/strong idea of ... Bf5 and ... Bxe4+,
overloading White’s queen. After 30 Rg1 (or 30 Rf1 Bf5 31 Nc3 Bxd3 32 Bxd3 Bc5! 33 Nd1
f3! 34 Rxf3 Nxh2!, and if 35 Qxh2 Rg1 mate) 30 ... Bf5 31 Rd2 f3! 32 Qxf3 Bxe4 (this
exchange removes White’s key defensive crutch) 33 Qxe4 Nf2+ wins White’s queen.
30 Qf2!
Woes sometimes render us devious, since deception is all we have when we are
busted. I completely missed this idea, which annoyingly removes the queens from the
board.
30 ... Qxf2
With this forced swap, my position is denied its main attacking breadwinner. I put
aside my ache of grief at having to swap queens, and resolved to convert the ending,
where I remain two pawns up, so I’m still winning. 30 ... Qh5?? allows him to simply chop
the knight with 31 Qxh2.
31 Nxf2
White’s relief in removing queens from the board when his king was previously so
exposed, is akin to the actor’s face in the ‘After’ scene in a laxative commercial.
31 ... Ng4 32 Nxg4 Rxg4 33 Rxe5
Now I’m only one pawn up. Fortunately the continued insecurity of White’s king, even
with queens off the board, makes it a relatively easy task to win.
33 ... Bd6 34 Rh5 Re8
Threatening a nasty check on e1.
35 Rd1
My h6-pawn is untouchable: 35 Rxh6+?? Kg7 36 Rhh3 Re1+ 37 Kh2 f3+ wins a rook
and soon forces mate.
35 ... Kg7 36 Bd2 Kg6 37 Rh3 Kg5!
Endgame Principle: the king is a functional piece in the endgame, so don’t hide him.
Instead, bring him out into the heart of the battle.
38 Ba2 a5 39 c4 Bc8!
Intending ... Bf5 and ... Be4+.
40 Rh2 Bf5 41 Bb1
Forced.
41 ... Re4
This move represents clear evidence that your writer isn’t as senile as many of my
readers mistakenly believe. Also good enough is the simple 41 ... Bxb1 42 Rxb1 Re4 43
b3 Bc5.
42 b3
Of course the rook is immune, since 42 Bxe4?? Bxe4+ isn’t likely to end well for White,
who loses an entire rook back.
42 ... Rd4!
The pin of White’s d2-bishop is decisive.
43 Bxf5 Kxf5 44 Rxh6
White evened the material, but this will be a temporary state. Instead after 44 Rf1
Rd3 45 Be1 Kg5 the b3-pawn falls.
44 ... Bb4 0-1
If 45 Rh2 f3 46 Rh5+ Kg6 47 Rh2 and the simplest is 47 ... Rg2 winning heavy
material.

Summary
Me e t any version or offshoot of the Bc4 Cro-Magnon line with the start of a Hippo
formation with ... e6! to blunt White’s light-squared bishop.

Game 11
Z.Peng-V.Malakhov
European Club Cup, Bilbao 2014

1 d4 g6 2 e4 Bg7 3 Bc4
Once again it’s difficult to describe this move as one infused with a high degree of
sophistication, but let’s not go the other direction and underestimate it.
3 ... e6
The blunting process begins immediately.
4 Nc3 d6
In this game Black opts to remain within the Hippo realms. He can also go for an
equal Exchange French-like structure with 4 ... Ne7 5 Bg5 h6 6 Bh4 d5!? 7 exd5 exd5 8
Bb3 0-0! 9 Nf3 (9 Nxd5?? hangs material to 9 ... g5) 9 ... c6. Black stands no worse, since
White’s light-squared bishop is for now out of play and hits a wall on d5, H.Kargoll-
G.Werner, German League 1988.
5 Nge2
The idea behind White’s choice of e2, rather than f3, is that he or she opens options of
a future Austrian Attack hybrid with f4, or can play f3, reinforcing the e-pawn.
5 ... Ne7 6 0-0 a6 7 a4 b6 8 Bg5

Tip: When I play the black side of a Hippopotamus formation, I am always


delighted when White ‘‘ins" with Bg5. The reason for my joy? It’s because Black
almost always needs ... h6, and when White plays Bg5, now our ... h6 comes with
tempo, and possibly we gain a bonus second tempo when White plays Bh4, since
we normally also want ... g5.

8 Be3 is in my opinion a more accurate move.


8 ... h6
Thanks for the free tempo!
9 Bh4 g5!
And thanks for the second one as well!
10 Bg3 d5!
The interval of mutual silence ends with some flexible thinking on Black’s part. He
plays ... d5 down a tempo from earlier, since he wasted a move with ... d6 first, but
circumstances altered and I like Black’s super-aggressive version of a hybrid Exchange
French structure. The routine 10 ... Nd7?! is unwise due to 11 f4! when White makes good
use of his e2 posting by pushing his f-pawn, with dangerous attacking chances.
11 exd5
After 11 Bd3 c5! (threat: ... c4, trapping White’s bishop) 12 dxc5 bxc5 13 b3 Nbc6 I
like Black’s active position and superior control over the centre.
11 ... exd5 12 Bb3
12 Bd3 c5! 13 dxc5 bxc5 14 b3 Nbc6 leads to the position mentioned in the above
note.
12 ... Be6
12 ... Nbc6 preventing a5, may have been slightly more accurate.
13 a5! b5

14 Qd2
14 f4 can be met with 14 ... Nbc6 15 Qd2 Nf5 16 Bf2 g4! with a light-squared blockade
on f5.
14 ... Nbc6 15 Rfe1 Qd7 16 Na2
Setting up a future hop to the b4 hole. 16 f3 is also a thought, the idea of which is to
hang on to his dark-squared bishop if Black plays ... Nf5.
16 ... Nf5
Going after the bishop-pair.
17 Nb4
17 c3? intending Nc1 and Nd3 may be an admirable plan, except it hangs his a5-pawn
after 17 ... Nxa5. Details matter!
17 ... Nxb4 18 Qxb4 h5!?
Very ambitious, and possibly inaccurate, since it allows White a trick on his next
move. Correct was 18 ... 0-0-0! and only then ... h5.
19 f3?!
This passive reaction should give Black a strong initiative. White missed 19 Be5! Bxe5
(White undoubtedly feared 19 ... f6?, but forgot that she had the overload combination 20
Bxc7! winning a pawn; sometimes we are actually pushed into happy confrontation by
chance lucky geometry, rather than by inclination) 20 dxe5 0-0-0 21 Qc5 when she stands
better.
19 ... 0-0-0?!
After 19 ... Nxg3! 20 Nxg3 0-0-0 Black’s attack is clearly more promising then White’s
on the other side.
20 Qd2
20 Bf2! should be played.
20 ... Rdg8
Both Black and White tarry with trifles, the consequences of which distract them both
from the position’s essence. Both parties perpetuate a false notion, by underestimating
20 ... Nxg3!.
21 c3

Exercise (critical decision): Black’s pieces continue to congregate around White’s


king, each one a possible assassin. His position continues to make appreciable
progress
and his options float on a reservoir of favourable conditions. Black’s three choices:
a) 21 ... Nxg3, bagging the bishop-pair.
b) 21 ... .g4 attempting to open lines.
c) 21 ... h4, intending ... g4 next.
All three favour Black, yet only one version gives him a winning attack. Which one?

21 ... g4
Black’s game remains quite promising after this, but this move does slightly impair the
efficiency of Black’s attack.
Answer: Much stronger was ‘c’: 21 ... h4! 22 Bf2 g4! 23 fxg4 Bh6 24 Qd3 Rxg4 25 Qf3
Rhg8 26 Kh1 Rxg2 and if 27 Qxg2?? Rxg2 28 Kxg2 Ne3+! (opening the diagonal for
Black’s queen and bishop is fatal for White’s king) 29 Bxe3 Bh3+ 30 Kf2 Qf5+ and White
can resign.
22 fxg4 Nxg3 23 Nxg3 Bxg4
Even more promising is to open the h-file with 23 ... hxg4! 24 Bc2 Bf8 25 Qf2 Bd6.
24 Qf2 h4 25 Nf1 Bh6 26 Kh1
After 26 Ne3 Bh3 27 Bxd5! Bxg2! 28 Bxg2 h3 White remains in trouble.
26 ... Be6
Stronger is 26 ... h3! 27 g3 Bf5 28 Bc2 Bxc2 29 Qxc2 Re8.
27 Bd1
White decides it is a good time to increase the number of sentries around his king,
with his bishop heading for f3. His last move clearly betrays unease.
27 ... Qd6
Once again Black should toss in 27 ... h3!.
28 Re5 f6 29 Rh5 h3 30 g3 f5 31 Bf3
White should risk 31 Rxh3!? f4 32 Rh5.
31 ... f4
Black’s attack is once again open for business, so he aggressively pursues open lines.
32 Re1 c6 33 Qe2 Bd7 34 Qe7?!
White won’t get mated, but she isn’t off the hook, since Black retains a winning
position in the ending. 34 g4 may offer superior survival chances.
34 ... Qxe7 35 Rxe7 fxg3 36 hxg3 Bc1!
He goes after all those queenside pawns who are conveniently lined up on dark
squares.
37 b4?!
Her last prayer lay in attempting to set up a fortress with 37 Re2 Rxh5 38 Bxh5 Rh8!
39 Bg6 (not 39 Bf3?? Rf8 40 Nh2 Re8 41 b3 Bb2! and White’s pawns begin to fall) 39 ...
Rf8 40 Kg1 Bg4 41 Rc2 Bg5 42 Bd3 when White’s game is miserable, but at least here she
can play on.
37 ... Rxh5 38 Bxh5 Rf8 39 Kg1
Exercise (combination alert): Malakhov found a cute way to pick off White’s
queenside pawns. How?

Answer: Deflect White’s knight from coverage of d2.


39 ... h2+!
“Forgive this betrayal, Noble Born. Please keep in mind that the lowly too have needs
and desires,” the h-pawn tells White’s king and knight, in his most apologetic tone.
40 Nxh2 Bd2
Principle: the base pawn is always the weakest of the herd. Now White’s pawn chain
begins to crack.
41 Nf3 Bxc3 42 Ne5
This hastens her defeat. 42 Rh7 Bf5 43 Re7 Be4 44 Kf2 Bxb4 wouldn’t have saved
White.
42 ... Bxd4+ 43 Kg2 Rf2+ 44 Kh1 Bxe5 45 Rxe5 Rb2
The weakness of White’s pawns continue to rise up like a serial killer’s murdered
corpses, arising to the lake’s surface.
46 Rg5 Rxb4 47 Kg2 Re4 48 Kf3
48 Bg6 Ra4 wouldn’t have saved White.
48 ... b4 49 Bg6
Exercise (combination alert): When we are busted, we remain in a high state of
preparedness, yet are always unready, since we lack the natural resources necessary
to defend our decaying position. White just threatened Black’s rook with his bishop.
How did Malakhov react to it?

Answer: Pawn promotion.


49 ... b3! 0-1
Black’s rook is his John the Baptist, who prepares the way for his b-pawn’s entry to the
queening square. Black ignores the threat and White’s rook is oddly misplaced to stop the
surging b-pawn from queening.

Summary
Keep in mind that the Hippo formation is open to various stylistic interpretations and may
sometimes morph into a hybrid Exchange French formation.
Chapter Five
Fianchetto Lines
1 e4 g6 2 d4 Bg7 3 Nc3 d6 4 g3

In this chapter White refuses to try and blow our Modern out of the water. Instead,
they fianchetto with g3 and attempt a slow build-up and squeeze, with perhaps a future
f4 to follow. I offer two completely different ways to meet the fianchetto line:
1. The standard way, which is to play ... c6, ... Nbd7 and ... e5.
2. We provoke, allowing White to chase our knight after an early ... Nc6.
Either path is fully playable for our side.

Game 12
S.Yudin-P.Smirnov
Moscow 2008

1 e4 g6 2 d4 Bg7 3 Nc3 d6 4 g3
This slow build-up from White can be deceptively dangerous for our side, if we drift
without finding a plan.
4 ... Nf6
Note: Just because this is a book on Modern Defence doesn’t mean that we are
obliged to play Modern lines exclusively. Black’s last move transposes to our
close cousin, the Pirc.

Next game we look at the pure Modern Defence option, 4 ... Nc6!?.
5 Bg2 0-0 6 Nge2
White usually develops his g1-knight to e2 to keep future f4 ideas open.
6 ... e5
Black takes back some central space.
7 h3

Note: The idea behind this common move is to be able to post a bishop on e3,
without fear of ... Ng4.

7 ... c6
We deny White’s pieces use of d5, while preparing to gain space with ... b5.
8 a4
White logically blocks Black’s queenside expansion attempts. Instead:
a) 8 Be3 b5 9 a3 Bb7 10 0-0 Nbd7 and Black achieved full equality.
b) 8 dxe5 dxe5 9 Qxd8 Rxd8 when the ending is even.
8 ... a5
Otherwise White can play a5 later on, gaining more queenside space. The pure piece
play option looks playable for Black as well: 8 ... exd4 9 Nxd4 Na6 (the knight heads for
b4) 10 0-0 Nb4 11 a5 (otherwise Black will play ... a5 himself) 11 ... Re8 12 Nde2 Re5!?
13 Ra4 c5 14 Bf4 Re6 15 Ra1 Nc6 16 a6 Rb8 17 axb7 Bxb7 and Black’s activity makes up
for White’s pressure on d6, P.Motwani-V.Ivanchuk, Novi Sad Olympiad 1990.
9 Be3 Na6

Tip: Just as in some Classical lines where White played a4, we can set up with ... a5,
... Na6 and ... Nb4, when our annoying knight is rather difficult to remove from its
post.

10 0-0
Instead, 10 Qd2 Nb4 11 0-0-0!? (obviously risky, since Black can later try to enforce a
... b5 break) 11 ... Qc7 12 g4 Be6 13 f4 Bc4 14 f5 was tried in L.Espig-R.Polzin, German
League 2002. Here Black should counter in the centre with 14 ... d5! with great
complications.
10 ... Nb4 11 f4
White thematically presses down the f-file.
11 ... Nh5! 12 g4
Continuing to expand his empire.
12 ... Nxf4 13 Nxf4 exf4 14 Bxf4 f5?!
This move appears to be more of a probing question than an actual statement.
Natural tacticians are willing to take on structural weaknesses to enhance activity, while
strategists view the opponent’s structural weaknesses as prey. This overly aggressive
thrust should be unfavourable for Black. IM John Watson suggests the calmer and
superior 14 ... Be6 after which Black looks fine.
15 exf5 gxf5 16 g5!
Now Black is saddled with an isolated f-pawn and a hole on f4.
16 ... Be6!

Okay, Black’s isolated f-pawn is blockaded, but his pieces are active.
17 Qd2
17 d5 looks powerful, but isn’t: 17 ... Bf7 18 dxc6 bxc6. Surprisingly, Black doesn’t lose
a pawn, since 19 Bxd6? is met with 19 ... Qb6+ 20 Kh1 Rad8 21 Re1 Nxc2 22 Qxc2 Rxd6.
Black’s raking bishops offer him a clear advantage.
17 ... Qd7 18 Rfe1
18 h4! intending to push to h6 looks quite promising for White.
18 ... Rae8 19 Ne2!?
Ceding control over d5 allows Black to equalize. White retains a structural edge after
19 h4!.
19 ... Bd5!
The swap of light-squared bishops weakens White’s king.
20 Ng3?!
With this move White uproots his own self interests. 20 c3 retains equality after 20 ...
Bxg2 21 Kxg2 Nd5 22 Ng3.
20 ... Be4!
Now Black stands better.
21 Rac1
If 21 Nxe4 fxe4 and White is unable to capture e4, since his h3-pawn hangs.
21 ... Nd5 22 c3 c5?!
In such messy positions there tends to be a distressing discrepancy between our
predictions and the actual outcomes. This is a case of the right idea, but wrong move
order. Black holds the advantage after 22 ... Nxf4! 23 Qxf4 c5!.
23 dxc5?
23 Nxe4 fxe4 24 Bg3 still looks okay for White.
23 ... Nxf4 24 Qxf4

Exercise (planning): Come up with a plan for Black to seize a dangerous initiative.

Answer:
Step 1: Centralize the dark-squared bishop with tempo.
24 ... Be5! 25 Qf2 Bxg3!
Step 2: Exchange it off for White’s knight. When we play, there is no governing agency
which oversees that we don’t violate chess laws and principles. In open games we are
taught that bishops are of greater value than knights, yet here we see a deliberate
violation. As it turns out, it’s a good deal for Black. One of the advantages of owning the
bishop-pair is that it is often easy to give it back, in exchange for the initiative, which
Black receives in this case.
26 Qxg3 f4
Step 3: Push the f-pawn down the board, which creates confusion in White’s camp.
27 Qf2 f3 28 Bf1 d5
It’s pretty clear that this is more than just a transitory initiative. Black for now may be
down a pawn, but his strong bishop and choking f-pawn, coupled with White’s loose king
position, more than makes up for it.
29 Rcd1 Qxa4
Black establishes material equality. 29 ... Re5! is even stronger.
30 c4?!
It’s possible to destroy the thing we love, in a misguided attempt to save it. Wishing
desperately for a result of our choosing tends to cloud impartial observation, and by
extension, the outcome itself. White’s bold freeing try is the beginning of a commendable
dream, but as events unfold, we later discover major issues dealing with the actual
implementation. The risky idea is to break up Black’s grip on the central light squares, at
the too high cost of loosening his position further. 30 Rd2 is a safer and probably superior
alternative.

30 ... dxc4 31 Rd4 Bd3 32 Rxe8


After 32 Bxd3?? Rxe1+ 33 Qxe1 f2+ White drops his queen.
32 ... Qxe8 33 Bxd3 cxd3 34 Rxd3 Qe4!
Despite the simplifications White’s king doesn’t look so safe, and neither do his pawns.
35 Rc3?
35 Rd4 was White’s only hope to resist.
35 ... Rf5! 36 Kh2 Qe5+! 37 Kh1 Qe2! 38 Rc2
Alternatively, 38 Qxe2 fxe2 39 Re3 Rf1+ and Black promotes.
38 ... Qe4?!
Missing 38 ... Qd1+! 39 Kh2 Re5! (threat: ... Re2) 40 Kg3 Rxg5+ 41 Kf4 Rg2! and
wins.
39 Rc3 Kf7 40 Re3?
In such difficult defensive positions we become the over-caffeinated, stressed-out air
traffic controller at a busy airport, forced to guide the landing of three planes
simultaneously, where it’s so easy to slip up and allow a crash.
Exercise (combination alert): Black to play and win.

Answer: Queen sacrifice/pawn promotion.


40 ... Qxe3! 0-1
Such shocks strike our psyche with the force of a physical blow upon tender flesh,
when we contemplate the depth of our opponent’s treachery. 41 Qxe3 (“I only hear the
toothless mutterings of a senile crone,” declares White’s queen, who may be
underestimating her sister’s ability at subterfuge) 41 ... f2 and for White this is that
unfortunate situation where you corner your deadbeat friend who owes you money. He
writes you a check, which then promptly bounces when you try and cash it. And then the
bank charges you the $10 fee. After 42 Qb3+ Kf8 White runs out of checks and Black
promotes.

Summary
A transposition to Pirc with an early ... Nf6 is perfectly playable against White’s fianchetto
system. Our plan is ... e5, ... c6, ... a5, ... Na6 and ... Nb4.

Game 13
A.Raetsky-K.Sakaev
St. Petersburg 1999

1 d4 d6 2 e4 g6 3 Nc3 Bg7 4 g3 Nc6!?


This is the Modern version of a queenside Alekhine’s Defence. Our provocation
actually scores quite well for Black.
5 d5
White takes up the challenge, hoping to punish Black’s provocation. Instead 5 Be3 e5
6 d5 (6 dxe5 Nxe5 is also just fine for Black and after 6 Nge2 exd4 7 Nxd4 Nf6 idea of ...
Ng4 are in the air; following 8 h3 0-0 9 Bg2 Re8 Black’s pressure on e4 compensates
White’s extra space) 6 ... Nce7 7 Bg2 f5! sees Black achieve a nice looking g3 King’s
Indian, since normally he plays ... Nf6, then later moves the knight and only then plays ...
f5 and returns his knight with ... Nf6. In our Modern Defence version Black got ... f5 with
two extra tempi.
5 ... Nd4!?

Note: This move ups the ante even further, since it appears as if our knight will be
in danger. The comps, however, assure us that it remains safe.

If the d4-square makes you nervous, then also possible is 5 ... Ne5 6 f4 Nd7 7 Bg2 c6
8 Nf3 cxd5 9 Nxd5 Nb6 10 0-0 Nxd5 11 Qxd5 Qb6+ 12 Kh1 Nf6 13 Qd3 0-0 14 Be3 Qc7
and Black achieved a Dragon-like equality, V.Spasov-J.Speelman, Biel Interzonal 1993.
This game is annotated in The Modern Defence: Move by Move.
6 Nb1!?
No this isn’t a typo and I’m nearly certain White didn’t suffer a debilitating stroke. The
idea is to trap Black’s wayward knight with c3 next. Instead, with 6 Be3 c5 7 dxc6 (or 7
Qd2 and now I like Komodo’s suggestion of 7 ... f5!) 7 ... Nxc6 8 Qd2 (8 Bg2 is met with,
you guessed it, 8 ... Bxc3+! 9 bxc3 Nf6 when I prefer Black’s game) 8 ... Nf6 Black
achieves a favourable g3 Dragon formation where White looks more passive than a
normal Dragon, since his knight sits on g1 rather than the traditional Dragon-approved
d4-square.
6 ... c6

The only way to extricate our knight.


7 c3
If 7 a4? and the idea to trap Black’s knight with c3 next, fails miserably after 7 ... cxd5
8 exd5 Qa5+ 9 Nc3 Bg4! 10 f3 Nxf3+! 11 Nxf3 Bxc3+ which is very much in Black’s
favour.
7 ... Nb5 8 Bg2 cxd5
Black can also retain the pawn tension with 8 ... Bd7 9 Ne2 Nf6 10 c4 Nc7 11 Nbc3 0-0
12 0-0 which also looks like an acceptable g3 King’s Indian hybrid for Black.
9 exd5 Nc7
“Although this is already the fourth move of the black knight, White has no advantage
in development at all” writes GM Alexander Volzhin. Such is the arcane magic of the
Modern Defence.
10 Ne2 Nf6 11 0-0 0-0 12 c4
It looks like White earned an edge due to his extra space. This assessment is
betrayed by Black’s next move.
12 ... b5!
Tip: When your opponent owns more space, keep chipping away with pawn
challenges, in an attempt to reduce it.

With each new theoretical novelty, we infuse the opening’s bloodline with a new and
hopefully improved gene. This new move breaks up White’s queenside and opens lines for
Black along the b-file. Sakaev’s move improves upon 12 ... Bd7 13 Nbc3 Qc8 14 Be3 Bh3
15 Qd2 Bxg2 16 Kxg2 Na6 17 b3 Nc5 18 Rac1 and White enjoyed a pleasant space
advantage in G.Franzoni-J.Hebert, Biel 1980.
13 cxb5 Nxb5 14 a4
This move weakens his queenside, but if he doesn’t play it, then White has trouble
developing his b1-knight to a good square.
14 ... Nc7 15 Nbc3
15 Nd4? has the idea to invade on c6, but fails to 15 ... Ncxd5 16 Nc6 Qc7 17 Bxd5
Nxd5 18 Qxd5 Bb7 (this move advantageously regains the piece) 19 Nxe7+ Qxe7 and
White is sure to suffer on his weakened light squares.
15 ... Bb7
Black’s pressure on d5 makes up for White’s extra space.
16 Rb1
He wants to gain queenside space.
16 ... Qd7
I would have tossed in 16 ... a5.
17 b4
Exercise (critical decision): Only adventurers feel comfortable entering a mystery.
Assess 17 ... e6. Would you play it, or hold off?

Answer: 17 ... e6!


A chess game can be won by merely observing the minutiae more closely than your
opponent. This freeing alteration, which displays exceptional strategic judgement, opens
up an entirely fresh range of possibilities. Sakaev’s move is actually a pawn sacrifice
which White is unable to accept.
18 dxe6!?
I have a feeling White underestimated Black’s coming piece activity when he played
this move. Safer is 18 Nf4 e5 19 Nfe2 which is about even.
18 ... Nxe6
Forced, since Black must protect the b7-bishop.
19 Bxb7 Qxb7 20 Nf4
The self-preservatory instinct has a way of overcoming pride. White, unable to snap
up the not-so-weak d6-pawn, wisely enters the abort-mission stage. However, his plan is
slightly out of phase with his position’s requirements, since he picks the wrong square to
blockade. In this case Black can seize control over d5, tactically.
Instead, after 20 Qxd6?! (when we seal a bargain with the devil, we can be certain
that he will renege on it) 20 ... Rfd8 21 Qe5 Qf3 22 Qe3 Qf5 White finds ... Qh3 is in the
air and he has trouble finding a good move. For example, 23 f3? is met with 23 ... Nd5 24
Nxd5 Qxb1. However, after 20 Be3! Rac8 21 Qd3 Rfd8 22 Rfd1 d5 23 Nd4 White should be
okay.
20 ... d5!

Principle: passed pawns should be pushed. Black is the one to take control over d5,
despite the fact that White, for now, trains a greater number of pieces on the square.
21 Qf3
Alternatively:
a) 21 Ncxd5? Rad8 22 Nxf6+ Bxf6 23 Qg4 (23 Bd2 Nxf4 24 gxf4 Bc3 is pretty awful for
White) 23 ... Nd4 24 f3 Rfe8 25 b5 Re1! 26 Rxe1 Nxf3+ 27 Kf2 Nxe1 and White is busted,
since he is unable to play 28 Kxe1?? Qe4+ 29 Qe2 Qxb1 with an extra exchange for Black
and a crushing position to boot.
b) 21 Nxe6! is psychologically difficult to play, since it straightens Black’s pawns, but
21 ... fxe6 22 Qe2 only a shade better for Black.
c) 21 b5! (principle: when you own a pawn majority, push it as fast and as far as
safely possible) 21 ... Rac8 22 Nce2 Nxf4 23 Nxf4 Rfd8 24 Bb2 d4! 25 Re1 and White only
stands a shade worse.
21 ... Rad8 22 Rd1?!
22 Be3 should be tried, fighting for control over d4.
22 ... Ne4! 23 Nce2
23 Ncxd5?? Nxf4 24 Nxf4 Rxd1+ 25 Qxd1 Nc3 picks up material.
23 ... Rfe8 24 Be3 Nxf4! 25 Bxf4
This move regrettably loses control over d4, which allows Black’s d-pawn to surge.
White’s problem is that 25 Qxf4? is even worse after 25 ... d4!, as Black can play the
move anyway. Now if 26 Bxd4 Rd5! 27 Bxg7 Ng5!! (the knight’s erratic zigging and
zagging has a kind of alien spaceship feel to it; this deadly zwischenzug, threatening a
fork on h3, is decisive) 28 Kf1 Rxd1+ 29 Rxd1 Qh1+ 30 Ng1 Nh3 and game over.
However, the ugly move 25 gxf4! keeps Black’s d-pawn in check, at the very high cost
of voluntarily defacing his own structure. Believe it or not, this may actually be White’s
best move in the position.
25 ... d4!
It becomes very clear that Black’s isolani is a source of strength, rather than a
weakness.
26 b5 Qd5 27 Rb3

Exercise (combination alert): Black can win a piece at a minimum, if you see
his combination.

27 ... Qc4
Black is still winning but he missed an instant game-ender.
Answer: Deflection/removal of the guard: 27 ... g5! 28 Bc1 g4!. White’s queen must
keep watch over his b3-rook, but there is no way to do so, since if 29 Qd3 Nc5 and Black
picks up the rook in any case.
28 Rbd3 Nc5 29 R3d2 d3 30 Nc1
Such forced concessions turn us into the sullen teenager, resisting/complying with an
unpleasant parental directive. The chess board may be a finite volume of space, yet here
it feels like the black d-pawn’s influence forced White’s scrunched up pieces to a non-
existent negative first rank.
30 ... Bc3
Black’s queen sends her best hatchet man to do her dirty work for her. This wins the
exchange.
31 Rxd3
If 31 Ra2?? d2 32 Bxd2 Bxd2 33 Raxd2 Rxd2, and if 34 Rxd2 Re1+ 35 Kg2 Qf1 mate.
31 ... Nxd3 32 Rxd3 Rxd3 33 Nxd3
33 Qxd3 Re1+ 34 Kg2 Qxd3 35 Nxd3 Ra1 36 Nc5 Bd4 37 Nb3 Rxa4 is an easy win for
Black.
33 ... Qxa4 34 Qd5 Qe4
Principle: swaps favour the side up on material.
35 Qd7 Qe7 36 Qd5 Rd8 37 Qc4 Bf6 38 Nb4 Rd4 39 Qc8+ Kg7 40 Nc6 Rd1+
41 Kg2 Qe4+
White’s king is all alone and finds himself caught within a force too great to contain.
42 Kh3
Have you ever had one of those nightmares where you are being chased, and the
path/escape route ahead of you, which previously stretched out to infinity, suddenly gets
blocked? Still, at least White avoids 42 f3?? (deliberately weakening the pawn front
around our king represents juicy attacker bait; I guess when there is but a single
available option, then we would be wise to pick option one) 42 ... Qe2+ 43 Kh3 Qf1+ 44
Kg4 h5 mate. The most shameful of all losses is to suffer the indignity of getting mated
by your opponent’s lowly pawn.

Exercise: (combination alert): White’s position yells ‘Danger!’ like a WWII


air-raid siren. How did Black finish the game?

Answer: Mating net. The threat of a rook mate on h5 costs White heavy material.
42 ... Rd5! 0-1
Answer no.2: 42 ... Rg1! also works.

Summary
I like our 4 ... Nc6!? provocation, which so far scores very well against White’s fianchetto
system.
Chapter Six
The Coward’s Variation
1 e4 d6 2 d4 Bg7 3 Nf3 d6 4 c3

It’s possible to love and hate our own opening repertoire. I call this one Coward’s
variation, since it’s my own favourite way of meeting the Modern as White, because its
dull byways are admirably suited for my chickenish opening needs. Yet I inwardly curse
my cowardly opponent when he or she plays it as White against my own Modern! If you
are okay with a draw as Black, then this chapter won’t scare you. If you are the ambitious
type and seek a win, then this variation can be a problem. In this chapter White hunkers
down with the most solid option in the entire book, as with:
Unlike other chapters, in this one we as Black lack targets, since White refuses to
create them. Our job is to be patient and later seize upon potential imbalances. But I
warn you be careful with this line. White’s play can be so subtle and overtly non-
threatening, that our opponent can gain strategic footholds without our side even
knowing it, and then melt back into the anonymity of its inherent dullness.

Game 14
K.Yang-C.Lakdawala
San Diego (rapid) 2015

1 e4

Did You Know? As a Caro-Kann player, I reach Coward’s Variation a move down
with Black, after the move order 1 e4 c6 2 d3 d5 3 Nd2 e5 4 Ngf3 Bd6 5 g3,
where we reach the line, but with colours reversed, where it is White who is playing
a kind of Modern Defence a move up.

1 ... g6
The Modern can, of course, also be reached via a 1 ... d6 move order: 2 d4 Nf6 3 Bd3
Nc6 4 c3 e5 5 Ne2 g6. Now the game transposes back to a hybrid of Coward’s variation:
for example, 6 0-0 Bg7 7 Bg5 h6 8 Bxf6!? (risky, because later White may pay for giving
up his custodian of his dark squares) 8 ... Qxf6 9 d5 Nb8 (principle: in closed games
quality trumps quantity, when it comes to development ; Black leisurely re-routes the
knight to d7, where it suppresses White’s future c5 breaks) 10 Nd2 0-0 11 f3 h5 12 Qc2
Nd7 13 Bb5 Bh6, V.Iyer-C.Lakdawala, San Diego (rapid) 2013. My once bad bishop is now
the most dangerous minor piece on the board, since it is unopposed by a white
counterpart.
2 d4 Bg7 3 Nf3
3 c3 intending to create a bind with f4, next can be met with the atonal idea 3 ... d5!?.

White has two ways to play it:


a) 4 exd5 Qxd5 5 Nf3 Nf6 6 Be2 0-0 7 0-0 c5 where Black reaches a decent version of
a kind of c3-Sicilian hybrid.
b) 4 e5 c5 5 Nf3 Nc6 6 Be2 Bg4 7 0-0 cxd4 8 cxd4 Nh6!, intending to hammer away at
White’s base d-pawn with ... Nf5. I don’t believe White stands better here.
3 ... d6 4 h3
Tip: This is a sneaky move order, where White hopes to trick us into a transposition
into a King’s Indian, since he hasn’t yet committed to c4. We may be better off just
immediately forcing a non-threatening Classical version of the Pirc with 4 ... Nf6 5 Nc3.

The idea behind this move is to deprive Black of all variations involving ... Bg4. The
secondary intent may be to later play Be3, without worry of ... Ng4. Instead, 4 c3 is the
most common move order for Coward’s Variation: 4 ... Nd7 (the simplest way to deal with
Coward’s variation is to go Pirc style, with ... e5 and ... Ngf6; we reach equality and then
it’s up to us to add some spice to the game, if we want to go for the full point) 5 Bd3 e5 6
0-0 Ngf6.
4 ... a6!?

Warning: Just as in the note above, be aware that when White hasn’t committed
to Nc3, your opponent still has time to switch to a King’s Indian versus Modern
by tossing in c4.

If I face this line today, I would probably opt for a Pirc with 4 ... Nf6 5 Nc3 and here
Black can either go for a 5 ... c6 or a 5 ... a6 set-up.
5 Bd3
5 c4 would turn the game into a King’s Indian, where Black committed to an early ...
a6. The game may continue 5 ... Nd7 6 Nc3 c6.
The idea is to play for ... a6 and ... b5 to gain queenside space.

Tip: In such structures meet 7 a4, which suppresses our ... b5 idea, with 7 ... a5!
when
White must deal with a hole on b4, which may later be occupied by one of our pieces.

Instead, 7 Be3 b5 8 a3 reaches a sharp King’s Indian-like position.


5 ... Nc6 6 c3
This move turns the game into Coward’s variation, perhaps White’s most annoyingly
solid and (almost) unloseable way of meeting Modern.
6 ... Nf6 7 0-0 0-0 8 Re1 e5 9 Na3
This is White’s main move. The knight reinforces d4, via c2. The alternative is 9 Nbd2
Bd7 10 Nb3 Re8 with even chances.
9 ... Nh5
9 ... Nd7 is the alternative.
10 Bc4!? Qe8
The idea is to be able to play a future ... Kh8 and ... f5, without worrying about my
h5-knight if he plays exf5 gxf5. Also playable is 10 ... Nf4 11 d5 Nb8 12 Bf1! Nd7 13 g3
Nh5 when the game resembles a g3 King’s Indian.
11 Kh2 h6 12 Nc2 Bd7 13 Bd2 Kh7 14 dxe5
He finally releases central tension.
14 ... Nxe5
Principle: swaps help the side which is more cramped.
15 Nxe5 Qxe5+
Tip: If you seek to relieve the boredom and create a more unbalanced structure in
this line, then consider recapturing on e5 with a piece, rather than a pawn. 15 ... dxe5
is equal and dull.

16 g3
He wants to gain space with a future f4.
16 ... Rae8 17 Bf1?!
White should lose the initiative after this passive retreat. Better was 17 Be3!.
17 ... Nf6?!
Human error is perhaps the most consistent principle in the chess universe. The
wonderful thing about being a naturally cautious person is that we are acutely aware of
the painful consequences of risk. On the flip side, though, we often refuse to take action
when we should, and thereby miss opportunity, as in this case. I incorrectly rejected 17 ...
Bc6! 18 Bg2 f5! when Black stands slightly better.
18 Bg2 Bc8 19 Nd4 Nd7 20 f4 Qe7 21 Qc2 Nc5 22 Rad1 Bd7
The position is about even, despite White’s kingside space. There isn’t much for my
side to do but await events, so for now, I put up an unresisting front.
23 Re3 Bc8 24 Rde1 Qd7
The idea was to entertain a queen swap with ... Qa4. The comp doesn’t like this
move, but I didn’t want to just wait forever with 24 ... Qd8.
25 f5!?
The things we do in the name of honour. A crucial decision in every game is: when to
act, and when to wait? My opponent is a rapidly improving young master and the 2017
Southern California high school champion, which is synonymous with raw ambition. So he
refuses to sit tight and wait.
White’s move appears to be some kind of demonstration, but just what he is
protesting is subject to debate. I was happy to see such a committal move, since now I
can play for the full point. I can tell you that I would never play such a move if I had the
white pieces, since a strategic environmentalist like me takes a dim view of pollution, in
the quest for vague, future tactical profits. I would play 25 b4!? Ne6 26 Nf3 and Black has
nothing to do but keep waiting.
25 ... Qd8 26 Rf1 Nd7
Seizing upon the freshly created hole on e5.
27 Ref3 Nf6
I remind him of his e4-pawn’s vulnerability.
28 R3f2 c5!?
I deliberately weaken d5 and also my d6-pawn to gain queenside space.
29 Ne2 g5
Depriving his pieces of the f4-square. Now White’s ‘attack’ (how should I put this
kindly?) isn’t exactly an imposing presence.
30 c4
He clamps down on the d5-square and hopes to later pressure d6 along the open d-
file.
30 ... b5 31 cxb5!?
I was happy to see him swap his c-pawn for my a-pawn. 31 b3 b4 deprives him of the
use of the c3-square.
31 ... axb5 32 Nc3?!
32 b4! kept the game in balance.
32 ... b4 33 Nd5

33 ... Bb7?!
I should jump at the chance to seal the d5-square with a pawn with 33 ... Nxd5! 34
exd5 Bf6. White’s kingside attacking chances are at an end and his structure is
overextended.
34 a3?!
We both overlooked 34 e5!, threatening discoveries on Black’s b7-bishop, and if 34 ...
Nxd5 35 f6+ Kg8 36 fxg7 Kxg7 37 exd6 Qxd6 with even chances.
34 ... bxa3?!
The tendency is to relax and not look deeper into a position, when we falsely believe
we already understand it. 34 ... Ba6! gives Black the advantage.
35 bxa3
White’s downfall comes, not for what he has done, but from what he left undone. His
blocked e-pawn lies at the core of his frustration. Once again we both miss the freeing
idea 35 e5!.
35 ... Bxd5!?
I was playing to hand him a bad bishop.
36 exd5 Kg8
I get my king out of the range of his queen.
37 Bc3 Re3
With a sneaky threat. I can’t tell you what it is, since it would give away the answer to
the coming combination alert.
38 Rf3 Rfe8 39 Rxe3 Rxe3 40 Re1??
Correct was 40 Rf3 Rxf3 41 Bxf3 Nd7 when White nurses weak pawns and is in danger
of getting saddled with a bad light-squared bishop, versus a powerful black knight which
can post on e5.

Exercise (combination alert): It was Napoleon who advised to never interrupt


your enemy when he is set upon the wrong path. When it comes to defence, the
absence of a necessary requirement can be as fatal as the presence of a malignant
agent within our territory. White, hoping to simplify further, just blundered in time
pressure. How did Black exploit it?

Answer: Attraction/discovered attack.


40 ... Rxc3! 0-1
Black wins a piece, since 41 Qxc3?? hangs the queen to 41 ... Ng4+.
Summary
Earning equality isn’t the issue when dealing with Coward’s variation. Our main problem
is how to infuse a touch of dynamism to the game, if we need to play for a win with
Black.

Game 15
Z.Mammadov-G.Guseinov
Nakhchivan 2015
1 e4 d6 2 d4 Nf6 3 Bd3 e5
Once again, I urge calm. We soon transpose to the Modern/Pirc Defence.
4 c3 g6 5 Nf3 Nbd7 6 0-0 Bg7 7 Nbd2 0-0 8 Re1 b6
This is a reliable set-up for Black. Our idea is to either force White to resolve the
central tension, or face a potentially radical shift in the structure, as we see this game.
9 Qc2
9 a4 is White’s main move in the position and met with 9 ... a6.

Note: We are already familiar with this structural idea from Chapter One, where
Black reacts to h4 with ... h6. The idea with Black’s last move is the same: we meet
a5 with the bypassing ... b5.

With 10 b3 White hopes to find a home for his dark-squared bishop on either a3 or b2.
After 10 ... Bb7 11 dxe5 dxe5!? (Shirov wants to keep the game complicated, and so he
avoids the simplifying/equal route 11 ... Nxe5 12 Nxe5 dxe5 with a dull equality) 12 Ba3
c5! 13 Qc2 Qc7 14 Nf1 (intending Ne3, c4 and Nd5) 14 ... Rfd8 15 Ne3 Nf8 16 c4 Rxd3!
the game looks dynamically balanced. I like this exchange sacrifice, where Black gets a
pawn plus the bishop-pair and also destroys White’s grip on d5, P.Leko-A.Shirov, Frankfurt
(rapid) 1996.
9 ... Bb7 10 Nf1
Don’t be afraid of 10 d5 since a closed centre usually favours Black, who can then
leisurely play for an ... f5 break in King’s Indian style.
10 ... Re8 11 Bd2 exd4!
Tip: Remember this unbalancing idea, which infuses a bit of fun back into the
normal tedium of Coward’s variation.

Normally such a move violates the Principle: don’t hand the opponent central control,
but if we followed that one all the time, we would be banned from playing the Modern
Defence altogether, since the opening’s hypermodern nature is to cede central control to
our opponent, and then work vigorously to dismantle it.
12 cxd4
After 12 Nxd4 Ne5 if White attempts to hang on to his bishop-pair with 13 Bb5?! then
comes 13 ... c6 14 Ba4 b5 15 Bb3 a6, threatening ... c5 and ... c4, trapping a piece.

Did You Know? This trap in its many permutations is called the Noah’s Ark Trap.

After 16 f4 Ned7 17 Nf3 c5! Black simultaneously threatens the Noah’s Ark with ... c4
and also e4 hangs. Following 18 e5 dxe5 19 fxe5 Nd5 20 c4 N5b6 White is overextended.
12 ... c5!
Now the game looks much more like a sharp Benoni or Ruy Lopez, than the tedious
landscape of Coward’s variation.
13 Ng3
13 d5?! is met with the tricky 13 ... c4! 14 Bxc4 (14 Qxc4 Rc8 15 Qa4 Nc5 16 Qa3
Ncxe4 heavily favours Black) 14 ... Rc8 (White experiences huge problems with the pin)
15 a4 Ba6 when White is losing back his extra pawn, with a clearly inferior game.
13 ... cxd4!
Tip: In such structures Black often voluntarily takes on an isolani on d6. First, the
pawn isn’t so weak, since Black may be able to dissolve it with a future ... d5.
If that isn’t possible, Black’s enhanced piece activity easily makes up for the isolani.

14 Nxd4 Rc8 15 Bc3 h5!


With this multipurpose pawn sacrifice:
a) Black gains time with ... h4, undermining White’s knight, and by default, weakening
White’s e4-pawn.
b) Black can keep pushing the pawn to h3, which weakens White’s king.
16 Nb5!?
Double attack, on d6 and a7. Black must hand over a pawn, but this isn’t the entire
story: Black also obtains a ferocious initiative in this version. Maybe White was better off
going for a semi-sound fishing expedition with 16 Qd2 Ne5 17 Bc2 h4 and now 18 Ngf5!?
gxf5 19 Nxf5. Of course the comp will like Black’s side due to the extra material, but for
us fallible humans, White’s attack isn’t going to be all that easy to suppress.
16 ... Ne5 17 Nxa7 Rc5!
Guseinov sees deeply into the position and allows his rook to be trapped.
18 b4!?
Tip: That which threatens one sector of the board may also endanger the whole.
Oddly enough, this queenside thrust indirectly affects White’s own king’s safety.

This is a case of ambition exceeding resources, and is the same as Noah deciding to
build his ark, but only after it begins to rain. I don’t fault White, since continually
accepting little defeats can be interpreted as worse than death itself. Black must hand
over the exchange, which he happily does on his next move. Somehow more depressing
is to go passive with the line 18 f3 h4 19 Nf1 h3 20 Ng3 hxg2 and if 21 Kxg2?! d5! White’s
king doesn’t have a prayer.
18 ... Rxc3
Now White’s defensive task is the part of the movie where on the Hudson River,
except in this movie, everyone on board drowns. The birds choke up both engines and
Captain Sully is forced to try and land the plane
19 Qxc3
From a materialist’s stance, White is in good shape, since he is up an exchange and a
pawn. In this position, however, Black’s wicked initiative and coming attack greatly
outweigh the material factor.
19 ... h4
Also promising is 19 ... Nd5! 20 Qb3 Nxd3 21 Qxd3 Bxa1 22 Rxa1 Nf4 23 Qf3 Qf6! 24
Re1 h4 25 Nf1 Ra8 26 Nb5 Rxa2 27 Nc3 Nh3+! (overloaded defenders) 28 Kh1 Qxf3 29
gxf3 Rxf2 with a completely winning position for Black, since f3 falls.
20 Nf1 h3
This move punctures the light squares around White’s king.
21 Bb5
21 g3?? is destroyed by 21 ... Nf3+ 22 Kh1 Nxe4.
21 ... hxg2 22 Ng3

Exercise (critical decision): Black’s rook is under attack. Should we bother to


move it, or should we go all out for White’s king with 22 ... Nfg4?

Answer: Guseinov prepares an ingeniously prepared tactical confection, where


Black’s attack greatly outweighs White’s material.
22 ... Nfg4! 23 Kxg2
Not 23 Bxe8?? Qh4 and White is mated.
23 ... Qh4
Threatening mate in two moves.
24 h3
Exercise (combination alert): How did Black continue his attack?

Answer: Annihilation of the king’s cover/discovered attack.


24 ... Nxf2!
Black continues to attack with the one-track-mindedness of Fate itself.
25 Bxe8
25 Kxf2 Ng4+! 26 hxg4 Bxc3 27 Bxe8 Bxa1 28 Rxa1 Qf6+ pops the white rook in the
corner.
25 ... Qxh3+!
Black’s open lack of remorse at past (and present) sacrificial indulgences infer that he
is a firm believer in the saying: what’s done is done.
26 Kg1
26 Kxf2 Ng4+ picks up White’s queen.
26 ... Ned3!
Interference. Now White’s g3-knight hangs. 26 ... Nfd3! is also crushing.
27 Bxf7+
Obviously White’s desperation is in full bloom. This is one of those times when we are
unconsciously preoccupied with seizing the initiative – even while completely on the
defensive!
27 ... Kh8
No thanks.
28 Qc7
28 Re3 Bxc3 is of course completely hopeless for White.
28 ... Qxg3+ 29 Kf1 Qh3+ 30 Ke2

Exercise: (calculation): Often our greatest moment of peril tends to arise when
we are in close proximity to the object of our desire. Go astray here and White’s
king may escape with our side down material; find the correct path and we force
mate in four moves. How should Black proceed?

Answer: 30 ... Nf4+!


Black’s elegant final sequence arrives with almost cathedral presence.
31 Kd2
31 Kxf2 Bd4+ 32 Re3 Bxe3+ 33 Ke1 Qh1 is mate.
31 ... Qd3+ 32 Kc1 Ne2+! 0-1
Attraction/Removal of the guard. 33 Rxe2 Qd1 is mate. “There is no greater time of
disillusionment as when you realize that your childhood hero is no god, but instead is
fallible, and therefore mortal,” whispers the queen into her older brother’s ear, as she
observes the effects of the poisoned wine take place.

Summary
Be alert to infuse dynamism into the otherwise dull landscape of Coward’s variation with
the structural shifting idea ... exd4, followed by ... c5.
Chapter Seven
The Austrian Attack
1 e4 g6 2 d4 Bg7 3 Nc3 d6 4 f4 a6

This chapter is not the place to wing it in the opening phase. In the Austrian Attack
White goes after our Modern with a central/kingside pawn avalanche with e4, d4 and f4.
At some point White usually lunges with e5, and odds are our opponent will either mate
us, or overextend.
I spoke reasonably lucid French when I lived in Montreal, 40 years ago. Then recently,
a French speaking tourist asked me: “Pardon-moi, monsieur ... .blah, blah, blah.” I only
understood the first three words spoken. Moral: if you don’t practice, you lose a skill. This
goes for our openings, and especially for this chapter, with its dizzying array of lines,
subset and sub-subsets. I strongly urge you to study the lines of the Austrian Attack more
carefully than any other in the book.

Game 16
H.Tsang-C.Lakdawala
San Diego (rapid) 2010

1 e4 g6 2 d4 Bg7 3 Nc3 d6 4 f4
Your writer, more a diplomat than a soldier, sometimes feels out of my element in the
Modern, especially when opponents bang out the dreaded Austrian Attack. I wish
congress and the president would pass legislation making the choice of playing Austrian
Attack against innocent positional players (i.e. me) a hate crime. In my opinion, this is
the scariest possible way for White to meet our Modern. Our job is to overextend White
before he or she mates us.
4 ... a6
We go with our normal ... a6, ... b5 set-up.

Tip: 4 ... c6?! which works in almost all other versions of the Modern is, to my
thinking,
too slow and I strongly recommend that we avoid it versus the Austrian.

One example: 5 Nf3 b5 (5 ... Bg4 6 Be3 Qb6 7 Qd2 Bxf3 8 gxf3 Nd7 9 0-0-0 is also
very much in White’s favour) 6 Bd3 Nd7, V.Anand-F.Caruana, Zurich (blitz) 2015. White
enjoys an unchallenged central space advantage after 7 0-0 and Black has trouble
developing his kingside, since ... Ngf6 is strongly met with e5: for example, 7 ... Nh6 8 h3
Nb6 9 Qe1 with a clear advantage for White.
5 Nf3 b5
Tip: Avoid the inaccurate move order 5 ... Nd7?! when White seizes a serious
advantage
with 6 a4! b6 7 Bc4! e6 8 f5! (principles: create confrontation and open the position
when leading in development) 8 ... exf5 9 exf5 Ndf6 10 fxg6 hxg6 11 Bg5 Bb7
12 0-0 with huge pressure for White along the open f-file, N.Olenin-S.Ovsejevitsch,
Alushta 2000.

6 Bd3 Nd7

Note: The knights before bishops rule applies, since Black is better off keeping his
bishop on c8 for a little longer. Why? Because if White plays an e5 and e6 sacrifice,
and we accept with ... fxe6, then our bishop on c8 helps covers our e6-pawn.

7 0-0
This early castling isn’t supposed to be all that dangerous for Black. Next game we
look at 7 e5 c5 8 Be4.
7 ... Bb7?!
I realized that I goofed up, the second I touched my bishop.

Tip: Black’s last move is inaccurate, since it weakens the e6-square. The Austrian
Attack argues extreme caution, since a single inaccurate move can prove fatal.

GM Tiger shows us the correct path with 7 ... c5! 8 e5 cxd4 9 Be4 Rb8 10 Nxd4 dxe5!
11 Nc6 Qb6+ 12 Kh1 Ngf6! (Tiger correctly offers the exchange to destroy White’s centre;
12 ... Rb7? 13 fxe5 leaves Black’s king in a bad way) 13 Nxb8 Qxb8 14 Bf3?! (this move
hands Black a pawn; better is 14 Bc6 0-0 when Black obtains full compensation for the
exchange, with a pawn and central majority) 14 ... exf4 15 Nd5 Nxd5 16 Bxd5 e5 and
Black had an excellent position, with two healthy central pawns for the exchange,
R.Burton-T.Hillarp Persson, Guernsey 2010. White can win one back, but at the cost of his
king’s safety after 17 g3?! 0-0! 18 gxf4 Nf6 19 Bg2 e4 with massive strategic
compensation, as well as one pawn for the exchange.
8 e5! c5!?
The danger with this idea is that Black wilfully violates the principle to avoid opening
the game when behind in development. The argument for playing the move is that we
must dismantle White’s centre quickly, before it overwhelms our side. The alternatives:
a) 8 ... e6?! is a safer alternative, but I don’t really like Black’s game after 9 a4 b4 10
Ne4.
b) 8 ... Nh6! brings Black one step closer to castling: 9 a4 b4 10 Ne4 0-0 when ... c5 is
coming and Black has reasonable chances.
9 Ng5!?
My opponent lunges at me with disconcerting directness. White’s best continuation is
9 Be4!.

Note: This Be4 manoeuvre, swapping off Black’s most active piece, is very common
in Austrian lines.

Here 9 ... Bxe4 10 Nxe4 cxd4 11 exd6 is an edge for White, A.Grischuk-V.Milov,
Torshavn 2000.
9 ... cxd4?!
I am the kind of person who tends to make major life decisions based upon movies I
have recently seen, in this case Titanic. 9 ... Nh6! was absolutely necessary. Black looks
okay after 10 e6 Bxd4+ 11 Kh1 Nf6 12 Ne2 fxe6. Here 13 c3 fails to trap Black’s bishop
after 13 ... c4!. However, Black must avoid 9 ... e6?.
Warning: In such positions were you play ... e6 to blunt White’s attacking ambition,
always factor in the weakening of your d6-square, which here is fatally weakened.

10 Nce4 (threatening to invade d6) 10 ... Bxe4 11 Nxe4 (same threat) 11 ... Bf8 (if
this move has to be played, then something has gone horribly wrong for our side) 12
dxc5! follows the principle: open the game when leading in development. In this case it
leaves White with a winning position.
10 e6! fxe6
The ground feels unsteady, the way if feels when we step off the cruise ship, after a
week on the rocking ocean. Maybe I should have tried to muck it up further with 10 ...
f5!? 11 exd7+ Qxd7 12 Ne2 h6 13 Nf3 e5. The comp likes White due to the extra piece.
For a fallible human, navigating White isn’t so easy, since he must deal with Black’s
lunging central pawn mass.
11 Nxe6 Qb6
I must not fear. Fear is the mind killer ... Oh, sorry, I was just reciting the Dune/Paul
Atreides mantra in order to remain calm. By now I had descended into a Sylvia
Plathesque level of depression and my main hope was that my lower-rated opponent
might be as confused as I was.
12 Ne4
After 12 Nxg7+ Kf7 13 Ne6 dxc3+ 14 Kh1 Ngf6! (if 14 ... Kxe6?? 15 f5+ and Black’s
exposed king doesn’t have a prayer) 15 Qe2 Rhe8 16 f5 Ne5 17 bxc3 Qc6 maybe I’m
crazy, but I dispute the comps’ favourable assessment for White and would be okay with
Black’s position.
12 ... Bf6 13 f5?!
This move hands Black control over e5. Better was 13 Re1.
13 ... Ne5 14 Bg5!?
Attempting to break down Black’s grip over f6.
14 ... Bd5
The convolutions of some positions surpass that which our limited minds are capable
of understanding at the board. Only later, when we examine the game with time and
comps do we begin to truly understand. I desperately wanted to eject his e6-knight from
my position. However, after 14 ... Bxg5! 15 N4xg5 Nf6 16 fxg6 hxg6 Black may actually
stand slightly better.
15 Kh1
Maybe he should try 15 Nxf6+! Nxf6 16 Nf4 Bf7 17 Qe1.
15 ... Bxg5 16 N4xg5
16 N6xg5 is met with 16 ... 0-0-0 with nightmarish complications.
16 ... Nf6 17 Nxd4! h6?!
By now you know that I am not a stranger to disenchantment and crushed dreams.
This is an overreaction. Black should respond with 17 ... gxf5! 18 Nxf5 Rg8, but certainly
not 17 ... Qxd4??, which hangs the queen to the basic discovery trap after 18 Bxb5+.
18 Nge6 g5
Sealing the kingside and denying White’s pieces use of f4.
19 c3?!
Now the advantage swings to Black. 19 a4! is promising for White.
19 ... Qb7!
Targeting g2. The vague outline of the combinational pattern begins to emerge, even
before I visualize the whole.
20 Rf2?
He had to try the visually repulsive 20 Nf3 Nxf3 21 gxf3 g4 22 Be2 when Black stands
better.
20 ... Neg4 21 Re2

Exercise (combination alert): Black turned everything around with a


combination here. Where is it?

21 ... Bxg2+!
“It’s better to have an honest critic who informs you of an unpleasant truth, than a
false friend who tells you exactly what you want to hear,” rationalizes the bishop.
22 Kg1
22 Rxg2?? is met with 22 ... Nf2+ picking up White’s queen.
22 ... Bd5 23 b3
He wants to eject the bishop with c4.
23 ... Ne5 24 c4

24 ... Bxe6!?

Tip: It has been my experience that in irrational positions our survival chances rise
when we play for board control, over material.

Also tempting was 24 ... Nf3+ 25 Nxf3 Bxf3 26 Qf1 Bxe2. I felt intuitively that White
still had some light-square practical chances/compensation for the exchange in this
version.
25 Nxe6
One may argue that White’s e6-knight is a powerful piece, gainfully employed, but in
my opinion, not gainfully enough, since it fails to actually attack anything. Black can
basically work around it. It kind of takes on the role of a 90-year-old billionaire’s 23-year-
old trophy wife, whose job is to just show up and look pretty at all times.
25 ... Kf7
Now ... Nf3+ is a huge problem for White.
26 cxb5 axb5 27 Rxe5!?
It’s human nature to lash out when we are in pain. This appears to be a desperate
attempt to shake off the decay engulfing White’s position. White’s game is hopeless if he
doesn’t play this move, since Black’s e5-knight is too powerful and his king fatally
exposed. 27 a4 bxa4 28 bxa4 Rhb8 29 Bc2 Nf3+ 30 Kf2 Nxh2 is hopeless for White.
27 ... dxe5 28 Qe2 Rhb8 29 Qxe5 Qd5?
Offering the exchange back to reach a winning ending, was my idea. At this point,
giddy with relief, I gave thanks to the goddess Caissia for having escaped punishment in
this game, despite my multiple opening indiscretions. What I didn’t realize was that my
last move endangered the win. Correct was 29 ... Qb6+!.

30 Qe2? 0-1
Now his game is once again hopeless. Both players missed that White has 30 Qxd5!
Nxd5 31 Be4 Nc3 32 Bxa8 Rxa8 33 a4 bxa4 34 Ra3! (I only calculated 34 bxa4? Rxa4 with
an easy win for Black, since White’s f5-pawn falls) 34 ... Nb5 35 Rxa4 when he still has
chances to still save the game.

Summary
The Austrian Attack is perhaps the most dangerously chaotic line versus our Modern. If
we forget our analysis (as I did this game), we may end up losing a miniature.

Game 17
Z.Jovanovic-R.Rapport
Austrian League 2015

1 e4 g6 2 d4 Bg7 3 Nc3 a6 4 f4 b5 5 Nf3 d6 6 Bd3 Nd7 7 e5


The move orders in the Austrian can be next to impossible to get down, but with
familiarity and study, we get closer and closer to the goal. Next game we look at this
position with the inserted moves Be3 and ... Bb7.
7 ... c5
Once again we immediately strike at White’s centre, despite the fact that our move is
in flagrant violation of the edict to avoid opening the game while lagging in development.
7 ... Bb7 doesn’t score as well for Black after 8 Qe2.
8 Be4

Note: In the Austrian this disruption idea is played, with or without Black’s
bishop on b7.

Instead, 8 Be3 Bb7 9 Be4 reaches the position we look at next game.
8 ... Rb8 9 Be3
Alternatives:
a) 9 0-0 cxd4 10 Qxd4 (or 10 Nxd4 dxe5 11 Nc6 Qb6+ 12 Kh1 Ngf6! 13 Nxb8 Qxb8 14
fxe5 Nxe5 15 Bf4, as in S.Karjakin-T.Hillarp Persson, Benidorm (rapid) 2003; Black
obtained decent compensation for the exchange and should continue 15 ... Nxe4 16 Nxe4
0-0 17 Qd4 Bf5 18 Ng3 Be6 19 Rae1 Nc6! escaping the pin) 10 ... Nh6 11 Qa7 0-0 12 Rd1
b4 13 Na4 Bb7! 14 Bxb7 Qc7 15 exd6 exd6 16 Qxa6 Rxb7 17 Be3 Nf5 18 Bf2,
A.Kovacevic-T.Hillarp Persson, Budva 2009. Black’s super active pieces and open
queenside lines are worth more than White’s pawn after 18 ... Nf6 19 Nd4 Ne4!.
b) 9 dxc5 (White scores poorly in this line, which voluntarily swaps d-pawn for Black’s
lesser c-pawn) 9 ... Nxc5 10 Bc6+ Bd7 11 Bxd7+ Qxd7 12 Be3 Rc8 and Black stood no
worse in A.Crut-B.Badea, Saint Affrique 2002.
c) 9 Ng5 is dangerous. White wants to slide in e6: 9 ... Nh6 10 e6 Nf6 11 exf7+ Nxf7.
Tip: One reason to play ... Nh6 is that it suppresses White’s f5. The second reason
is seen here: our f7-square is given much needed support.

Following 12 Bc6+ Kf8! (12 ... Bd7? allows 13 Ne6! with advantage to White) 13 dxc5,
as in J.Furhoff-T.Hillarp Persson, Stockholm 2007, Black’s pieces coordinate just fine in
this admittedly crazy position.
9 ... b4 10 Ne2
10 Na4 can be met with the unplayed novelty 10 ... Nh6!, and if 11 dxc5 Ng4 12 Bd4
dxe5 13 fxe5 Ndxe5 when Black looks good due to his extra central influence.
10 ... Nh6
A multipurpose move, developing, suppressing f5, reinforcing f7, and preparing to
castle.
11 dxc5
Tip: In the Austrian, when they play dxc5, we often respond with ... dxe5 (in this
case with a zwischenzug added with ... Ng4).

Instead, after the 11 h3 0-0 12 c3 of R.Forster-E.Bacrot, Bermuda 1999, Black should


continue 12 ... bxc3 13 bxc3 cxd4 14 cxd4 Nb6 with growing control over the central light
squares, which compensates for White’s space.
11 ... Ng4 12 Bg1
12 Qd3 dxe5 13 c6 Ndf6 14 Qxd8+ Kxd8 15 Ba7 Kc7! transposes to the game
continuation.
12 ... dxe5 13 c6!
White counted on this new move, but Black stands fine, according to Komodo. Still it
looks like an improvement over 13 fxe5 Qc7 14 e6 fxe6 15 Ned4?! Nxc5 16 Qe2 0-0 17 0-
0-0 Qf4+ 18 Nd2, M.Hoffmann-H.Ree, Dutch League 2001. White is busted after 18 ...
Nxe4 19 Qxe4 Qxe4 20 Nxe4 Bb7!.
13 ... Ndf6
Now defence of the dangling e4-bishop is a problem for White.
14 Qxd8+ Kxd8 15 Ba7
A move made with the thought: if a person is unable to earn, then he must borrow; if
he is unable to borrow, then he must steal. Sometimes the only way to finance our
strategic debt is to pause to swipe material.
15 ... Kc7!
Note: A very common theme in the Austrian attack is for Black to hand over
the exchange for the bishop-pair, dark squares and a pawn. In this case Black
gets full compensation.

The Art of War advises us to appear weak when we are strong, and to act strong
when we are weak. This is an example of the former. Confusing variable upon variable
piles on. Strategic anomalies like this one make us realize the truth of the Heisenberg
Uncertainty Principle. Our opening system is not something to be studied and solved, but
instead an experience to be absorbed via the osmosis factor of over-the-board play.
It’s really difficult to get your bearings in this line and your semi-senile writer often
stumbles about, feebly grasping to any strategic guide post within the Austrian Attack. In
this case just realize that an exchange sacrifice is often baked into our line, but not here
with 15 ... Rb5? 16 Bd3 Rd5 17 Bc4 and White wins the exchange after all, under far more
favourable circumstances.
16 Bxb8+ Kxb8
If White fails to find the perfect move, his position suffers serious diminishment.
17 Nd2?
Now the happy haze of the narcotically induced dream comes to an end. 17 h3! was
the only way. After 17 ... Nxe4 18 hxg4 exf4 I actually slightly prefer Black’s position,
since White’s pawns look loose and Black’s bishops work well in the open position.
17 ... exf4
Also highly favourable to Black is 17 ... Ne3 with direct and indirect attacks on g2 and
c2.
18 Bd3
18 Nxf4 Bh6 19 g3 Rd8 20 Bf3 e5 21 Nd3 Bxd2+ 22 Kxd2 e4 and Black upgrades from
an exchange down to picking up two pieces for a rook.
18 ... e5

White’s strategic woes which are on prominent display, remind us of the drunk driver
who runs over a mother and her baby, who then tells the judge: “Your honour, what
about the fact that I had an excellent driving record before this incident?”. Advantage
Black:
1. Who has a solid pawn for the exchange.
2. Whose massive 5-2 kingside majority begins to roll.
3. Who has access to a supported knight outpost on e3.
4. Who rules the dark squares.
19 a3?!
This move doesn’t have the feel of a profit-generating exercise. White hopes to
activate his a1-rook and gets the exact opposite of his intent. 19 h3 was necessary.
19 ... e4!
Rapport finds a powerful idea, despite the confusing and contradictory data. This
move opens the unopposed g7-bishop’s diagonal and inflicts terrible damage upon
White’s queenside.
20 Nxe4 Nxe4 21 Bxe4 Bxb2 22 Rb1
22 0-0 is met with 22 ... bxa3! with a winning position for Black.
22 ... bxa3
It’s just a matter of time before the passed a3-pawn costs White material.
23 0-0 g5 24 Nc3
White hopes to regain some measure of security by seizing control over a2, but as we
soon see, this is merely a temporary state of relief. Instead, if 24 Rxb2+ axb2 25 Rb1 Kc7
26 Rxb2 Rd8 (threatening mate) 27 Nc3 f5 28 Bd3 Ne5 29 Be2 Nxc6 with two extra pawns
for Black.
24 ... Ka7 25 Rxb2
White’s choices are to dig in for defence or launch a desperate counter-sacrifice. His
problem is that the chances of success in either, feel equally remote. Also hopeless was
to hang on to the exchange and remain bound up for life: for example, 25 Na4?? loses to
25 ... Bd4+ 26 Kh1 a2.
25 ... axb2 26 Rb1 f5 27 Bd5 Re8
Also strong is 27 ... Rd8 28 Rxb2 Ne3 29 Bf3 Rd2.
28 h3
28 Rxb2?? meets with 28 ... Re1 mate.
28 ... Nf6 29 Rxb2

Exercise (combination alert): Find one move and you overwork the white
defences.

Answer: Overloaded defender/simplification. Black has a choice of two different wins.


29 ... Re3!
Black attacks the sole defender of White’s wobbly d5-bishop.
30 Rb3 Nxd5!
Rapport goes for the path of simplification over material, which is stronger than the
also winning 30 ... Rxc3 31 Rxc3 Nxd5.
31 Nxd5 Rxb3 32 cxb3 Be6
White will lose both his queenside pawns, after which he is helpless against Black’s
passed a-pawn.
33 Ne7
The knight comes to the rescue, but who will rescue the rescuer? Now his knight,
which stands out like a lone building in an otherwise flat landscape, gets trapped.
However, after 33 Nf6 Bxb3 34 Nxh7 a5 35 Nxg5 a4 36 Nf3 a3 37 Nd2 (or 37 Nd4 Ba4!)
37 ... Bc2 the clumsy knight is unable to stop Black’s a-pawn.
33 ... Kb6! 0-1
Now if 34 b4 Kc7 35 Kf2 Kd6 when White’s knight runs short of safe squares.

Summary
Be aware that in many lines of the Austrian Attack, Black is obliged to sacrifice an
exchange. When we do so we usually gain a pawn, the bishop-pair and white pawn
targets, generally offering us full compensation.

Game 18
Wei Yi-M.Carlsen
Bilbao 2016

1 d4 g6
When a world champion takes up the mantle of our opening, we take it as an article
of faith in its inherent soundness.
2 e4 Bg7 3 Nc3 d6 4 Be3 a6
So far we are transported back to Chapter One of the book, but this alters with
White’s next move, which transposes to the realm of the Austrian Attack.
5 f4 b5 6 Nf3 Nd7 7 e5 Bb7 8 Bd3 c5
The dismantling of White’s centre is top priority. We reach a crucial tabiya position of
the Austrian Attack. In The Modern Defence: Move by Move, I focused on 8 ... Nh6 where
Black played for a light-square strategy without opening the centre early.
9 Be4
White plays the familiar manoeuvre, depriving us of our best piece. 9 0-0 is known to
give Black no trouble at all after 9 ... cxd4 10 Bxd4 Nh6, reaching dynamic equality. Don’t
fear 11 e6 which can be met with 11 ... Bxf3! 12 exd7+ Qxd7 13 Bxb5 axb5 14 Rxf3 0-0
and Black’s extra central pawns should give him at least equality, if not a shade more.
9 ... Bxe4 10 Nxe4 Nh6

Tip: When defending the black side of Austrian Attack, it’s crucial to avoid falling
too far behind in development. Avoid non-developing/game opening moves like
10 ... cxd4?! 11 Bxd4 dxe5 12 fxe5 Nh6 13 e6! Nf6 14 Neg5 with a big
advantage for White.

11 dxc5
After 11 Qe2 cxd4 12 Bxd4 dxe5 13 fxe5 Nf5 14 e6 Nxd4 15 Nxd4 Bxd4 16 0-0-0! Ne5!
17 exf7+ Nxf7 18 c3 0-0 19 Rxd4 Qb6 Black’s activity makes up for the slight weakness of
his e-pawn isolani.
11 ... dxe5 12 c6
White chases away a defender of e5.
12 ... Nf6
Nobody has ever tried the contortionist theoretical novelty 12 ... Nf8!? 13 Qxd8+ Rxd8
14 fxe5 Rc8. Black’s game looks just fine to me.
13 Qxd8+ Rxd8 14 Nxf6+ exf6 15 c7 Rc8 16 Bb6
This is scary stuff, since now White plans to transfer a rook to d8.
16 ... Kd7!
When we face an obstacle we have two choices:
a) Issue a direct challenge and attempt to go through it.
b) Save energy and go around it.
Carlsen’s last move is an example of number two on the list. He
utilizes the endgame principle which guides us to move our king
out of the safety zone and put it to work in an ending. In fact in
this game Carlsen’s hardworking king moved a whopping total
of 11 times, like a temporary labourer who is hired when
demand for goods is high, and then laid off and discarded in the
off season.
17 Ba5
17 0-0-0+ Kc6 18 Ba5 Nf5 transposes to the game.
17 ... Nf5
This isn’t the time to get greedy with 17 ... exf4?! 18 0-0-0+ Kc6 19 Rd8. Black must
hand over the exchange. If he doesn’t with 19 ... Nf5? then follows 20 Rhd1 Kb7 21 R1d7
with a winning bind for White.
18 0-0-0+ Kc6 19 Rd8
Exercise (planning): White threatens to increase his choke hold with Rhd1.
Carlsen found an efficient way to dismantle White’s bind. What would you play here?

Answer: Transfer the knight to either c4 or b7, which dislodges White’s bishop, and
therefore White’s bind.
19 ... Nd6! 20 fxe5 fxe5 21 Rd1
21 b3? keeps the knight out of c4, but not b7: 21 ... Nb7 22 Rxh8 Bxh8 23 Bc3 f6 and
the c7-pawn falls.
21 ... Nc4 22 Bc3 Rxc7
With this move Black completely equalizes. Even stronger was 22 ... b4! 23 Rxh8 Bxh8
24 Bxb4 e4 (simultaneously attacking f3 and b2) 25 Nd4+ Kb7 26 Bc3 Rxc7 when Black’s
majority and piece activity are more potent than their white counterparts.
23 b3 Ne3 24 Bxe5
24 R1d6+ Kb7 25 R8d7 Kc8 26 Rxc7+ Kxc7 27 Rd3 Ng4 holds everything together,
since 28 h3?! is met with 28 ... Nf2 29 Ba5+ Kc6 30 Rc3+ Kd6 when it is Black who
stands better due to his functional kingside pawn majority.
24 ... Nxd1!?
Any world champion’s great weakness is that they know nothing but victory and
success, the logical extension of which is that there is no place to go but down if they
continue to play. When great players plunge into unfathomable complications, it falsely
presupposes that they have a duty to entertain us with dazzling and risky ideas. The truth
is their main objective is simply to win.
In our electronic and wireless chess world, news of a new idea travels fast. A couple of
months later 24 ... Rxd8 25 Rxd8 Bxe5 26 Nxe5+ Kb7 27 Rd2 f5 led to even chances in
the ending, S.Vokhidov-A.Bodnaruk, Baku 2016.
25 Rd6+
The ending is still even. Wei’s main obstacle is that he faces the greatest natural
endgame genius since Capablanca and Fischer.
25 ... Kb7 26 Bxg7 Rg8 27 Bd4 Nc3
Black won the exchange, but as Wei demonstrates, White wins it back.
28 Rb6+! Kc8 29 Be5! Rd8
Black must hand back the exchange since 29 ... Rc5?? hangs material to 30 Rb8+ and
29 ... Rb7?? gives up two pieces for a rook after 30 Rc6+ Kd7 31 Rxc3. Instead, 29 ...
Nxa2+ 30 Kb2 Nb4 31 Bxc7 Kxc7 32 Rf6 sees White maintain the balance.
30 Kb2
After 30 Rxa6?? Rd1+ 31 Kb2 b4! (threatening mate on b1) 32 Bxc3 bxc3+ 33 Ka3
Rd2! White’s c-pawn falls and Black’s resulting passed c-pawn will win the game.
30 ... Nd5 31 Bxc7
31 Rxa6?? is met with 31 ... Rxc2+! 32 Kxc2 Nb4+ 33 Kc3 Nxa6 and this time Black is
going to stay the exchange up.
31 ... Kxc7!

Brilliant judgement. Carlsen sees that White’s future awkward rook’s position on a6
more than makes up for Black being down a pawn. 31 ... Nxc7?! 32 Rf6 places Black on
the defensive.
32 Rxa6 Kb7 33 Ra3 Ne3 34 c4!
Principle: activate and push your pawn majority with vigour. 34 g3? is too slow. Black
seizes a decisive initiative after 34 ... Nd1+ 35 Kc1 Nc3. White is curiously helpless, while
Black threatens ... b4, followed by a rook check on d1 and mate on b1.
34 ... bxc4
The ending is still even here, but after White’s next move mistake, little by little, his
capacity for self-determination decreases.
35 Ne5?
Correct was 35 b4! Nxg2 36 Ne5 when the position remains tense and approximately
balanced.
35 ... Rd2+ 36 Kc3 Rc2+! 37 Kd4
If 37 Kb4 Nd5+ 38 Kc5 c3! and White can’t stop the c-pawn’s surge without loss of
heavy material.
37 ... Nf5+ 38 Kd5
The immediate 38 Kc5?? walks into the discovered check 38 ... cxb3+.
38 ... c3!
White’s pieces are oddly out of synch, when it comes to halting the surging c-pawn.
39 Kc5

Exercise (combination alert): An annotator’s job is to explain a position with


clarity,
but I confess that at this point I’m having trouble forming ideas into even a single
rational sentence. Carlsen found a study-like way to win. Take a shot at trying to
solve it. Should Black play 39 ... Rxg2, or does the solution lie elsewhere?

Answer: 39 ... Nd6!!


We witness a unique alignment of geometric stars. How inadequate our own chess
feels when measured against a player of monumental natural talent. It isn’t as if Carlsen
has large reserves of capital to invest, but here he offers a knight, which can’t be
touched. Carlsen avoids Wei’s insidious trap 39 ... Rxg2? 40 Nc6! c2?? (40 ... Ng7! is
forced) 41 Ra7+ Kc8 42 Kb6! c1Q (this extra queen isn’t of much help for Black) 43 Rc7
mate.
40 Nc6!
When heavily stressed, our mind can either suffer panic/depression, or turn
courageous/inventive, as we see here. On the one hand we are paralyzed, and on the
other, we experience a kind of hyperconscious state, where we turn our resistance into a
genetically enhanced super-soldier. In inferior positions it’s crucial to remain alert yet
calm, in a state of preparedness to seize upon any future chance to disturb the
opponent’s equilibrium, as Wei tries here, while avoiding 40 Kxd6?? Rd2+ 41 Ke7 c2 and
nothing can stop promotion.
40 ... Ne4+?!
Stronger is 40 ... Kc7! when a rook check on a7 is met with a knight block.
41 Kb5
White hopes for a mating pattern, but when we crunch the numbers, we see that
there is no way to implement his intent.
41 ... Kc7!
41 ... Rxg2?? walks into the same mating pattern as the above note, after 42 Ra7+
Kc8 43 Kb6 c2 44 Rc7 mate.
42 Ra7+ Kd6
Carlsen’s king is safe from attempted mating nets and his pawn is ready to surge
forward.
43 Ra4
White’s best chance to put up resistance lay in 43 Nb4 Rxg2 44 Rxf7 Rxh2 45 Rf1.
43 ... f5! 44 Kb6 Rxg2 45 Rd4+ Ke6 46 a4
It looks like a close race between White’s a-pawn and Black’s c-pawn, but analysis
proves this just isn’t the case.
46 ... Ra2?!
Stronger was 46 ... Rb2!.
47 a5 Rb2?!
This represents the loss of a tempo, since he could have played to b2 in one move,
rather than two.
48 Kc7?! Nc5!
Carlsen slows the advance of White’s passed a-pawn.
49 Rc4 Na6+! 50 Kb6
Exercise (combination alert): Find Carlsen’s winning idea.

Answer: Step 1: Force White’s king in front of his passed pawn by giving up a piece.
50 ... Rxb3+!
This is his second knight offer in the ending.
51 Kxa6 Kd5!
Step 2: Double attack.
52 Rb4!?
Clever tactics, but not good enough. After 52 Ne5 Kxe5 Black wins easily, since his
pawn can promote, while White’s can’t.
52 ... c2!
Step 3: Promote and win the queen versus rook and knight ending.
53 Ne7+ Kc5 54 Rxb3 c1Q 55 Rb6 Qc4+ 56 Kb7
If 56 Ka7 Qe4 57 Nc8 Qa4 58 a6 Qd7+ 59 Kb8 Qxc8+! (simplification) 60 Kxc8 Kxb6
and White’s precious a-pawn falls as well.
56 ... Qe4+ 57 Nc6 Qd5! 58 Kc7
58 a6?? Qd7+ 59 Kb8 Kxb6 and mate in two moves.
58 ... Qd6+ 59 Kb7
This is the part of the movie where the contrite, cheating husband tells his aggrieved
wife: “I realize I don’t deserve your forgiveness, but I’m asking anyway.”
59 ... Qd7+ 60 Kb8 f4! 0-1
White’s a-pawn is weirdly frozen on its square and so he resigned. 61 Nb4 f3 62 Rb7
Qxb7+! 63 Kxb7 Kxb4 64 a6 f2 65 a7 f1Q 66 a8Q Qf3+ 67 Kb8 Qxa8+ 68 Kxa8 would be
totally hopeless.

Summary
The position after 8 ... c5 is probably the most critical for us in the entire Austrian Attack,
so let’s learn it thoroughly.
Chapter Eight
The Dzindzi-Indian
1 d4 g6 2 c4 Bg7 3 Nc3 c5 4 d5 Bxc3+ 5 bxc3 f5

We enter the realms of the queen’s pawn version of the Modern. In this chapter we
look at 1 d4 g6 2 c4 Bg7. The Dzindzi-Indian can only be played when White opts for 3
Nc3 before playing e4. We continue with 3 ... c5 4 d5. Am I advocating the Benoni? No. I
suggest we enter the slightly offbeat Dzindzi-Indian line with our favourite idea: we once
again give White our powerful dark-squared bishop to inflict damage with 4 ... Bxc3+!? 5
bxc3 and now we toss in 5 ... f5, which inhibits (but doesn’t fully prevent) White’s e4
push.
The resulting positions are closer to the Sämisch variation of the Nimzo-Indian than
they are to a Modern. I checked my database, and so far I have a perfect score with this
system and still believe it in.

Game 19
M.Ginsburg-C.Lakdawala
American Open, Los Angeles 2002

1 c4 g6 2 d4 Bg7 3 Nc3
Note: The Dzindzi-Indian can only be played against White’s early Nc3 move order.
Against 3 e4 which we look at next chapter, I advocate the Averbakh line 3 ... d6
4 Nc3 e5!?.

3 ... c5

Tip: If the thought of entering the Dzindzi-Indian gets you nervous, then you can
bypass this chapter altogether and simply play 3 ... d6 4 e4 e5 which is the
Averbakh System, which we cover in the next chapter.

4 d5
I don’t think 4 Nf3 is as accurate, since we obtain a favourable version of a Maroczy
bind after 4 ... cxd4 5 Nxd4 Nc6 6 Nc2 (White’s main move, but I absolutely love Black’s
game after our next move; instead with 6 e3 Nf6 7 Be2 d5! White ends up in an equal
line of the Grünfeld, Wang Hao-V.Anand, Stavanger 2013) 6 ... Bxc3+! 7 bxc3 Qa5 8
Bb2?! (White should offer the c3-pawn with 8 Ne3 when I would decline with 8 ... Nf6) 8
... Nf6 9 f3 d6 10 e4 Qa4! 11 Qd2 Be6 12 Ne3 Rc8 13 Nd5 Nd7 14 Be2 Nce5 15 0-0 Nxc4
16 Bxc4 Rxc4 17 Ne3 Rc8 18 c4 f6, and White lacked compensation for his missing pawn,
R.Ignacio-C.Lakdawala, San Diego (rapid) 2006.
4 ... Bxc3+!?
Let’s push White out of his or her easy routine. We continue to parade our defiance of
the assumption that our dark-squared bishop is a thing of great value, to be cherished
and protected at all times. This move turns it into a Dzindzi-Indian. We once again follow
our always-chop-c3 philosophy.
At first glance, it feels like our awful decision is the equivalent of the novelist who
begins with: “It was a dark and stormy night.” Empathy, or its lack, can play strange
tricks on us. We can watch the news and remain relatively unmoved, as we hear the
news of 100,000 people killed in a tsunami, and then go outside and be touched to tears
at the plight of a limping sparrow with an injured wing. The question is: just how much
empathy do we have for our (allegedly) precious dark-squared bishop? In the Dzindzi-
Indian, the answer is: not too much.
5 bxc3 f5

Did You Know? This line was invented by GM Roman Dzindzichashvili, who began
to play it in the mid-1980’s with success. We fight for control over e4.

This is the starting position of the Dzindzi-Indian, an infant line, just recently born,
which mixes ideas from the Nimzo-Indian and Leningrad Dutch. The parties pursue
distinctly opposing agendas, each seemingly insulated from the other’s. Let’s look at the
imbalances:
1. White owns the bishop-pair.
2. Since Black gave up the precious dark-squared bishop, White may later gain power
on the dark squares.
3. White’s c-pawns are doubled and vulnerable to attack, with ideas like ... Qa5, ...
Nbd7, ... Nb6 and ... Qa4. Of course, such pawns can also be simultaneously
assets/burdens, since White can try and make something of his open b-file, should Black
castle queenside.
4. The position is closed for now, which means that we can rightfully challenge the
assumption that White’s bishop-pair constitutes an actual advantage.
6 Bg5
This idea, borrowed from the Trompowsky, menaces d6 and dares Black to chase the
bishop with ... h6, which weakens Black’s pawn front around my king. And if Black plays
... Nf6, White will chop it, handing back the bishop-pair to devalue Black’s structure.
In the next two games of the chapter we look at 6 Nf3 and 6 e4!. That just leaves:
a) 6 Bf4 Nf6 7 Qc2 d6 8 e3 Nbd7 9 Be2 Qa5 10 f3 Nb6 11 Nh3 Bd7 12 Bg5 Qa4! 13
Qxa4!? (White is better off playing 13 Qb3) 13 ... Bxa4 14 Bxf6 exf6 15 Nf4 Kf7 16 Kd2
Rhe8 17 Bd3 Re7 18 Rae1 Rae8 19 Re2 Nd7 20 Rhe1 Ne5 21 e4?! (he should try 21 h4)
21 ... fxe4 22 Rxe4 Bd7! (threat: ... Bf5) 23 Bf1 f5 24 R4e3 g5! 25 Nh3 f4! 26 Nxg5+ (not
26 Re4?? Bxh3 27 gxh3 Nxf3+ and White can resign) 26 ... Kg6 27 Rxe5 Rxe5 28 Ne4 Bf5
29 Bd3 Bxe4 30 Bxe4+ Kf6 31 g4 h5! (principle: when you are up the exchange, your
rook or rooks need open lines) 32 h3 hxg4 33 hxg4 Rh8 34 Re2 Rh1 35 Kc2 Kg5 36 Kb2
Kh4 37 Ka3 Kg3 38 Rb2 Rxe4 39 fxe4 f3 0-1, J.Pryor-C.Lakdawala, San Diego (rapid)
2011.
b) 6 h4 is an attempted refutation of the line, but 6 ... Nf6 7 h5 Rg8! scores well for
Black (I would stay clear of 7 ... Nxh5?! 8 e4!, threatening Rxh5 when White’s position
looks promising).

After 8 hxg6 hxg6 9 e4!? fxe4 10 Qa4 Qb6 11 Bg5 Kd8! 12 Ne2 d6 13 Ng3 Bf5 I prefer
Black’s chances, A.Salem-C.Bauer, Biel 2011.
c) 6 g4!? is a dangerous attempt to refute our line. This game made chess history,
since it was the first ever Dzindzi-Indian game: 6 ... fxg4 7 h3, D.Gurevich-
R.Dzindzichashvili, Berkeley 1984. Here Black’s best scoring move is 7 ... d6 8 hxg4 Bxg4
9 Bh3 Nf6 and now the comp suggests the unplayed 10 Bxg4 Nxg4 11 Nh3 Nd7 12 Qc2
Qc8! 13 f3 Nge5 14 Bh6 Nxc4. White has a huge bind, but Black is up two pawns. I feel
the chances are approximately balanced.
6 ... d6
Preventing d6.
7 Nh3!?
Innovation suggests progress, but as we all understand, a new move doesn’t
necessarily equate with an improved move. His idea is to put heat on e6, with Nf4 next.
It looks no better or worse than White’s main move 7 h4 h6 8 Bf4 Nd7 9 Nf3 Qa5 10 Qc2
Nb6 11 Nd2 Qa4, which also looks fine for Black.
7 ... Nd7 8 Nf4 Qa5 9 Qb3 Ngf6 10 Ne6

Tip: Don’t fear a knight invasion to e6, since you can either swap away the intruder,
or just develop around it. The knight’s placement looks rather intimidating, but
when we look closer, we see that it isn’t really attacking anything at all.

10 ... h6 11 Bf4 Nf8!


Out! The e6 intruder is ejected.
12 Nxf8 Kxf8 13 f3
Preventing ... Ne4 ideas.
13 ... Kg7 14 e3 Bd7
White’s queen is tied down to defence of c3, so b7 isn’t really hanging.
15 Bd3 Rab8
Now ... b5 ideas are in the air.
16 a4
He halts ... b5, but now must watch over his a-pawn.
16 ... Nh5
Depriving White of the bishop-pair.
17 0-0 Rhe8
I’m in no rush to take his bishop, since a recapture opens the e-file for his rooks.
18 g4!?
My IM opponent, an aggressive attacking player, intends to go after my sensitive g6-
square, but the question remains: whose king is going to be the more exposed? 18 Bg3 is
the tamer alternative: 18 ... Nxg3 19 hxg3 e5 20 dxe6 Bxe6 21 e4 Qd8 22 exf5 gxf5 23 f4
Qf6 24 a5. The comp assesses this at even.

18 ... Nxf4 19 exf4 Rf8!?


Avoiding 19 ... fxg4? 20 Qc2! when g6 falls and my king is seriously endangered.
However, 19 ... e6, which follows the principle: counter a wing attack with a central
distraction, looks like Black’s best move. After 20 Qc2 Qd8 21 Kh1 Qf6 22 Rg1 Kh8 I prefer
Black’s superior structure to White’s attacking chances.
20 Rfe1
The wrong rook. I would feed White’s potential attack with 20 Rae1! and forget the
a4-pawn.
20 ... Qd8
Exercise (critical decision): Black threatens to chop White’s g4-pawn. Should
White release the tension with 21 gxf5, or should he offer to sacrifice it by
swinging his inactive a1-rook into the attack with 21 Ra2? One line is equal,
while the other one lands White in a losing position. Where are White’s priorities:
should he play it safe, or should he go for it?

21 Ra2?
It’s only a matter of time between the intense desire for a thing, and the act of
grasping for it. His attack is the hungry sparrow chick with beak open, waiting to be fed.
White overestimates his attacking chances and opts for an unsound sacrifice. Of course,
such tempting ideas are rationalized with the thought: ‘We shouldn’t attack in a safe
antiseptic manner, as if playing a video game. Instead, when we attack, we must commit
to getting our hands a bit bloodied.’
Answer: White maintains equality with 21 gxf5!. When it comes to unpleasant but
reality-based defensive obligations, we are often followers of the out-of-sight, out-of-
mind philosophy. Natural attackers like IM Professor Ginsburg tend to regard defence as
unpleasant menial labour. Here 21 ... Bxf5 22 Bxf5 Rxf5 23 Re6 Rxf4 24 Qc2 Kh8! 25
Qxg6 Qg8 is an even ending.
21 ... fxg4!
Violence is the natural next step when diplomacy fails. Although your writer’s chess
reminds many my readers of an affable bunny, I hasten to point out that I’m ready to
brawl when cornered. I correctly gauged that White’s king is more exposed than Black’s.
22 Rae2 gxf3!
My opponent’s build-up on e7 is met with blank unconcern. Also winning is 22 ... Rxf4!
23 Rxe7+ Kf8 24 Bxg6 gxf3 25 Kf2 Qxe7 26 Rxe7 Kxe7 and Black’s passed f-pawn will win
him the game. I’m shocked that I didn’t chicken out so.
23 Rxe7+ Rf7
Also playable is 23 ... Kg8!, and if 24 Bxg6 f2+! (attraction) 25 Kxf2 Rxf4+ 26 Kg2
Rg4+ when White hangs his loose bishop.
24 Rxf7+ Kxf7 25 Qb1 Qh4!
Threat: ... f2+.
26 Bxg6+
A parasite is itself a kind of prisoner, unable to live life without being dependent upon
the one it feeds upon. White’s bishop isn’t so secure on g6, as we will see in the coming
moves, but if 26 Kh1 f2 27 Rf1 Bh3 28 Bxg6+ Kf6 29 Qc2 Rg8! and White’s game
collapses.
26 ... Kf6 27 Kh1

Exercise (combination alert): Black’s dark-square power keeps his king


surprisingly safe in the vicinity of mid-board, while White’s is in grave danger.
Black has two ways to win. Find one of them.

Answer: Double attack.


27 ... Bh3!
White has no answer to Black’s coming ... Qg4 and ... Qf2 threats.
Answer no.2: Even quicker is 27 ... Qf2! 28 Rg1 Bh3 when there is no reasonable
defence to the coming ... Bg2+ threat.
28 Rf1
Total desperation.
28 ... Qg4! 0-1
29 Rf2 Bg2+ 30 Kg1 Qxg6 wins. We often get away with little deceptions, but rarely
do we pull off a grandly extravagant vulgarity/fabrication of the kind White attempts. Of
course, indeed, not 28 ... Bxf1?? which allows 29 Qf5+ Ke7 (29 ... Kg7 30 Qf7+ Kh8 31
Qh7 mate) 30 Qe6+ Kd8 31 Qxd6+ Kc8 32 Bf5 mate.

Summary
The Dzindzi-Indian is kind of the Modern’s hybrid version of a Sämisch variation of the
Nimzo-Indian and the Leningrad Dutch.

Game 20
B.Baker-C.Lakdawala
San Diego (rapid) 2016

1 c4 g6 2 Nc3 Bg7 3 d4 c5 4 d5 Bxc3+ 5 bxc3 f5


At this point White’s choice is between silence or action. To me, only the harsh action
lines are cause for concern. I love to face the quiet lines.
6 Nf3

Such passionless neutrality is too mild to frighten our side. White makes no attempt to
refute the Dzindzi-Indian, but instead simply develops, and then only later plans to make
use of his increased dark-square power.
6 ... Nf6 7 g3!?
I just love it when opponents meet my Dzindzi without ostentation and with non-
violent passive resistance. It feels like they internally obey the universal mom edict: “If
you don’t have anything nice to say about someone, then don’t say anything at all.”
Fianchettoing introduces an artificial element in White’s play, as if he strains to remain
casual and easy going, while in the midst of a structural emergency.
A fianchetto doesn’t make sense to me, (yet so far it’s the most common white set-up
I face) since White’s c4-pawn is seriously weakened if the bishop leaves the f1-a6
diagonal. I would go for something like 7 Bh6 Qa5 8 Qc2 d6 9 e3 Nbd7 10 Bd3 Ng4 11
Bg5 Nge5 12 Nxe5 Nxe5 13 Be2 Bd7 14 f4 Nf7 15 Bh4 0-0 16 0-0 Rae8 with a distinctly
Nimzo-Indian-like position, where White’s bishop-pair and Black’s superior structure may
balance out.
7 ... d6 8 Bg2
Alternatively, 8 Nd2 Qa5 9 Qb3 Nbd7 10 Bg2 Nb6 11 0-0 Bd7 12 e4 fxe4 13 Nxe4 Nxe4
14 Bxe4 Qa4! (remember the ... Nb6 and ... Qa4! trick, which adds heat to White’s weak
c4-pawn) 15 Rb1 0-0-0 16 Bg5 (after 16 Bd3 White’s coverage of c4 is undermined with
16 ... Bf5) 16 ... Qxc4 17 Rfe1 (White hopes his bishop-pair and pressure on e7 make up
for the lost pawn) 17 ... Rde8 18 Bg2 Bf5 19 Rb2 Qxb3?! (I missed 19 ... h6! when he
can’t take on e7, due to the pin 20 ... Rh7!, and if 20 Bc1 e5 21 dxe6 Rxe6 with an extra
pawn for Black) 20 axb3 e5. White’s bishop-pair does offer some compensation for his
missing pawn, but I still prefer Black, P.Graves-C.Lakdawala, San Diego (rapid) 2009.
8 ... Qa5 9 Qb3 Nbd7

10 Ng5
Just like last game, my opponent rushes for the e6-square. Again, I’m not worried
because I can either chop or chase away the intruder when the time is right. I would have
tried 10 Bh6.

Tip: Always keep in mind the manoeuvre ... Qa5, ... Nbd7 and ... Nb6, picking on
White’s weak c4-pawn. If White plays a4, to stop our ... Qa4, then we play ... Bd7,
which adds a third attacker on the one very weak white a4-pawn.

10 ... Nb6
Threat: ... Qa4, picking off his lightly protected c4-pawn.
11 Ne6 Kf7!?
I could also go for 11 ... Bxe6 12 dxe6 0-0, and if 13 Bxb7?! Rab8 14 Bg2 Qa6. The c4-
pawn falls and I like Black’s structure, despite White’s bishops.
12 0-0
12 Ng5+?! gives Black a superior version of the above variation after 12 ... Kg7 13
Ne6+ Bxe6 14 dxe6 Qa6.
12 ... Qa4 13 Bg5?!
This isn’t forceful enough. White should open lines in order to go after Black’s loosely
protected king with 13 e4!.
13 ... Qxc4 14 Bxf6 Kxf6!

I take the Steinitz endgame principle of activating your king one step further, with
queens still on the board. After 14 ... exf6? 15 Nc7 Rb8 16 Nb5 White double attacks d6
and a7, regaining his lost pawn.
15 e4?!
Lashing out often serves as a poultice to our frustrations. He wants to get at my king
and so logically opens the centre, yet there are two factors against successful
implementation of his intent:
a) Black can remove queens from the board, greatly defusing White’s attacking
potential.
b) Black can play ... Bxe6. When White recaptures with his d-pawn, his own pawn acts
as a human shield which clogs his central line opening plans.
White should try and mix it up with 15 Nc7! Rb8 16 Qa3! (principle: keep queens on
the board when attacking) 16 ... Bd7! 17 e4! (now the break is well timed) 17 ... Rbc8 18
Ne6 Bxe6 19 dxe6 fxe4 20 Rfe1 d5 21 Qc1 Kg7 22 a4. Black’s airy king offers White some
but probably not enough compensation for the missing pawns.
15 ... Bxe6 16 dxe6 Qxb3!
Sure, it’s a concession to straighten out his queenside pawns, but in return I get the
peace of mind of king safety. I think trading is more accurate than 16 ... Qxe6!? 17 exf5
Qxb3 18 axb3 d5 19 fxg6 hxg6 20 c4! when White’s open files for his rooks and powerful
bishop may save the game for his pawn-down side.
17 axb3 fxe4 18 c4!
I think this move which prevents ... d5 is his best practical chance to save the game.
After 18 Bxe4?! d5 19 Bf3 Kxe6 20 c4 Kd6! 21 cxd5 Nxd5 22 Rfd1 e6 23 Rd2 b6 24 Rad1
Rad8 25 Bxd5 exd5 26 Rxd5+ Kc7 27 Kg2 a5 Black should comfortably win the pawn-up
rook and pawn ending, due to his 3:1 queenside super-majority.
18 ... Kxe6
My king’s years of wandering in the wilderness are over, since queens – and therefore
danger – have been removed from the board. Also highly promising is 18 ... a5! 19 Bxe4
a4! 20 bxa4 Rxa4 21 Rxa4 Nxa4 22 Bxb7 Rb8 23 Rb1 Nc3 24 Rb2 Ne2+! 25 Rxe2 (forced,
otherwise Black’s knight gets to d4 and e6 falls) 25 ... Rxb7 26 Re4 Rb4! (threat: ... d5!)
27 f3 Rb3 28 Kf2 Rb2+ 29 Kg1 Rd2! when the coming ... Rd4 gives Black a winning rook
and pawn ending.
Calculation in our mind remains an abstraction until we apply it over the board, where
it assumes a flesh-and-blood transformation. Of course I was unable to work out all (or
even part!) of this line, since with the clock ticking, the human mental slide rule is a joke,
when juxtaposed with a machine’s vast and virtually instantaneous computational ability.
19 Bxe4 a5!
Intuitive/rogue impulses may sometimes be of greater benefit than carefully nurtured
logic. I decided to channel opposing energy into an unravelling pawn
sacrifice/counterattack, rather than to pure defence. We must take care that the remedy
doesn’t inflict more damage than the actual ailment. Here I relied on the endgame
principle: fix your opponent’s pawns on the same colour of your opponent’s remaining
bishop. In this way, he or she remains weak on the opposite colour of the bishop and
pawns, since they will be underrepresented.
This is the human move. I’m two pawns up, so I return one to unravel. Of course, the
super-greedy comp screams bloody murder at my generosity and demands that Black
hangs on to everything. I couldn’t get myself to take the confinement chamber route with
19 ... Rhb8.
20 Bxb7
This is the part where under-fed Oliver Twist meekly walks up to Mr. Bumble with
gruel bowl in trembling hands and asks for “more”. White reduced his pawn deficit to one,
yet faces new troubles, since both his queenside pawns are now fixed on the same (and
therefore unfavourable) colour of his remaining bishop.
20 ... Ra7 21 Bc6 Rc8 22 Rfe1+ Kf6 23 Bb5 Na8!
Oh, the Nimzowitschian bliss! It took the utmost will power not to award myself with a
triple exclam for this knight retreat. It re-routes to c7 to eject the b5 obstacle.
24 Re2
More accurate was 24 Re3.
24 ... Nc7
Exercise (critical decision): White is in deep trouble. Should he play 25 Bd7
and then 26 Bh3 to keep Black’s knight out of e6, or should he play 25 Ba4 to
protect his weak b3-pawn. One line continues to resist, while the other loses.

25 Ba4?!
“God is testing me,” bemoans the harried bishop, who becomes the object of the black
pieces’s contemptuous jests. It is slow death to allow Black to transfer his knight to d4.
Answer: He had to play 25 Bd7! Rd8 26 Bh3 Rb8 27 Re3 e6. To claim that Black has
good winning chances is a fact; to claim that Black is absolutely winning, on the other
hand, is a misleading oversimplification, since conversion still requires great care.
25 ... Ne6 26 f4 Nd4 0-1
It’s understandable that he didn’t want to play this out. The ending is completely
hopeless for White: 27 Rb2 Rb8 28 Kf2 Rb4 29 Ke3 Rab7 30 Rab1 e6 31 Kd3 d5 32 Kc3
R7b6! (zugzwang) 33 cxd5 (33 Kd3 is met with 33 ... dxc4+! when White is unable to
recapture) 33 ... exd5 34 Kd3 (34 Kd2 c4 wins) 34 ... Nxb3! 35 Bxb3 Rxb3+ 36 Rxb3
Rxb3+ 37 Rxb3 c4+ 38 Kc3 cxb3 39 Kxb3 Kf5 with an easily won king and pawn ending.

Summary
In my opinion we should rejoice when White plays any quiet line against our Dzindzi-
Indian, since they allow us to catch up in development and then play on our superior
structure.

Game 21
Q.Loiseau-R.Wojtaszek
French Team Championship 2013

1 d4 g6 2 c4 Bg7 3 Nc3 c5 4 d5 Bxc3+


It would be redundant to mention that after this move White’s and Black’s strategic
interests become largely contradictory to one another.
5 bxc3 f5 6 e4!
When the law makes an example of someone, the idea is to inflict a penalty so harsh
that it forever dissuades others from committing the same crime in the future. White’s
last move is played with this same principle. He reasons: you can’t possibly give me your
powerful dark-squared bishop and expect to survive to a ripe old age, so White attempts
to blow Black out of the water with a refutation attempt of the Dzindzi-Indian in Staunton
Gambit fashion. In my opinion, this move, along with 6 g4!? (which is covered in the
notes to the first game of this chapter) are two of White’s most worrisome options
against our Dzindzi-Indian.
6 ... fxe4
We should grab e4. If 6 ... d6?! 7 exf5 Bxf5 and here the unplayed 8 h3! intending g4
looks quite promising for White.
7 f3!
White doesn’t attempt to regain his lost pawn on e4. Instead he turns it into a true
gambit, in order to open lines and increase development lead.
7 ... d6!

Tip: In my opinion the best way for Black to survive White’s gambit lines is to
first accept the offer and then later return it for a concession.

Warning: I strongly advise against the greedy 7 ... exf3!?. This move may be
technically playable, but the practical chances all lie with White. After 8 Nxf3
he obtains a kind of super-Staunton Gambit, with enhanced dark-square power.

8 fxe4 Nf6

Tip: If you don’t care for the position Black got in this game, then maybe try a line
I have used with success in online blitz games: 8 ... e5!?. The idea is to make the
pawn structure completely rigid to decrease the efficiency of White’s bishop-pair.

For example, 9 Nf3 (9 dxe6? is well met with 9 ... Qh4+ when White’s king is forced to
move) 9 ... Nd7 10 Bd3 Ndf6!? 11 0-0 Kf8 12 Nh4 Kg7 13 Qf3 Qe7 14 Rb1 Rb8 15 Rb2 Bd7
16 Rbf2 Rf8 17 Bg5 h6 18 Bd2 Qe8 when White’s position looks a lot better than it
actually is, since he has no points of attack in Black’s fortress, G.Battaglini-A.Istratescu,
Caleta 2010.
9 Bd3
9 e5 shouldn’t scare us. Black looks good after 9 ... dxe5 10 Be2 Qa5 11 Qb3 Ne4!.
9 ... Nbd7 10 Nf3 Ne5!?

Black seems to compete with his own strategic valour/recklessness.

Tip: Remember this idea, whose intention is to clog the centre to inhibit
White’s bishops. Black’s doubled e5-pawn isn’t really weak.
Warning: Make absolutely certain you keep watch over the e5-square. Avoid lines
like:

a) 10 ... 0-0? 11 Bh6 Re8 12 h3! (this move prevents ... Ng4 and ... Nge5, and Black’s
king looks pretty shaky to me) 12 ... e5 13 Ng5 when White has all the markings of a
winning attack, since Black’s rook is unable to challenge the f-file.
b) Black can also try 10 ... Ng4 11 h3 Qa5 12 0-0 Nge5 13 Qc2 0-0 14 Bh6 Rf7 15 Ng5
Rf6, but I prefer White’s chances after 16 Ne6.
11 Nxe5

Note: The tempting 11 Bf4?! is met with 11 ... Bg4!, which is a promising
pawn sacrifice.

Following 12 Bxe5 dxe5 13 Qa4+ Kf8! 14 Nxe5 Qd6! 15 Nf3 Bxf3 16 gxf3 Nh5! Black’s
dark-square domination and super-knight versus crappy white bishop offer him more than
enough compensation for the missing pawn.
11 ... dxe5 12 0-0
12 Bh6! has never been played, but worries me: 12 ... Qd6 (I analyzed 12 ... Ng4, but
couldn’t make it work to my satisfaction after 13 Qd2!?; White offers his precious dark-
squared bishop to infiltrate h6 with his queen, and I prefer his position) 13 0-0 Bd7 14
Rb1 b6 15 Qd2 with an acceptable but still slightly inferior version of a Nimzo-Indian for
Black’s side.
12 ... Qd6?!
Tip: If possible, don’t allow White’s dark-squared bishop to slip into h6.

12 ... h6! preventing Bh6 is Black’s most accurate move: for example, 13 a4 a5 14 Ra2
Qd6 15 h3 Bd7 16 Be3 Kf7! 17 Qe1 Kg7 18 Kh2 b6 19 g3 Raf8 and Black achieved a
Nimzo-Indian-like equality, A.Braun-K.Bischoff, Altenkirchen 2001.
13 Rb1
13 Bh6! Ng4 14 Qd2! is promising for White.
13 ... b6 14 a4
He wants to disrupt with a5, which Black won’t allow.
14 ... a5
Black’s b6-pawn isn’t much of a weakness, since Black can easily defend it as many
times as White can attack it. Also, White weakened his own a4-pawn, so neither side
really profited from this structural shift.
15 Qb3
Both sides continue to underestimate 15 Bh6!.
15 ... Rb8 16 Rb2 h6!
At last!
17 Be3 Bd7
A reminder to White that he must keep watch over a4. Black equalized.
18 h3 Kf7!

Tip: When we can’t castle, then consider castling by hand. Here g7 is the best
square for Black’s king, so he simply walks over to the square.

19 Kf2!?
Note: In closed positions like this one, we may get away with elaborate,
sometimes even long-winded plans.

White intends the following:


1. To transfer his king to the queenside, possibly even a3, where he resides in safety.
2. Once his king is safe, he can begin to push his g- and h-pawns with impunity, and
attempt to attack Black’s king.
19 ... Kg7 20 Ke1 g5!?
Principle: place your pawn on the opposite colour of your remaining bishop – in this
case every single pawn he owns. Now White must worry about ideas like ... Nh5 and ...
Nf4.
21 g4!?
The idea is to prevent the black knight’s transfer to h5 and then f4, but in doing so he
allows the knight to reach f4 by another route, via h7, f8 and g6.
21 ... Nh7!
Worrying White about a possible transfer to f4, with the aforementioned knight-route.
22 Rh2
Intending h4, but I’m not so sure about his plan, since White may be damaging his
own position more than harming Black’s with this idea.
22 ... Rhf8
Tip: Never prevent your opponent from following an incorrect path.

Black should allow White his intent with 22 ... Nf8!, and if 23 h4!? Ng6! 24 hxg5 hxg5.
Now if White grabs the pawn with 25 Rxh8? Rxh8 26 Bxg5 Nf4! 27 Qd1 Rh3 28 Bc2 Qg6
29 Bxf4 exf4 Black has a winning position. If 30 Rxf4?? Qg5 31 Rf1 Bxg4 when White’s
game collapses.
23 Rg1
He backs up his g-pawn before his intended g4.
23 ... Rf3 24 Kd2 Qf6!
The idea of a future strategic exchange sacrifice strobes in and out of Black’s mind.
25 Rhh1

Tip: Never play a combination without first checking to see if your opponent
can respond with a counter combination.

In this case 25 Bxc5?? is destroyed by 25 ... bxc5! 26 Qxb8 and White’s ‘win’ of
material is short lived after 26 ... Qf4+ which picks off both white rooks and White’s
bishop.
25 ... Nf8!
Intending ... Ng6. Black’s move is actually a sound exchange sacrifice, leading to dark-
square domination.
26 Be2
White gains the exchange, but not an advantage, while avoiding 26 Bxc5?? bxc5 27
Qxb8 Qf4+ 28 Kc2 Rf2+ 29 Kb1 Qd2 30 Qb3 Bxa4! 31 Qa3 Qxd3+ 32 Ka1 Bc2 when he
can resign.
26 ... Rf4!

Did You Know? A positional player refuses to gamble without a strong chance
of return. It was world champion Tigran Petrosian who highlighted the possibilities
of gain through a strategic exchange sacrifice, similar to the one we see here.

Black’s move is more accurate than 26 ... Rxe3!? 27 Kxe3 Qf4+ 28 Kd3 Ng6 29 Qd1
when White holds things together.
27 Bxf4
Not much choice, since after 27 Bd3?! Ng6 28 Qc2 Rf8 Black continues to make
progress without material cost.
27 ... exf4
Does Black obtain compensation for the exchange? Significant points to consider:
1. Dark-square domination.
2. Use of the e5-square for a piece.
3. Lack of useful avenues for White’s dormant rooks.
4. White owns a bad bishop.
5. White’s structure remains inferior, with a weak pawns on e4 and a4.
6. Black’s f-pawn is a tumour for White. Whether or not it’s malignant or benign is for
now open to question.
Conclusion: Black stands no worse, despite his material deficit.
28 e5!
White gained from his sacrifice as well:
1. By returning a pawn his remaining bishop is no longer a bad one.
2. By returning the pawn White opens the e-file for his rooks.
Instead, 28 Bf3? Ng6 29 Rf1 Ne5 is strategically lost for White, who can do nothing but
wait.
28 ... Qxe5 29 Bd3 Qf6 30 Re1 Re8 31 Qc2
If 31 Rb1!? f3! and White is unable to play 32 Qxb6? Qf4+ 33 Kd1 Qe3! 34 Bc2 f2 35
Rf1 h5!, which leaves him busted.
31 ... e5 32 dxe6?!
This move activates Black, far more than it does White. He was better off blockading
with 32 Be4.
32 ... Bxe6 33 Kc1 Rd8 34 Kb2 Bf7 35 Rh2 Qd6 36 Bf5!?
He gives up another pawn rather than tie himself down with 36 Rd2 Ng6 37 Rdd1 Qf6
intending ... Ne5.
36 ... Bxc4 37 h4
With two pawns for the exchange, Black is actually up material now. White’s only
chance is to expose Black’s king in order to play for perpetual check or mate.
37 ... Bf7 38 hxg5 hxg5 39 Qg2!
He wants to sneak into b7.
39 ... Ng6
I prefer 39 ... Qd5! 40 Qxd5 Rxd5 with decent chances to convert in the ending.
40 Qb7?!
He misses a draw with 40 Bxg6! Bxg6 41 Qb7+ Rd7 42 Qa8 (threatening to infiltrate
to h8) 42 ... Rd8 43 Qb7+.
40 ... Qd5 41 Qxd5
Maybe 41 Qxb6!? Qa2+ 42 Kc1 Qa1+ 43 Kc2 Qxa4+ 44 Kb1 Rd1+ 45 Rxd1 Qxd1+ 46
Kb2 Qg1 47 Rh5 and matters are far from clear.
41 ... Rxd5

Exercise (critical decision): Should White retain his bishop with 42 Bc2, or should
he force a swap with 42 Be6. One version offers him decent drawing chances,
while the other achieves the opposite.

42 Be6?
At times like these we realize why innocent people willingly sign confessions while
under intense interrogation. White’s game goes into dramatic decline with the swap of
bishops.
Answer: He may yet hold the draw with 42 Bc2! Ne5 43 Rh7+ Kg8 44 Kc1 when he
stands worse, but still hangs in there.
42 ... Re5! 43 Rxe5 Nxe5 44 Bxf7
After 44 Bc8 Be8 45 Kb3 Bg6 46 Kb2 f3 47 Rh1 Bd3! 48 Rh2 Be4! and either ... f2 or ...
Nd3 and ... f2 are coming.
44 ... Kxf7
White’s rook is incompetent when it comes to defence of the g4-pawn.
45 Kc1
Black wins after 45 Rg2 f3 46 Rg1 Nxg4! 47 Rg3 Ne5 48 Rxg5 f2! since his f-pawn is
tactically covered.
45 ... Nxg4 46 Rg2 f3!

Beautiful handmaidens strew roses in the passed f-pawn’s path to promotion.


47 Rg3
If 47 Rxg4 f2 and both white king and rook are helpless to halt promotion.
47 ... Nh2!
Seizing control over the promotion square.
48 Rxg5 Ke6! 0-1
“Please. Enough of this ignoble bleating. Accept your fate, for you have no other option,”
lectures Black’s king to the frustrated rook. White continues to receive unsettling shocks,
of steadily increasing magnitude. The f5-square remains a locked room for White and
there is no answer to the coming ... f2.

Summary
In my opinion only the violent options are White’s best against our Dzindzi-Indian. Be
super prepared against 6 e4! and 6 g4!?. Implicit in both moves is a harsh criticism on our
decision to chop the c3-knight with our precious dark-squared bishop.
Chapter Nine
The Averbakh Variation
1 d4 g6 2 c4 Bg7 3 e4 d6 4 Nc3

In this chapter our opponent avoids the 3 Nc3 move order, which denies us our
Dzindzi-Indian. So in this one we concentrate on 1 d4 g6 2 c4 Bg7 3 e4 d6 4 Nc3 e5!?,
where we enter the Averbakh line. Black’s last move may feel incorrect, since White can
take on e5 and disrupt our castling. We shouldn’t fear the ending, which in my opinion is
at least even for Black. We also look at other responses, like 5 d5 and 5 Nge2, which lead
to King’s Indian/not King’s Indian positions, which resemble the KID, yet contain odd
shifts. These shifts can be weaponized, since they tend to remove our opponents from
their familiar comfort zones, but not ours.

Game 22
E.Agrest-A.Morozevich
Mallorca Olympiad 2004

1 d4 g6 2 e4 Bg7 3 c4 d6 4 Nc3

Tip: Don’t fear 4 f4, the Four Pawns Attack, which is far less dangerous against
our Modern move order.
We respond with 4 ... Nc6! 5 Nf3 (after 5 Be3?! e5! White already struggles for
equality: for example, 6 d5?? is met with 6 ... exf4 7 Bxf4 Qf6! with a double attack on
the b2-pawn and f4-bishop, and after 8 Bc1 Qh4+ White’s game collapses) 5 ... Bg4 6
Be3 Bxf3 7 gxf3 e5! 8 fxe5 dxe5 9 d5, L.Engel-D.Svetushkin, Thessaloniki 2007. Black
stands well after 9 ... Nd4. Now if 10 Qa4+ Kf8 11 Nd2 c5 when ... Bh6 is in the air and
Black stands no worse due to his monster d4-knight and dark-square domination.
4 ... e5!?
Note: This peculiar move order is our gateway into the Modern Averbakh lines.

Warning: 4 ... c5 5 d5 is not the same as our Dzindzi-Indian, since White already
played e4, which denies us ... f5.

If we play 5 ... Bxc3+!? which is playable (I have tried it in online blitz games with
mixed success), just realize that White obtains a better version than in a normal Dzindzi-
Indian after 6 bxc3, since we are denied our normal Dzindzi ... f5 idea.
5 dxe5
Next game we look at 5 d5, while 5 Nge2 will be seen in Game 25 of the next chapter.
White also has 5 Nf3 when 5 ... exd4 6 Nxd4 Nc6 7 Be3 will be seen in note ‘b’ to Black’s
6th move in that aforementioned Game 25. Instead, 5 Be3?! is inaccurate in my opinion.
With 5 ... exd4! 6 Bxd4 Nf6 7 f3 Nc6 Black gains a tempo on White’s bishop, which he
must preserve. Black can play for a future plan of ... a6, ... Be6, ... Ne5, ... c6 and ... b5.
5 ... dxe5 6 Qxd8+ Kxd8
Tip: Don’t fear loss of castling, since it’s an ending, where our king should be utilized,
rather than hidden.

7 f4
This is considered White’s best try for an advantage:
a) 7 Be3 is too mild to bother Black: 7 ... c6 8 0-0-0+ Nd7 9 Nf3 f6 10 Be2 Bh6! and
Black stood at least equal in the ending, M.Rohland-C.Lakdawala, U.S. Open, Palo Alto
1981.
Tip: Remember this ... Bh6! idea which swaps off White’s good bishop and leaves
him with a remaining bad one.

b) To my mind 7 Bg5+ is even worse than 7 Be3, since Black is given ... f6, a move
we want to play, for free. After 7 ... f6 8 Be3 Be6 9 0-0-0+ Nd7 10 f4 White landed in the
f4 lines a move down: 10 ... Nh6 (threat: ... Ng4) 11 h3 Nf7 12 Nf3 exf4 13 Bxf4 c6 14
Be2 Ke7 15 Nd5+!? cxd5 16 cxd5 Nc5 17 dxe6 Nxe6 18 Be3 Bh6!. There is our hand-
White-the-bad-bishop mechanism again, B.Baker-C.Lakdawala, San Diego (rapid) 2012. I
quote from The Modern Defence: Move by Move: “Principles:
1. When the opponent has the bishop-pair, exchange one of them off.
2. Black’s king will be a lot safer with the removal of White’s best minor piece.
3. White is in danger of being left with a slightly bad remaining bishop.”
7 ... Be6
This move develops the bishop before playing ... Nd7 and covers the f7-square.
8 Nf3 Nc6
8 ... Nd7 is more commonly played. Following 9 Be3 (or 9 Ng5 exf4 10 Bxf4 Bxc3+! 11
bxc3 h6 12 Nxe6+ fxe6; I don’t mind taking on both white bishops with my knights here,
especially since the structure favours our side) 9 ... h6 10 0-0-0 I suggest we go for our
favourite strategic destabilizer with 10 ... exf4 11 Bxf4 Bxc3! (our beloved unbalancing
idea is once again employed) 12 bxc3 Ngf6 13 Bd3 Nc5 14 Bc2+ Nfd7 15 Ne5 Kc8 .As you
may have guessed, I prefer Black’s structure over White’s bishop-pair and development
lead.
9 Kf2
A new move in the position, but no improvement over 9 fxe5 Nxe5 10 Bf4! (this idea
forces a concession from Black, who must slightly fix White’s structure with a swap on f3)
10 ... Nxf3+ 11 gxf3 c6 (also possible is the riskier 11 ... Bxc3+!?) 12 0-0-0+ Ke8 13 h4
Nf6 14 Be5 Nh5 15 Bxg7 Nxg7 when Black stood no worse, A.Vouldis-V.Kotronias, Greece
(rapid) 2002.
9 ... Nf6 10 Be2
10 Ng5 Nd4 11 h3 c6 12 Be3 Nh5 also looks fine for Black.
10 ... h6
Morozevich cuts out Ng5 ideas for good. There is no benefit or harm in 10 ... Ng4+ 11
Kg3.
11 h3
White cuts out ... Ng4+ irritations, just so he doesn’t have to think about them on
every turn.
11 ... Kc8
Don’t mistake a person who bides his time as a sign of his future defeat.

Tip: Black has three plans to remove his king from the centre and connect rooks:
1) Black plays ... c6 and ... Kc7.
2) Black plays ... Nc6 controlling d4, so this means he may connect rooks via a ... Kc8,
... b6 and ... Kb7 sequence.
3) Black can move his ... c6-knight, and then play either ... c6 or ... c5, and then ...
Kc7,
as seen in this game.
12 g3
This way White can recapture on f4 with a pawn, taking on hanging pawns, but
controlling e5.
12 ... Nd7 13 Kg2?!
He should continue developing with 13 Be3, and if Black plays the same plan with 13
... exf4 14 gxf4 Bxc3 15 bxc3, White gets a better version than he got in the game.
13 ... exf4!
Morozevich decides to seriously unbalance the position, handing White hanging pawns
and soon-to-be doubled c-pawns, in exchange for his dark-squared bishop.
14 gxf4

Did You Know? White’s e- and f-pawns represent a structure know as hanging
pawns.

Hanging pawns are a connected duo, with no pawn on either side (i.e. in this case no
white d-pawn and no white g-pawn).
14 ... Bxc3!
To some of us the resolve to disrupt becomes the guiding force of our chess lives. The
... Bxc3! exchange seems to represent the secondary theme of this book, where we
commit ourselves to a risky/irreversible strategic unbalancing action. There is no way to
reconcile the opposing interpretations. One side is correct and the other incorrect in its
assumptions, but which one is which?
In this instance I feel like Black got by far the better of the deal, since White’s bishop-
pair and enhanced dark-square control doesn’t feel equally balanced by his devastatingly
heavy structural damage. From this point on, White’s life is one of an unending series of
structural emergencies.
15 bxc3 Na5
This is the forerunner of a future extortion attempt. Moro eyes White’s c4-pawn with a
distinctly proprietary attitude, which forces his opponent to contort to the pawn’s defence.
16 Nd2 c5!
Principle: make the structure as rigid as possible when your opponent owns the
bishop-pair. Now c4 is transformed into an eternal target for Black.
17 Ba3 Kc7
This is the secondary benefit of ... c5!. His king moves to the second rank and Black
connects rooks.
18 Rae1 b6 19 Rhf1
Threat: f5, winning a piece.
19 ... f5
To be considered is 19 ... f6!? 20 f5 gxf5 21 exf5 Bf7 22 Bc1 Rae8 when I prefer
Black’s position.
20 Bd3
The comp likes 20 e5.
20 ... Rhe8 21 Bc1
Now that his a1-rook is centralized, White attempts to reactivate the a3-bishop.
21 ... Rad8 22 Rf3 Nf6!
Moro demands resolution of the central tension.
23 Kf2?!
With the bishop-pair, White doesn’t want to close the game, but should still do so with
23 e5 Nh5 when Black will play to undermine with ... g5 ideas.
23 ... fxe4!
Moro picks the most favourable moment to open the game.
24 Nxe4
Exercise (combination alert): Black has no less than three promising
combinational ideas, all which touch upon the core of White’s difficulty:
defence of his c4-pawn. You need to find only one of them.

Answer: 24 ... Bxc4!


Overloaded defender. Our ‘requests’ tend to sway when they are backed up with the
implicit threat of violence.
Answer no.2: Also winning is 24 ... Bf5! 25 Nxf6 (25 Rfe3? is met with the deflection
shot 25 ... Rxd3! 26 Nxf6 Rexe3 27 Rxe3 Rd1 28 Re1 Rxe1 29 Kxe1 Bxh3 with a hopeless
ending for White, since c4 also falls) 25 ... Rxe1 26 Kxe1 Rxd3 27 Rxd3 Bxd3 28 Ng4 Bxc4
29 a3 Be6 with an extra pawn plus the superior structure. I don’t think the presence of
opposite-coloured bishops will save White here.
Answer no.3: Just as promising is 24 ... Rxd3! (attraction/pin) 25 Rxd3 Bf5! 26 Nxf6
Rxe1 27 Kxe1 Bxd3 28 Ng4 Bxc4 29 a3 Be6, reaching exactly the same position as answer
no.2.
25 Nxf6 Rxe1 26 Kxe1 Bxd3
In the aftermath White may still be alive, but it’s no more than a husk of existence.
Not only is he a pawn down, but every one of his remaining pawns is isolated and
therefore a potential target.
27 f5!
Principle: the material down side should try and swap off as many pawns as possible
to maximize their drawing chances.
27 ... Bxf5 28 Bxh6 Rh8 29 Bf4+ Kc6 30 Re3!?
He gives up another pawn, rather than go passive with 30 Ng4.
30 ... Rh4!?
Moro wants to play ... Ra4 and ... Rxa2, creating a 3-1 queenside pawn majority.
There is nothing wrong with the simple 30 ... Rxh3.
31 Bg3 Ra4 32 Re7 Nc4 33 Rc7+ Kb5
Black’s king is perfectly safe along the queenside light squares.
34 Rh7 Rxa2 35 Nd5 a5
Principle: passed pawns should be pushed.
36 h4 Be4 37 Nc7+ Ka4 38 Re7 Bf3 39 Re6
Threat: Rxg6.
39 ... Kb3!
An opponent who ignores our threat is more insulting than one who meets it with an
angry demonstration. Black’s move is stronger than the meek 39 ... Bh5.
40 Rxg6
Taking a pawn like this is the chess equivalent of killing, cooking and eating a rat for
your dinner, during a time of famine. 40 Bf4 prevents mate, but even here White’s
defensive resources lie stark.

Exercise (combination alert): Having visualized the ends, now comes the difficult
part: how to formulate the means? Black to play and force mate in four moves.

Answer: 40 ... Re2+! 0-1


41 Kf1 Ne3+ 42 Kg1 Rg2+ 43 Kh1 Rxg3+ 44 Kh2 Nf1 is mate. White’s pursued king
finally slumps forward in the eternal lassitude of non-existence.
Summary
Don’t fear the ending we reach after 6 ... Kxd8. Our king may be more of an asset than a
liability in the centre, and we control d4, which in my opinion, offers us dynamic equality.

Game 23
W.So-D.Norwood
Bunratty 2015

1 c4 g6

Did You Know? English GM David Norwood always opens with 1 ... g6 as Black.
He likes the Modern so much, that he opens with 1 g3 as White, reaching a Modern
a move up. His Internet Chess Club handle is the rather misleading ‘Davenogood’.

2 d4 Bg7 3 e4 d6 4 Nc3 e5
Also possible is 4 ... Nc6!? (the Modern can be played many different ways) 5 Be3 (5
d5 is considered White’s most critical try) 5 ... e5 6 d5 Nce7 7 g4 f5!? (just as in the So-
Norwood game, my philosophy is ... f5 or bust) 8 gxf5 gxf5 9 Qh5+ Ng6 (this doesn’t lose
a piece due to my following move) 10 exf5 Qh4 11 Qf3 N6e7 with messy complications,
J.Benjamin-C.Lakdawala, San Mateo (rapid) 1989.
5 d5

In this version White isn’t interested in maintaining the central pawn tension, so he
seizes space and closes the centre.
Tip: Many of our opponents may falsely believe they will soon reach lines of the
King’s Indian proper after 5 ... Nf6. GM Norwood’s next move is the Modern
player’s risky/potentially profitable method of evading King’s Indian and
remaining within Modern territory.

5 ... f5!?
In this way we avoid the King’s Indian and remain in Modern territory:
a) 5 ... Nd7 is a safer (but not better) version: 6 Qc2 (I’m a little nervous about the
line 6 Bd3! Ne7 7 h4! when I prefer White’s game) 6 ... Ne7 7 g3 0-0 8 Bg2 f5,
L.Sussman-C.Lakdawala, San Diego (rapid) 2012. Black achieved a King’s Indian position
at least one move up. Why? In the normal King’s Indian Black plays ... Nf6, moves the
knight again, then plays ... f5 and then often moves the knight right back to f6. My
version in the Sussman game represents a position at least one tempo up for Black.
b) If you don’t like Black’s position in this line, then there is nothing wrong with
transposing to a normal King’s Indian with 5 ... Nf6.
6 exf5!
This is White’s only challenging line. I feel that the commonly played 6 Bd3?! is
inaccurate since we get our one to two move up King’s Indian after 6 ... Nf6.
6 ... gxf5!

This is the part of the movie where the music gets louder and more scary, as the hero
faces grave danger. Be brave, reader, and realize: agreeing to play the Modern – the
standard-bearer for risky openings – is a high-risk courtship, akin to proposing marriage
to Lorena Bobbit. This position is kind of a not-King’s-Indian/King’s Indian, and really
lacks a basis for a true comparison, since I don’t know of any King’s Indian line where
Black is plunged into full crisis/opportunity mode before move 10!

Tip: In the Modern we had better get used to moving our king about in the opening.
We saw it last game, and we see it here. I like Norwood’s brave choice over the
strategically suspect 6 ... Bxf5?! 7 Nge2 Nf6 8 Ng3 when White soon seizes
control over the crucial e4-square.

7 Qh5+ Kf8
Your writer, the Thomas Alva Edison of crazy opening ideas, may one day try the
bizarre but possibly playable provocation 7 ... Ke7!? 8 Bd3 Nf6 9 Qg5 and now Black can
avoid playing the weakening ... e4 with 9 ... Rg8! which indirectly protects f5, since 10
Bxf5?? hangs material to 10 ... Bh8!.
8 Qd1
Sometimes when we are not sure of a position’s pattern, we look to the absence of a
pattern for clues. This is Stockfish’s top choice. White’s queen, having inflicted damage
upon the black king’s right to castle, returns home before it is even hit, since the tempo-
gaining ... Nf6 is coming anyway.
8 Bd3 is White’s main line: 8 ... Nf6, and if 9 Qh4 (after 9 Qg5 Komodo suggests the
unplayed 9 ... h6 10 Qg6 e4 11 Be2 Qe8 12 Qg3 Kg8! – the idea is to slide the king to h7,
and then play ... Rg8 to bear down on the open g-file – 13 Nh3 Kh7, intending ... Rg8
next, with a sharp position) 9 ... Na6 10 Nge2 Qe7 11 0-0, as in S.Wehmeier-I.Sutton,
Wisla 1992, Black can try 11 ... f4! when ... Nxd5 really is a threat (the immediate 11 ...
Nxd5?? is met with the crushing 12 Bg5! Bf6 13 Bh6+ Bg7 14 Bxg7+ Kxg7 15 Qg3+ Kf8
16 Nxd5 and White wins). After 12 f3 Nc5 13 Bc2 Nxd5 14 Qf2 Komodo claims White
obtains compensation for the pawn, but I have trouble seeing it and prefer Black.
8 ... Qe8
The queen gets ready to slide over to g6, where it covers f5 and applies pressure
down the g-file.
9 Nf3 Qg6 10 Be2 Ne7
The g2-pawn is poisoned: 10 ... Qxg2? 11 Rg1 Qh3 12 Rg3 Qh5 13 Nxe5 when White
regains the pawn with a strategically won game since he soon places a knight on f4.
11 Rg1!?
Super-GM So is understandably afraid to castle with Black’s control over the g-file.
11 ... Na6 12 h3!?
The distinction between reality and speculation grows thin. This move implies an
equivalence in kingside attacking force, which just isn’t the case. Perhaps the idea is a
future g4, which in this game he never gets around to. Safer and probably better is 12 g3,
intending to develop normally with Be3.
12 ... e4!
A move like this must be considered very carefully. In this instance I think it’s a good
deal for Black, since he acquires at least as much as he concedes:
1. Black gains central space.
2. Black enhances his dark-squared bishop.
3. Black clears the e5-square for either a bishop or a knight.
4. He allows White’s pieces access to both d4 and f4-square, which is a dangerous
strategic concession.
13 Nd4
White can also consider 13 Nh4!? Qf6 14 g3, intending Ng2 and Nf4.
13 ... Be5
Threat: ... Bh2, which wins the exchange.
14 Nb3!?
Wow! White ignores Black’s ... Bh2 and actually invites it as a strategic exchange
sacrifice.
14 ... h5!
I think this is very wise. White obtains monster play along the dark squares if Black
gets greedy with 14 ... Bh2!? 15 Kf1 Bxg1 16 Kxg1 Rg8 17 Bf1 f4!? 18 Bxf4 Bxh3 19 g3
Bxf1 20 Qxf1 when I wouldn’t mind taking on the white pieces.
15 Kf1 Bd7
15 ... h4! was a good idea, to inhibit White’s g3 plans.
16 f4?!
A rare strategic misjudgement from So. The game remains tensely even after 16 h4.
16 ... Bxc3!
Hooray! GM Norwood makes our favourite move, correctly gauging that Black’s
strategic plusses involved with the damage of White’s queenside pawns, slightly outweigh
White’s enhanced dark-square control.
17 bxc3 b6?!
He locks down the wrong square. Black should prevent White’s next move with 17 ...
h4! 18 Be3 b6 19 Bf2 Nc5 20 Qd4 Kf7. White’s bishops have nothing to do and Black is the
only one who can make progress on the kingside.
18 g3!
Now chances are back to even.
18 ... Kf7 19 Be3 Rag8
Black prepares an eventual ... h4 break.
20 a4
So begins to eye the queenside with covert ambition.
20 ... Nc5 21 Nd4
21 Nxc5!? bxc5 denies White use of the d4-square.
21 ... a6?!
When we make a modification, then be certain it enhances, rather than decreases our
power. This move allows White unnecessary play on the queenside, when he should shut
it down with 21 ... a5!.
22 a5!
White gains a touch of queenside counterplay with this line opening.
22 ... bxa5 23 Nb3?!
Correct was 23 Rxa5, and if 23 ... h4 24 g4 fxg4 25 Bxg4 Bxg4 26 Rxg4 Qf6 with a
complete mess.
23 ... Nxb3 24 Qxb3 h4!
The h-pawn is the bearing upon which Black’s attacking machinery runs. Now the air
around White’s king solidifies with malicious intent.
25 g4?
25 gxh4 Qf6 26 Rg5 Rxh4 27 Bh5+ Ng6 is heavily in Black’s favour, but this is still
better than what he obtained in the game’s continuation.
25 ... fxg4 26 Bxg4

Exercise (critical decision): Black’s choices are 26 ... Bxg4 and 26 ... Nf5.
Both favour Black, but only one line leads to a forced win. Which one would you play?

26 ... Nf5
Black still holds a big advantage after his move, but immediately decisive is:
Answer: 26 ... Bxg4! 27 hxg4 (or 27 Rxg4 Qh7 28 Qd1 Rxg4 29 hxg4 h3 30 Kg1 Nf5!!
when taking the knight allows a queen check along the g-file, mating, and 31 Qe2 h2+
wins) 27 ... h3! 28 g5 h2 29 Rh1 Qh5 30 Qd1 Qh3+ 31 Ke2 Qg2+ 32 Bf2 Rb8! is decisive.
27 Kf2 Rb8?!
27 ... Qf6! is still highly favourable for Black.
28 Qc2! a4 29 Qxe4!
The most precious gift we can receive in a chess game is a second chance. The comp
hates this move, while I think it’s worth an exclam. So remains contemptuously calm in
the face of a crisis. Here he forces his will upon an unruly gang of probabilities, hoping to
force order upon them. Opening the e-file looks like a terrible mistake, but I was unable
to find a win for Black from this point on, since Black has nothing better than to enter the
favourable (but still not winning!) ending Norwood chose.
29 ... Rb2+ 30 Kf3 Re8 31 Bxf5 Qxf5 32 Qxf5+ Bxf5
In the aftermath White struggles in the ending, since his pawns are weak and his king
insecure.
33 Bd4 Be4+! 34 Kg4!
The only move, since 34 Ke3?? leads to 34 ... Bxd5+ 35 Kd3 Be4+ 36 Ke3 Bg2+ 37
Kd3 c5! winning a piece, as White is unable to play 38 Be3?? because it leads to a
helpmate after 38 ... Be4 mate.
34 ... Rg8+ 35 Kxh4 Rxg1
Black relies on the power of his passed a-pawns in the ending.
36 Bxg1 Rb1

Black will eventually be up two pawns, but still can’t win since normally pure opposite-
coloured bishops endings are drawn. This position is no exception. The coming dozen or
so moves resolves many uncertainties about its won or drawn nature.
37 Rxb1 Bxb1
Black threatens ... a3, ... a2 and a1Q, so White’s bishop must force its way on to the
a1-h8 diagonal, and also move his obstructing c3-pawn. Norwood’s vision may be clear,
yet as we soon see, he faces crushing organizational and logistical challenges in the
implementation/conversion stage.
38 c5!
The only move, and not 38 Bd4?? a3 39 c5 dxc5 40 Be5 c4! when Black’s forward a-
pawn will promote.
38 ... a3 39 c4 a2 40 Bd4
Just in time. White’s defence is one of sturdy serviceability, rather than elegant
efficiency.
40 ... dxc5 41 Be5 Bd3
41 ... Kg6! keeping White’s king out is a better try, yet White should still hold the
game. I took Black and Komodo held the draw in the following fashion: 42 Kg3 Bf5! 43 h4
Bd3! 44 Kf3 Kf5 45 h5 Bxc4 46 Ke3 Bxd5 47 Kd2 Bf7 48 h6 Kg6 49 Bg7 Be6 50 Kc2 c4 51
Kb2 c3+! 52 Bxc3 Kxh6 53 Be5 c6 54 f5 Bd5 (54 ... Bxf5 55 Kxa2 is also completely
drawn) 55 f6 Kg6 56 Ka1 Kf5 57 Bb2 Ke4 58 Bc3 Bf7 59 Ba5 Kd3 60 Kb2 c5 61 Be1 c4 62
Kxa2! c3+ 63 Kb1 a5 64 Bxc3! Kxc3. The ending is drawn.

Did You Know? A bishop and lone rook’s pawn is unable to win unless the bishop is
on the same colour as the remaining pawn’s promotion square. We notice here this
is unfortunately not the case for Black.

42 Kg5 Bxc4 43 f5 Bxd5


The bishop picks up yet another pawn, continuing his eat-drink-and-be-merry lifestyle.
44 h4 Kg8 45 Kg6 Bf7+ 46 Kg5 ½-½

When in college I got a summer job at the State Fair. I was assigned to a ring toss game,
with pay based on commission. So few fair goers came to the ring toss that after 12 hours
work I earned a grand total of $3.25. I learned: there is nothing more depressing than
working hard for little reward. Now GM Norwood knows exactly how this feels, since Black
is up two pawns but is unable to win, because, infuriatingly, White’s lone bishop is fully
capable of halting all four of Black’s passed pawns.
Summary
5 ... f5!? avoids King’s Indian byways and leads the game into strange Modern waters. If
you don’t feel comfortable with the position Black got in this game, then remember that
you still have the option of 5 ... Nf6, which bails out into a normal King’s Indian.
Chapter Ten
Other d4 and c4 Lines
1 d4 g6 2 c4 Bg7 3 e4 d6 4 Nf3

We conclude our queen’s pawn coverage with a variety of set-ups, but mainly with
White playing d4, c4 and e4. In this chapter we can play for the traditional ... e5 push,
but I also add a few ... c5 options, which resemble the Benoni and even Benko Gambit.

Game 24
P.Graves-C.Lakdawala
San Diego (rapid) 2010

1 Nf3 g6 2 c4 Bg7 3 d4 d6 4 e4
Because of White’s 1 Nf3 move order, we have a way to begin pressuring his d4 point
with our next move.
Instead, 4 Nc3 can be met with 4 ... c5 and then:
a) 5 d5 Bxc3+ 6 bxc3 f5 transposes to the non-threatening version of the Dzindzi-
Indian, seen in the Baker-Lakdawala game from the Dzindzi chapter.
b) 5 e4 Bg4!? we look at this later in this chapter.
c) 5 e3 is too mild to give Black concern: 5 ... Nc6 6 Be2 Nf6 7 0-0 0-0 8 d5 Na5 9 Nd2
e6 10 e4, J.Pryor-C.Lakdawala, San Diego (rapid) 2012. We reach a kind of hybrid Benoni
position where Black’s a5-knight may be offside, but Black also gains from the fact that
White is in essence down a tempo, since he took two moves to play e4.
4 ... Bg4

Tip: We fight for the d4-square, so go after one of the square’s key defenders
with ... Bg4, and then add further pressure with ... Nc6.

Otherwise:
a) 4 ... Nf6 5 Nc3 transposes to King’s Indian.
b) Also good is 4 ... e5 and if White goes into the ending he doesn’t have f4 like in the
normal positions where his knight is on c3: 5 Be2 Nc6 6 d5 Nce7 7 Nc3 h6!? (the idea is to
play ... f5 without fear of Ng5) 8 Nd2 f5 9 exf5 gxf5 10 Bh5+ Kf8 (remember, moving our
king is part of the deal in many of these Modern lines; in this case White will either waste
time moving his bishop back, or he will allow Black the bishop-pair with ... Nf6 and ...
Nxh5, as in this note) 11 f4 e4 12 Nf1 Nf6 13 Ng3 c5 14 0-0 Nxh5 15 Nxh5 Bxc3! (there it
is again) 16 bxc3 and I prefer Black’s position, J.Hultin-T.Hillarp Persson, Gothenburg
2015.
5 Be2 Nc6
Threat: ... Bxf3 and ... Nxd4.
6 d5
6 Be3 e5 7 d5 Bxf3 8 Bxf3 Nd4 9 Nc3 c5, as in C.Milton-C.Lakdawala, San Diego
(rapid) 2008, is similar to the game continuation. Black’s monster knight easily makes up
for White’s enhanced light-square control.
6 ... Bxf3!

Tip: In Modern lines where Black plays an early ... Nc6 without ... e5, respond to
White’s d5 with ... Bxf3 and ... Nd4, and then back it up with ... c5. Our powerful
d4-knight compensates our loss of the bishop-pair.

7 Bxf3 Nd4 8 0-0


8 Nc3 c5 9 dxc6 to my mind helps Black, who now gains an open b-file: 9 ... bxc6 10
0-0 Rb8 11 g3 e6 (11 ... c5 12 Qa4+ Qd7 13 Bd1 Nf6 also looks fine for Black) 12 Bg2 Ne7
13 Bh6!? Bxh6 14 Qxd4 0-0 15 Qxa7 Rxb2 when Black ended up with both the superior
structure and superior remaining bishop, D.Janosevic-B.Ivkov, Majdanpek 1976.
8 ... c5 9 Nc3 e5!?
I prefer to triple down on the reinforcement of the d4-knight, rather than cede White
space with 9 ... Nf6 10 Be3 Nd7 11 Bg4.
10 dxe6!?
He wants to open the game due to my development lack. On 10 Qa4+ I intended 10
... Kf8!?, intending to castle by hand with ... Bh6 and ... Kg7.
10 ... fxe6 11 Qa4+ Kf7!?
I felt this was more accurate than to allow White to go for a line like 11 ... Qd7 12
Nb5! (threat: Nc7+) 12 ... Kf7 13 Qd1 Ne7 14 Be3 Nec6 15 e5!? Bxe5 16 Bxc6 Nxc6 17
Bxc5 Rhd8 when I slightly prefer White’s position.
12 Bg4 Nf6 13 Bh3
His idea may be g3 and f4 in the future.
13 ... Nd7
The long-winded idea is to get the knight to c6.
14 Qd1
During the game I was concerned about 14 f4, but Black looks fine after 14 ... Rf8,
intending ... Kg8 next. Now 15 f5?! weakens the e5-square and can be ignored with 15 ...
Kg8!, and if 16 fxe6?? Rxf1+ 17 Kxf1 Qf6+ 18 Kg1 Rf8 when White is unable to resolve
his weak back rank issues.
14 ... Re8
Reinforcing my weakest point, while planning to castle by hand with ... Kg8 next.
15 Be3
After 15 f4 Kg8 16 Be3 I planned 16 ... Nb8! intending ... Nbc6, when Black’s grip on
the d4-square remains unchallenged.
15 ... Ne5
The idea is to play this knight to c6 to retain eternal control over d4.
16 b3 Kg8 17 Rc1 Nec6 18 f3!?
I felt this safe move also veered to passivity. The game remains balanced after 18 f4
a6, preparing ... Rb8 and ... b5.
18 ... a6!
I plan to chip away at his central space with ... b5.
19 Rf2 b5
Black’s grip on d4, coupled with White’s passivity, gives him the superior position.
20 Ne2 Nxe2+
Perhaps Black should expand his dark-square empire with 20 ... b4!? intending ... a5
and ... a4.
21 Rxe2 Bd4!
Principles:
1. When your opponent owns the bishop-pair, swap one of them off.
2. If possible, stick your opponent with a remaining bad bishop on the same colour of
his pawns.
Black’s remaining knight sitting on d4 will clearly exert its superiority.
22 Kh1 Bxe3 23 Rxe3 Qg5?!
This move may be inaccurate:
a) 23 ... e5?? intending to reinforce d4 is a bit too positional, since it forgets about 24
Qd5+, picking up the loose knight on c6.
b) I should lock down the queenside with 23 ... b4! 24 f4! Nd4 25 e5 a5 26 exd6 Qxd6
27 g3 a4 with advantage for Black, who applies more pressure on b3 than White does on
e6.
24 Rd3 Nd4 25 g3?!
White is better off destabilizing the queenside with 25 b4!.
25 ... e5
White’s position exudes the cheerlessness of a granite monument of a dead leader.
He deals with huge problems:
1. Black’s monster knight towers over White’s undernourished bishop.
2. In time Black will play ... b4, ... a5 and ... a4, with a strategically won game. So
passivity is not an option. White must destabilize the d4 point with either 26 f4, or 26 b4.
He is in trouble after both moves. Which one offers him the best odds of survival?
26 f4?
Rationality tends to be emotion’s victim. When we are in trouble, the lies we tell
ourselves are so much easier to believe than the harsh truth. My opponent and friend,
National Master Peter Graves, is rightfully afraid that if he sits around and does nothing,
then Black will achieve a winning bind on the queenside with ... b5, ... a5 and ... a4. His
move, although well intentioned, may be too frantic an attempt at counterplay, since he
doesn’t get enough for his coming exchange sacrifice.
White’s best chance to survive is 26 b4! which is a superior way to undermine Black’s
grip on d4. Both 26 b4! and 26 f4?! are limbs from the same tree, yet only one is correct.
After the former, 26 ... Qh6 27 Bg4 Qf8 28 a4! bxa4 29 Qxa4 Rab8! 30 bxc5 dxc5 31 Qxa6
Rb2 and now ... Qh6 is in the air. Black has enormous compensation for the missing
pawn, but this is still a better chance for White to save the game, when compared to the
line he played.
26 ... exf4 27 Rxd4!?
This is the logical extension of White’s idea. He destroys Black’s best piece and hopes
his remaining bishop will give Black’s king some trouble in the now open position. 27 Bg2
Re5 28 gxf4 Qxf4 isn’t going to save White.
27 ... cxd4 28 Qxd4 fxg3 29 Rd1 Qh5
Keeping watch over d5, attacking h3 and indirectly pressuring the mating square on
h2.
30 Kg2

Exercise (planning): While it’s true that Black’s position is a model of


economic achievement, this isn’t a time to sit back and savour the moment.
It’s a time for action. Should Black go for a consolidating swap with 30 ... Qe5, or
should he go for the kill with 30 ... Qe2+? One plan is clearly superior to the other.
30 ... Qe5?
If you just robbed a bank, then don’t call for an Uber as your getaway vehicle. The sad
truth is that your writer can at times (in fact, most of the time!) be an attacking illiterate.
My instinct is to swap, even when the position screams for me to go for the kill with:
Answer: 30 ... Qe2+! 31 Kxg3 Rxe4!, and now if 32 Qd5+ Kg7 33 Qb7+ Re7! 34
Qxa8 Re3+ 35 Kf4 Qf2+ 36 Kg4 h5+ 37 Kg5 Qf6 mate. “Burn him. I will use my brother’s
ashes to fertilize my roses,” orders the queen.
31 Qxd6 Qxe4+ 32 Kxg3 Qe5+?!
After moody reflection I incorrectly decide to swallow pride and back down by forcing
a won technical ending. I seem to be missing the ‘fight’ portion of the fight-or-flight
survival mechanism. For me, it’s pretty much flight, 98% of the time. Here is further proof
that your writer is utterly devoid of attacking skills, especially when low on the clock.
Once again I should stop worrying about my own king and go after his with 32 ... Qe3+!
33 Kg2 Re5!. White has no checks and ... Rg5+ is coming, with a mating attack. 34 Rd4 is
met with 34 ... Rf8 when there is too much firepower aimed at White’s king.
33 Qxe5 Rxe5 34 Bg2 Ra7
Covering the second rank, as well as the a6-pawn.
35 Rd8+ Kg7 36 Bd5 Re2 37 c5
This is a desperate attempt to gain traction with his passed c-pawn. If 37 a4 bxa4 38
bxa4 Ra2 39 Bc6 Kf6 and Black’s win is a matter of time.
37 ... Rxa2 38 c6

It feels as if White stoically awaits death to take him out of his misery, but in reality
he plots one last desperate cheapo in the time pressure phase.
38 ... Kf6!
38 ... Rc2?? is the cheapo White was playing for: 39 Rd7+ Rxd7 40 cxd7 when both
black rook and king are helpless to halt promotion.
39 Rb8 Re2 40 Rb7 Re7 41 Kf4 g5+
Seizing control over e5 for my king, who assists in blockading White’s passed pawn.
42 Kg4 Ke5 43 Bf3 Kd6
My g5-pawn isn’t important.
44 Kxg5 Raxb7 45 cxb7 Kc7 0-1
Black wins easily due to his queenside majority which soon manifests as a passed
pawn.

Summary
When White commits to an early Nf3 and then follows up with d4, c4 and e4, we can play
for control over the d4-square with the plan ... Bg4 and ... Nc6, and if d5, we play ... Bxf3,
... Nd4 and ... c5. Our domination of d4 makes up for White’s bishop-pair.

Game 25
C.Lakdawala-B.Xia
San Diego (rapid) 2017

1 d4 g6 2 e4 Bg7 3 c4 d6 4 Nc3 e5 5 Nge2

Every player I know has within his or her repertoire, a secret line or two which they
hate to enter, much like being assigned an unpleasant duty by our boss. I dislike this set-
up when I’m Black in the Modern, but love it when I’m White, playing against the Modern.
I think this is one of White’s more dangerous versions of the queen’s pawn Modern.
Note: The idea is to maintain pawn tension in the centre for a while, and then switch
to a King’s Indian Sämisch set-up with f3, later on.

5 ... Nc6 6 Be3

Tip: In such structures it’s usually in White’s favour to retain central tension, since
in doing so, Black’s ... f5 ideas become far more risky than in cases where White
closes the centre.

6 ... h5!?
The way we study openings today is more ritual, than actual thinking. So for me at
least, it’s refreshing to see a really weird opening move. I was prepared for virtually every
theoretical contingency, except this one. To my surprise, I discovered that this rare move
scores 75% for Black in my database, but only based on four games, so it’s not a reliable
sample. My 13-year old student Brandon, who just needed a draw in this game to go over
the master’s rating, does his best to confuse by imitating Duncan Suttles and takes the
mad scientist route. The idea may be to play a future ... Bh6 to swap away White’s good
bishop.
Black has more conventional alternatives in:
a) 6 ... Nh6 (this is normally the line I play as Black) 7 f3 f5 8 Qd2 Nf7 9 d5 Ne7 10 0-
0-0 0-0 11 Kb1 c5!.

I have come to the conclusion this move is Black’s best, since it greatly slows White
on the queenside if White refuses to capture en passant on c6 (and next time I will avoid
11 ... f4 12 Bf2 g5 13 g3 Ng6 with a pleasant King’s Indian Sämisch-like edge for White,
N.Lita-C.Lakdawala, San Diego (rapid) 2017). If 12 dxc6 bxc6 13 c5!? Black offers a pawn
with 13 ... d5!. This particular variation can be an inhospitable tactical wasteland for
those untrained in its ideas: for example, 14 exd5 cxd5 15 Nxd5 f4! 16 Bf2 Bf5+ 17 Kc1!
(17 Ka1? is met with 17 ... Nxd5 18 Qxd5 e4! 19 fxe4 – his knight can’t capture f4, due to
19 ... Qb8!, hitting b2 and f4 simultaneously – 19 ... Bxe4 20 Qxd8 Raxd8 21 Nd4 f3! and
Black’s massive development lead offers tremendous compensation for his missing pawn)
17 ... Nxd5 18 Qxd5 Qa5 19 Qc4 Rad8 20 Rxd8 Rxd8 21 Nc3 e4! 22 fxe4 Bxc3 23 Qxc3
Qxa2 24 Qa3! Qxa3 25 bxa3 Bxe4 with full compensation for the missing pawn.
b) 6 ... exd4 7 Nxd4 Nge7 8 h4! (White exploits the fact that Black can no longer play
... Nf6, with a push of his h-pawn, which makes castling kingside much riskier for Black) 8
... f5!? (the top players seem to favour this aggressive thrust over the meeker but more
solid 8 ... h5) 9 h5 fxe4 10 hxg6 hxg6 11 Rxh8+ Bxh8 12 Nxe4 was V.Kramnik-
S.Mamedyarov, Moscow 2007. Black still looks okay after 12 ... Qd7! intending to
complete development with ... b6, ... Bb7 and ... 0-0-0.
c) After 6 ... f5!? 7 exf5 gxf5!? 8 dxe5 dxe5 9 Qxd8+ Nxd8 the position looks playable
for Black. I still prefer White, since he still leads in development and Black’s hanging
pawns may be more of a future target than a strength in the ending.
7 f3!

Turning points in history often have their origins in either a trivial shift of the normal,
or a happy accident, as in this case. I was totally unaware that my move was a
theoretical novelty. It seems to be an improvement over the other tries, since the idea is
to meet ... Bh6 ideas with Bf2!, preserving White’s good bishop.
7 ... Nh6!?
I thought he would play 7 ... exd4 8 Nxd4 Nge7 9 Be2 0-0 10 0-0. It feels like his ... h5
is played for no reason and represents potential weakening of Black’s king in the future.
8 Qd2 f6!?
I expected 8 ... f5 9 Bg5 Qd7 and I wasn’t sure how to proceed. The comp suggests 10
Nd5! with a clear advantage for White, who pressures Black on f6, c7 and the centre in
general.
9 0-0-0 Kf7?
What? Now his king and h6-knight give off the strange picture of someone who comes
home drunk and forgets to undress before showering. I thought he would play 9 ... Nf7 10
Kb1 0-0 11 d5 Ne7 12 h3 intending g4, with advantage for White.
10 Kb1 a6 11 Nd5
Black will have to contort to get rid of this guy. I also considered 11 h3, intending to
go after his king with f4 next.
11 ... Bd7 12 dxe5!
Well timed, since now he must recapture with his knight, furthering White’s central
control.
12 ... Nxe5
Essential:
a) 12 ... dxe5?? is crushed by the simple discovered attack 13 Nxc7.
b) 12 ... fxe5? 13 Bg5 Qb8 and now White can favourably destabilize Black’s centre
with 14 c5! when Black is unable to capture due to tricks on the loose d7-bishop.
13 Rc1
Covering the c4-pawn, while preparing c5.
13 ... b6 14 h3!
Intending to eject the e5-knight with f4.
14 ... Rc8 15 Nec3 Ng8 16 Be2 Ne7 17 f4
Multiple negative conditions foretell strategic disaster for Black:
1. White holds a huge central territorial advantage.
2. Black’s king isn’t so safe, since White will soon play g4 and continue to try and open
the kingside.
3. Black’s position is passive and he can only await events. If he gives in to even the
slightest of White’s demands, then that action may produce further concessions.
17 ... N5c6 18 g4!
The feeling of constriction continues to grow in Black’s position.
18 ... hxg4 19 hxg4 Qe8 20 Bf3 Nxd5!? 21 cxd5
The idea of 21 exd5!? is to go after his g6-pawn, via the b1-h7 diagonal. After 21 ...
Rxh1 22 Rxh1 Na5 I mistakenly felt Black was getting some queenside play, but this isn’t
the case: 23 Bd4! (I missed this idea) 23 ... Nxc4 24 Qd3 b5 25 Be4! Kg8 26 Bxg6 Qe7 27
g5! and White has a winning attack, since 27 ... fxg5 is met decisively with 28 Rh7.
21 ... Na5
He hopes to annoy with threats of knight infiltration to c4.
22 Rxh8 Bxh8 23 Be2?!
This is a case of discarding the beneficial with the unwanted. A passive retreat isn’t
necessary. 23 Bd4! Bg7 24 g5! strips the pawn cover around Black’s king.
23 ... b5!
Threat: ... b4 and ... Qxe4+.
24 f5!
I decide to carry myself with aristocratic hauteur, pointedly ignoring his threat to
undermine my e4-pawn’s support. White’s attack doesn’t yet exist, so this is an attempt
to plant the seeds for it to grow. I realized that he was beginning to distract me from my
intent, so I gambled (correctly) that his exposed king would more than make up for a lost
pawn or two from my side.
24 ... b4
He picks his poison. 24 ... g5 25 Qd4 leaves White with a strategically won game. I
am a million times more competent in positions like this one, when compared to the wild
open one in the game’s continuation.
25 fxg6+ Kg8
Principle: avoid opening attacking lanes for your opponent. 25 ... Kxg6? 26 e5!
decisively opens lines for White. If 26 ... bxc3 27 Bd3+ Kg7 28 Qh2 Qg8 29 exf6+ Kf8 30
Bh6+ Ke8 31 Qh5+ Kd8 32 f7 wins.
26 Nd1 Qxe4+ 27 Bd3
White’s bishops point at Black’s king like an accusation. I achieved a winning attack,
but please keep in mind that your writer – an accountant posing as a movie action hero –
is sort of an IM un-Tal, who routinely botches such positions, especially when my clock
runs low.
27 ... Qxg4!
A fish can nibble on the bait and still escape if its bite avoids the hook. Danger may
approach us closely and we still may not see it. Brandon told me after the game he
considered 27 ... Qxd5??, but missed that 28 Bc4! winning his queen.
28 Nf2 Qf3 29 Ka1
The position becomes submerged in vague and even contradictory meaning. I feared
swaps with ... Bf5. To my shock, the comp says the swap isn’t a problem for White and
suggests 29 Bh6! Bf5 30 Ka1 Bxd3 31 Nxd3 Bg7 32 Bxg7 Kxg7 33 Qh2! Rh8 34 Rxc7+
Kxg6 35 Qxh8 and there is no back rank mate for Black.
29 ... Bf5

30 Be2?!
Both clocks are very low by this point. Once again 30 Bh6! is crushing, but downright
stupid would be to open the a-file for him with 30 Bxa6?? Ra8 31 Bc4 Bxg6 32 Nd1 Nxc4
33 Rxc4 b3 34 a3 f5!, threatening ... Rxa3+. Black has a winning attack.
30 ... Qg3 31 Bd4?!
I had neither the time nor the tactical ability to see comp-generated lines like 31
Bg4!!. It’s possible to be filthy rich in opportunities and not even know it: 31 ... Bxg4 32
Nxg4 Qxg4 33 Qh2! Bg7 34 Qh7+ Kf8 35 Bh6 forces mate.
31 ... Bxg6?!
I think he should leave the pawn alone, since he conveniently opened the g-file for
me.
32 Qxb4?!
Missing 32 Ng4! Qh4 33 Qxb4 Re8 34 Bf3 Qg3 35 Qc3 and if Black attempts to defend
f6 with 35 ... Rf8??, White has 36 Nh6+ Kg7 37 Qxc7+! Kxh6 38 Be3+ forcing mate.
32 ... Qg5?!
Threatening mate in one. Black may hold the game with 32 ... Re8! 33 Bf1 Qf4 34 Qc3
c5! 35 dxc6 Nxc6 36 Qxc6 Qxd4 when it’s anybody’s game.
33 Rd1?!
33 Rh1! Qxd5 34 Bc3! wins since Black is unable to retreat the knight due to Bc4,
pinning and winning Black’s queen.
33 ... Bc2?
He should have tried 33 ... c5! 34 dxc6 Nxc6 35 Qb7. I calculated to here, but missed
that he has the tricky 35 ... Ne7! when his knight is immune, due to a back-rank mating
threat on c1.
34 Rh1! Qxd5
He covers his loose knight.

Exercise (combination alert): White has two ways to win. Find one of them:

Answer: Double attack; a5 and g4 are the points of convergence. White threatens
both Qxa5 and also Qg4+, picking up his loose rook.
35 Bc3!
Answer no.2: 35 Bxf6! Bxf6 36 Qg4+ and White wins since Qxc8 follows.
35 ... Bg7
35 ... Nc6?? 36 Qg4+ pops the rook.
36 Qxa5 Qxa5
Keeping queens on the board will probably result in the black king getting mated.
37 Bxa5 f5 38 Rc1 Ba4 39 Bxa6 Rb8
39 ... Ra8 fails to regain the lost piece due to 40 Bc4+.
40 Bc3 Bxc3 41 Rxc3 c5 42 Rd3 Re8 43 b3 Re1+ 44 Kb2 Be8
If 44 ... Re2+ 45 Ka3 Rxf2 46 bxa4! and White’s passed a-pawn will cost Black the
game.
45 Rxd6 1-0
Summary
To my mind 6 ... Nh6 is perhaps Black’s most combative method of meeting White’s
dangerous set-up.

Game 26
S.Dordjevic-C.Lakdawala
San Diego (rapid) 2015

1 d4 g6
In this game I am confronted by my stylistic identical twin, so I played the Modern
since my very strong near-Senior Master opponent Stevan usually draws when I play 1 ...
d5?! and he responds with 2 Bf4!.
2 c4 Bg7 3 e4 d6 4 h3!?

A move made with the thought: sophisticated weaponry is never a substitute for a will
to fight. He wants to remain within his normal h3 King’s Indian lines, which are all the
rage these days. So he plants that flag. Still, this isn’t yet a King’s Indian. Secondly, his
move is unusual when played this early. I later exploited his move order by tossing in an
early ... f5 where he didn’t have access to his usual queen check on h5.
4 ... Nc6!
Black’s highest-scoring move in the database. Instead, 4 ... e5 5 d5 Ne7 6 Nc3 0-0 7
Bd3 Nd7 8 Be3 f5 9 f3 was reached in the later S.Dordjevic-C.Lakdawala, San Diego
(rapid) 2016. Here I shouldn’t allow White to play g4 and should play 9 ... f4 with a
decent position for Black, who will eventually play for ... g5, ... h5 and ... g4 at some
point.
5 Nf3 e5

6 d5

Tip: 6 Be3?! may be slightly inaccurate here as it can be met with 6 ... exd4!, since
in such structures White’s h3 is a redundancy, as he may require f3 to hold up the
e4-pawn.

After 7 Nxd4 Nf6 8 Nc3 0-0 9 Be2 Re8 there is White’s trouble: he really needs f3, but
if he plays it now, his kingside dark squares will be weakened by the insertion of the
unnecessary h3. Following 10 Nxc6 bxc6 11 Bf3 Rb8 12 Qc2 c5 Black will follow with ...
Nd7 and ... Ne5, with an edge, V.Guerra-T.Abergel, Lisbon 2017.
6 ... Nce7
Also possible is 6 ... Nd4!? 7 Nxd4 exd4 8 Nd2 Nf6 9 Bd3 0-0 10 0-0 c5 11 f4. I feel like
White’s central control is worth more than Black’s passed d-pawn.
7 Nc3
I thought he would play 7 g4!?. I was going to respond with the risky 7 ... f5!? 8 gxf5
gxf5 9 Rg1. Now Black can try 9 ... Ng6 (also possible is 9 ... fxe4!? 10 Rxg7 Nf5 11 Rg5
exf3 12 Qxf3 Qd7! when Black looks fine) 10 Bd3 f4 11 Ng5 Nf6! 12 Nxh7?! Rxh7 13 Rxg6
Kf7, P.Graves-C.Lakdawala, San Diego (rapid) 2011. Here Black has a clear advantage
after 14 Rg1 Qh8!.
7 ... f5
Note: And there it is: Black achieved ... f5 without blocking it with ... Nf6 first, which
means a King’s Indian formation a full move up, with no ill-effects, since in this version
White doesn’t get to disrupt castling with a queen check on h5.

8 Bd3
The alternative is 8 exf5 gxf5.

Warning: Always watch out for White’s Ng5 and Ne6 ideas. Our opponent may
even give up a pawn to bag our light-squared bishop, which generally offers
White strategic compensation.

For example, 9 Ng5 Nf6 10 Bd3 0-0 11 0-0 h6 12 Ne6. Always be on the lookout for
this potential pawn sacrifice to gain the bishop-pair and light squares. Here, however,
after 12 ... Bxe6 13 dxe6 Qc8 14 Nd5 Nfxd5 15 cxd5 c6 16 Bc4 Rf6 17 dxc6 Qxc6 18 Qb3
Rg6 19 f3 d5 20 Bd3 Qb6+ 21 Qxb6 axb6 the e6-pawn falls and I don’t see compensation
with White’s bishop-pair, since Black also gets a massive central majority.
8 ... Nf6 9 Qc2
I expected 9 exf5 gxf5 10 Ng5 0-0 11 Qb3 h6 12 Ne6 Bxe6 13 dxe6 Qc8 14 0-0 c6. At
some point White’s extended e6-pawn will probably fall and I’m not sure if White’s
bishop-pair and enhanced light-square control fully compensate. The comp thinks they
do.
9 ... 0-0 10 Bg5
He may have been better off just playing the bishop to d2, since ... h6 may help Black
more than harm. On 10 0-0 I planned to play 10 ... f4 intending ... h6 and ... g5, with a
kingside attack. Now if White tries 11 Ng5 h6 12 Ne6 Bxe6 13 dxe6 Nc6 (threat: ... Nd4
and ... Nxe6) 14 Nd5 Re8 the advanced e-pawn falls and I don’t believe in White’s light-
square compensation for the missing pawn.
10 ... h6

11 Bd2
I thought he would go for a light-square strategy with 11 Bxf6!? Rxf6. Even here I
prefer Black, since the game may later open to my bishop-pair’s advantage.

Tip: In such positions where your opponent is nervous about castling kingside,
then maybe threaten to open queenside lines in order to worry him or her
about castling long.

11 ... c6 12 dxc6!?
Risky. His idea is to pressure my backward d6-pawn, but this won’t be so easy since
right now there is a traffic jam of bishops clogging his potential d-file pressure. The
alternatives:
a) Castling long feels like being drawn into a fool’s battle, which can only be lost: 12
0-0-0? cxd5 13 cxd5 b5! looks exceedingly dangerous for White since attacking lanes
rapidly open for Black.
b) 12 0-0!? also feels like he may be castling into the teeth of Black’s attack. I
intended to first clog the centre with 12 ... c5 13 Rfe1 f4 and then proceed to attack down
the g-file with ... g5 and eventually ... g4, with dangerous attacking chances.
12 ... bxc6 13 0-0-0?!
Exercise (combination alert): White’s last move looks both logical and illogical.
He stays away from the pawn avalanche dangers of castling kingside, and instead
castles into an open b-file. However, the all-powerful comp says it’s a blunder.
This one is really difficult to see, but give it a shot. What would you play?

13 ... c5?!
The idea is to play ... Nc6 and ... Nd4.
Answer: Komodo found the unnaturally strong 13 ... d5!!. I looked at this briefly and
then incorrectly rejected it as suicidal. The move is counterintuitive, because it looks like
Black walks into the teeth of White’s power on d5, but maths is maths, and the comp
worked it out to a win after 14 cxd5 fxe4! (I completely missed this zwischenzug) 15
Nxe4 Nxe4 16 Bxe4 cxd5 17 Bc3 Be6! 18 Nxe5 (18 Bxg6 loses to 18 ... Nxg6 19 Qxg6 Bf5
20 Qg3 Rc8 21 Kd2 d4 with a winning attack for Black) 18 ... Qb6! 19 Bf3 Bxe5! (another
impossible-to-see comp move) 20 Bxe5 Rac8 21 Bc3 Rf4!, threatening ... d4, winning
material, no matter how White plays.
14 Nh4!?
This attempt to interfere – a hand under a running tap – fails to block Black’s intent.
He wants to resolve central tension, so I oblige, since he loses time.
14 ... f4 15 Nf3?!
Veering away from the correct plan is the equivalent of leaving your email only
partially composed and unsent. Now he loses the initiative. Correct was 15 g3! with even
chances.
15 ... Nc6 16 Nd5 g5 17 Bc3 Be6 18 a3 a5
Clamping down on future b4!? ideas.
19 Rhe1
This mechanical move indicates that White lacks a useful plan and must simply wait.
19 ... Nd7
The idea is to play ... Nd4 and when he swaps, play ... cxd4 and then slip my knight
into c5.
20 Qa4 Nd4 21 Bxd4!?
The price of folly is generally paid in the future. There will be a heavy dark-square
cost for this move. He may have been better off with the humble 21 Rd2.
21 ... cxd4 22 Qc6?!
His queen is close to trapped here. 22 Kb1 Nc5 23 Qc2 Rb8 doesn’t look like much fun
for White, but should be played.
22 ... Nc5 23 b4!?
This isn’t a situation where a rising power attempts to meet an already established
power. It’s more an attempt to shake off the pressure before the rot sets in. Alternatives
are also grim:
a) 23 Qc7 Nxd3+ 24 Rxd3 Qxc7 25 Nxc7 Bxc4 wins material.
b) 23 Nc7 Qc8! 24 Qxd6 Nxd3+ 25 Rxd3 Bxc4 26 Rd2 Ra7 27 Nd5 Bxd5+ 28 Rc2 Bc4
and if White attempts to regain the lost piece with 29 b3 then comes 29 ... Ra6! 30 Qe7
Bf6 when White’s queen is trapped on an open board, or he loses heavy material.
23 ... axb4 24 axb4

Exercise (combination alert): Black has two ways to win here, based on the
inauspicious positioning of White’s queen. Find either.
Answer: Queen trap.
24 ... Rc8!
Answer no.2: I didn’t see the comp’s variation which comes at White’s queen from
another angle: 24 ... Ra6! 25 Qb5 Nxd3+ 26 Rxd3 Qa8!, threatening mate in two moves,
starting with 27 ... Ra1+. After 27 Rb3 simplest is 27 ... Rb8 28 Nc7 Rxb5 29 Nxa8 Rxa8
30 cxb5 Bxb3. Black bagged an extra piece.
25 Qb5
25 Qb6 Nxd3+ 26 Rxd3 Rxc4+ 27 Kd1 Qxb6 28 Nxb6 Rxb4 is also completely hopeless
for White.
25 ... Bxd5!?

Tip: Our goal isn’t always to unearth the position’s truth. The practical player simply
wants to win. How he or she gets there is a secondary point.

Sometimes our position is at odds with our predisposed stylistic and psychological
tendencies, and when nervous doubt creeps into our mind, we fight a two front war. I
was in my usual time pressure and chose (perhaps wisely) the safer, more clear, practical
route. A braver/less practical player would have gone for 25 ... Rb8! 26 Qc6 Rxb4!! 27
Nxb4 Qa5 28 Qb5 Qa1+ 29 Kd2 Qb2+ 30 Nc2 Qc3+ 31 Kc1 Nb3+ 32 Kb1 Ra8 which is
decisive. The trouble is this line is very difficult to see in time pressure, while the one I
played in the game is easy to see.
26 bxc5
White is crushed after 26 exd5?? Rb8 27 bxc5 Rxb5 28 cxb5 Qa5.
26 ... Rxc5 27 Qb4 Be6
I didn’t see 27 ... Qa8! 28 exd5 Rb8, trapping his queen, since 29 Qd2 walks into 29 ...
Qa1+ 30 Kc2 Qb2 mate.
28 Kd2!
The best practical chance is for his king to attempt to run to the other side.
28 ... Qc7
We begin to see the true extent of the damage. Not only is Black about to win a
second pawn, but he also continues to attack.
29 Ke2 Rb8 30 Qa4 Rxc4!
Stronger than the capture with my bishop.
31 Qa2
31 Bxc4 Bxc4+ 32 Kd2 Rb2+ 33 Kc1 Rb3! is deadly for White.
31 ... Bf7
This simple move breaks White’s temporary pin of my rook.
32 Kf1 Rc3 33 Qe2 Rcb3 34 Rc1 Rc3 35 Rb1 Rxb1 36 Rxb1 Rc1+
Principle: swap when up on material.
37 Rxc1 Qxc1+ 38 Ne1 Bf6!
Intending ... Bd8 and ... Ba5.
39 Qc2 Qxc2 40 Nxc2
Intending Nb4, which clamps down on Black’s d5 break.
40 ... d5 0-1
White is two clean pawns down and reasonable technique will win.

Summary
White can sometimes be duped into entry into a King’s Indian position a full move down,
if they allow an early ... f5 without harmful effects for Black.

Game 27
P.Hepworth-T.Gharamian
London (rapid) 2015

1 e4 g6 2 d4 Bg7 3 c4 c5
Our move order runs 3 ... d6 4 Nf3 Bg4 5 Nc3 c5.
4 Nf3 d6 5 Nc3 Bg4

The unknown is so much more alluring than the known. This idea, a kind of controlled
mutation of the opening’s norm, is similar to my game with Graves from earlier in the
chapter. Black targets all the defenders of d4. Instead, 5 ... cxd4 6 Nxd4 Nc6 7 Be3 leads
to Maroczy bind positions which are solid for Black, but also very difficult to win.
6 d5

Note: 6 dxc5 dxc5! is the exception to our normal ... Bxc3-all-the-time-rule.

After 7 Qxd8+ Kxd8 8 Be3 b6 9 0-0-0+ Kc8 Black’s loss of castling is made up for by
White’s potential weakness on the d4-square. Instead, 6 Be3 cxd4 7 Bxd4 Nf6! gives
Black a very favourable version of the Maroczy bind, since White loses a tempo on the
coming ... Nc6.
6 ... a6
This is a useful move for Black in such Benoni-like structures. White will have to be on
the lookout for future ... b5 ideas. As mentioned above, Black’s ... Bxc3 idea is less
favourable if White already achieved e4, as in this case, but 6 ... Bxc3+!? is still playable:
7 bxc3 Qa5 8 Qc2 Bxf3 9 gxf3 Nd7 10 f4 Ngf6 11 Bd3 0-0-0 12 0-0. I prefer White’s
bishop-pair and space, over Black’s superior structure, R.Kreisl-V.Malakhov, Novi Sad
2016.
7 a4
White suppresses ... b5 ideas.
7 ... Nf6 8 h3
Instead, 8 Be2 0-0 9 h3 Bxf3 10 Bxf3 Nbd7 11 0-0 Ne8 12 Be2 Bd4! 13 Ra3 Ng7 14
Kh1 e5 15 dxe6!? fxe6 16 f4 was D.Debashis-V.Malakhov, Doha (blitz) 2016. Here Black
can play the retro 16 ... Nb8! 17 Bf3 Nc6, and if White attempts to attack with 18 g4
Black can consider the sacrifice 18 ... g5!? to dominate the central dark squares.
8 ... Bxf3 9 Qxf3

Tip: In most Benoni or quasi-Benoni structures like this one, Black’s light-squared
bishop is often the problem piece, with no useful function early in the game.
So it’s logical to play ... Bg4 and ... Bxf3 in many versions.

9 ... 0-0 10 Be2 e6 11 0-0


In such structures, some of my students are often nervous about dxe6. This shouldn’t
worry us in the vast majority of Benoni structures. In this instance Black’s central control
is enhanced, easily compensating the potential weakness of d6: for example, 11 dxe6
fxe6 12 e5 Nd5! 13 Qg4 Nb4! (threat: ... Nc2+ and ... Nxa1) 14 Qxe6+ Kh8 15 0-0 Bxe5
16 Bh6 Re8 17 Qg4 N8c6. Black’s domination of the central dark squares makes up for his
weakness on the light squares.
11 ... exd5 12 cxd5
The symmetrical recapture 12 exd5 is more solid and duller. Play may go 12 ... Re8 13
Bf4 Nh5 14 Bd2 (14 Bh2 is met with 14 ... Be5! which eliminates White’s bishop-pair) 14
... Nd7 15 g4 Bxc3! 16 Bxc3 Nhf6 17 Rfe1 Ne4 18 Bf1 Nxc3. Black eliminated White’s
fearsome bishop-pair and stands at least even, since White’s remaining bishop travels on
the same colour of many of its remaining pawns.
12 ... Nbd7

Black achieved a pleasant Benoni position, because he managed to eliminate his


problem light-squared bishop early on. Chances look balanced.
13 Bf4 Ne8!?
Protecting d6, while enhancing control over e5. The alternative is to set up with 13 ...
Qe7 which seems more natural to me.
14 Qg3 Rb8!
Ignoring White’s ‘threat’.
15 a5
White sees that his threat isn’t real, since 15 Bxd6?? Nxd6 16 Qxd6? Be5 traps the
queen.
15 ... Qc7 16 Bg5
Threat: Be7, winning the exchange.
16 ... Nef6 17 Bf4 Ne8 18 Bg5
White hopes for a repetition draw against his GM opponent.
18 ... Nef6
18 ... Bf6 fails to escape White’s attempt to force a repetition draw after 19 Bh6 Ng7
20 Bf4 Ne8 21 Bh6, repeating the position.
19 Bf4
Draw?
19 ... Ne5!?
No! Once we choose to enter such a dramatic structural alteration, we can never
unchoose. This is a high-risk winning attempt for Black, who allows White a protected
passed pawn.
20 Bxe5 dxe5

Komodo frowns upon Black’s position, which on first glance looks rather sorry. After
all, White owns a protected passed pawn, and Black’s bishop looks like the inferior one in
this opposite-coloured bishops situation. Don’t underestimate Black’s resources. My
rival/buddy IM Dionisio Aldama once showed me a position very similar to this one,
where he as Black defeated GM Akobian with the plan ... Ne8, ... Nd6 and eventually ...
f5. As it turned out, Black’s dark-squared bishop eventually proved more valuable than
White’s light squared counterpart.
21 f4?

Tip: If we don’t know what action to take, then the next step is to figure out what
actions we are unable to take. The absence of proof of problem A, can be proof
of problem B. Then again, it may not be proof of anything!

White has trouble finding a plan and lashes out with a pseudo-active move which
assumes the Incredible Hulk’s ‘You-won’t-like-me-when-I’m-angry’ act-first, think-later
mindset. The problem with the move is it greatly enhances his weakness on the dark
squares. Correct was a line like 21 Qe3 Ne8! (principle: blockade your opponent’s passed
pawns) 22 Rfb1 (intending b4) 22 ... Nd6 23 Na4 c4 24 Nb6 f5! with a sharp game with
chances for both sides.
21 ... exf4 22 Qxf4
This leads to a clearly inferior ending. Even worse is 22 Rxf4? Ne8 and the threat of ...
Be5 forces 23 Rf3 Be5!. “Maleficos non patierio vivere,” mutters the bishop, which
translates to: ‘Thou shalt not suffer a witch to live’. Principle: opposite-coloured bishops
favour the attacker. White is in far worse trouble here than he got in the game
continuation.
22 ... Qxf4 23 Rxf4 Nd7
Black immediately seizes upon the weak e5-square.
24 Na4 Rfe8 25 Bd3 Ne5
Of course, c5 isn’t really hanging, since White’s knight gets overloaded.
26 Bc2?
Correct was 26 Be2 keeping control over the c4-square. Then 26 ... c4 maintains a
strategic advantage for Black.

Exercise (combination alert): White blundered on his last move. How did Black
take advantage?
Answer: Double attack. The b2 and a5 points are loose.
26 ... Nc4!
The knight acts like a micro-saboteur, continually provoking weakness in White’s
camp.
27 b3!?
Selecting our manner of torment doesn’t really do much to ease our pain. White
realized that 27 Ra2 is met with 27 ... Ne3! 28 b3 Bd4 29 Kh1 f5! 30 Nb6 g5 31 Rf3 fxe4,
which leaves him in deep trouble.
27 ... Nxa5!
I like this decision. Black indicates that he will not be bought off. He rejects the
material route with 27 ... Bxa1 28 bxc4 when White has some, but clearly not enough
compensation for the exchange.
28 Ra2
A more accurate square for the rook was a3.
28 ... c4! 29 bxc4
Alternatively:
a) 29 Nc5 cxb3 30 Nxb3 Nc4 leaves Black up a pawn with two connected passers.
b) 29 b4 Nb3 30 Kh1 b5 31 Nc5 Nxc5 32 bxc5 b4! when Black’s passed pawns are
faster than White’s.
29 ... Nxc4
Black just achieved two connected queenside passers.
30 Bb3 Nd6
Pressuring the weak e4-pawn.
31 Bc2 Rbc8 32 Bd3 b5! 33 Kf2
White decides to play on, down a full piece. 33 Nb2 Bd4+ 34 Kh1 Rc1+ 35 Rf1 Rxf1+
36 Bxf1 Nxe4 is equally hopeless, since ... Ng3+ is threatened.
33 ... bxa4 34 Rxa4 Ra8
The a-pawn is kept alive, since taking it costs White his e-pawn. The death-watch isn’t
far from the wake. To play on here against a strong GM is a waste of time, since a titled
player would regard such a position with the eye of the undertaker who merely needs to
measure the corpse for the coffin. I’m sorry Scarlett, but your team lost the war and your
new anti-bellum reality is to dig for ... Oh, wait, sorry. I already used this line in an earlier
game in the book.
35 g4 a5 36 g5 Ra7 37 Rg4 Bc3 38 Rg1 Rb7 39 Kf3 Rb3 40 Rb1 Rxb1 41
Bxb1 Kg7 42 Ke3 h5 43 gxh6+ Kxh6 44 h4 f5! 45 Kd3 Nxe4 46 Kc4 Bb4 0-1

Summary
When White sets up with d4, c4, e4 and Nf3, meet it with ... Bg4 and ... c5, pressuring
White into resolving the central tension with d5.

Game 28
P.Nikolic-V.Topalov
Monaco (blindfold) 1998

1 d4 g6 2 Nf3 Bg7 3 c4 d6 4 g3
White hopes to enter g3 King’s Indian lines.
4 ... c5
Topalov’s move keeps the position in Modern realms, at least for a while. If you don’t
care for the Benoni structures, then there is nothing wrong with 4 ... e5 5 Nc3 (or 5 dxe5
dxe5 6 Qxd8+ Kxd8 7 Nc3 c6 with a perfectly playable ending, similar to the ones we saw
in the Averbakh chapter) 5 ... Nd7 6 Bg2 Ne7!? (this move keeps it within Modern realms,
since we don’t block our f-pawn, as in King’s Indian lines) 7 0-0 0-0 8 e4 exd4 9 Nxd4 Nc6
10 Nde2 (principle: the side with extra space should avoid swaps) 10 ... a5. Black’s
activity for the most part makes up for White’s extra space.
5 d5
Alternatively:
a) 5 Bg2 Nc6 6 d5 Na5 7 0-0 a6 8 Qc2 Nf6 9 Nfd2 b5 10 e4 0-0 11 h3 Rb8 and I
already like Black, due to his accelerated queenside counterplay, E.Magerramov-
V.Tseshkovsky, Daugavpils 1978.
b) 5 dxc5 Qa5+ 6 Nbd2 (after 6 Bd2 Qxc5 7 Nc3!? Qxc4 8 Rc1 Nc6 9 e4 Qe6 the comp
says White obtains compensation, while your human writer says it’s not enough and Black
stands better) 6 ... Qxc5. White reaches a passive version of the norm, since his knight on
d2 isn’t placed well and obstructs the development of his queenside.
5 ... b5!

Note: The queen’s pawn versions of the Modern are slippery systems, where Black
can reach positions resembling the King’s Indian, Benoni, and also Benko Gambit, an
opening where White’s and Black’s position appeals to completely opposite
constituencies.

It’s a marketplace of haggling. In this case it is initiative in exchange for material, with
Black buying and White selling. I think the fianchetto line is a rather harmless way for
White to meet the Benko, so don’t be nervous about being a pawn down and trying it out.
6 cxb5 a6
This gambit comes with a pre-formed agenda for both sides. It’s one of those plastic
toys which you shape to your will, but then it immediately reverts to its original form.
Black hopes to justify his pawn-down status by exerting pressure down the open a- and
b-files, while White must walk the tricky line between consolidation and overextension of
his extra pawn. In my opinion Black gets full compensation in this version of the gambit.
Likewise, with 6 ... Nf6 7 Nc3 a6 8 bxa6 Qa5 9 Bd2 0-0 10 Bg2 Bxa6 11 0-0 Nbd7 12 Qc2
Rfb8 13 Rfd1 Ng4 14 Be1 Nge5 15 Nxe5 Bxe5 16 Rdb1 Ra7 17 a4 Rab7 Black achieved full
compensation for the pawn due to his queenside pressure, H.Reefschlaeger-G.Kasparov,
Hamburg (simul) 1987.
7 bxa6 Nf6 8 Bg2 0-0 9 0-0 Bf5!?

This is a solution which instantly produces new questions. The move is odd in the
Benko, but I like it here. The idea:
1. Black will play ... Nxa6 and ... Nb4!.
2. White will be nervous about infiltration into c2, and be provoked into e4.
3. Black plays the retrograde ... Bc8!, intending to return the bishop on the newly
weakened f1-a6 diagonal, with full compensation for the pawn.
Instead, 9 ... Bxa6 reaches normal Benko Gambit lines, where Black’s compensation
for the pawn looks decent.
10 h3
After 10 Nc3 Nxa6 11 Nd2 Nb4 12 e4 Bc8! Black’s bishop is ready to transfer to the
newly weakened f1-a6 diagonal.
10 ... Nxa6 11 g4?!
Neither side seems to be in a mood to bandy opening platitudes. This perhaps too-
aggressive move looks unjustified, since it weakens White’s king and allows Black to
regain the pawn at the cost of the bishop-pair. It isn’t too late for 11 Nfd2 Nb4 12 e4 Bc8,
which is similar to the above line.
11 ... Bxb1! 12 Rxb1 Nc7
Black simultaneously attacks d5 and a2, regaining the pawn, and with it the better
position, since he still retains his Benko Gambit advantages – except without the gambit
part anymore.
13 a3 Nfxd5
Black looks better, since his central preponderance outweighs White’s queenside
pawns.
14 Nh2 e6 15 g5!?
It isn’t easy to predict the chain of consequences which follow this decision. White
continues unconscious of restraint or fear and it feels like the desire to attack holds undue
sway on his decision-making process. He hopes to play Ng4 and make something of the
dark squares around Black’s king, but it’s hard to ignore the artificial appearance of
White’s moves.
15 ... Rb8 16 Qd3 f5!?
There’s a war brewing on the kingside. Also promising is 16 ... Nb5 17 e4 Ndc7 18 Ng4
e5! 19 b4 Ne6 20 bxc5 dxc5 21 Qxd8 Rfxd8 22 a4 Nbd4 23 Rxb8 Rxb8 with advantage to
Black.
17 Bxd5?!
I don’t understand why the grandmaster gave up his light-squared bishop. White can
minimize his disadvantage with 17 gxf6 Qxf6 18 Ng4 Qd4 19 Qc2.
17 ... Nxd5
It will be difficult to eject this knight.
18 e4!?
The idea is to pressure e6 and allow his knight access to g4, but in doing so he
weakens his king.
18 ... fxe4 19 Qxe4 Qd7 20 Ng4
20 ... Ne7
Topalov clears the way for his central pawns to move forward. He may also be
thinking about playing ... Nf5 and ... Nd4 in the future. Instead, 20 ... Qf7! 21 Re1 Kh8!
looks very promising for Black: ... Bd4 may be coming and White can’t play 22 Qxe6??,
since it hangs a rook to 22 ... Rbe8 23 Qxf7 Rxe1+ (zwischenzug) 24 Kg2 Rxf7.

Did You Know? The German chess term zwischenzug means in-between move.

21 Re1 e5 22 b4!
It’s critical for White to chip away at Black’s central pawn mass.
22 ... d5!?
When we grasp at multiple priorities, we end up with no priorities. Black retains a
clear advantage with 22 ... Nc6!.
23 Nxe5! Qxh3
After 23 ... Qd6! 24 Nc4! Qc6! 25 Qg4 Nf5 White’s king looks dangerously exposed.
24 Qg4!
Desperation often gives birth to clarity. The black queen’s plans for world domination
just hit a glitch, since White’s king will be a lot safer with the removal of queens. GM
Nikolic, from a position of earlier inferiority, managed to outplay a future world champion
in this high-quality blindfold epic, and comes close to dynamic equality.
24 ... Qxg4+ 25 Nxg4 Nc6 26 Bb2 cxb4 27 Bxg7 Kxg7 28 axb4 Rf5
28 ... Rxb4 29 Rxb4 Nxb4 30 Nh6! (intending to give check on e7) 30 ... Nc6 31 Re6
Nd4 32 Re7+ Kh8 33 Nf7+ Kg7 34 Nh6+ is perpetual check.
29 b5 Rxg5 30 bxc6?!
Missing 30 f3! (if this were Monopoly, then this move represents White’s get-out-of-
jail-for-free card) 30 ... Nd4 31 Re7+ Kh8 32 Kh1! (the f-pawn is untouchable due to
White’s Nf6 threat) 32 ... Rxb5 33 Rxb5 Rh5+! 34 Kg2 Nxb5 35 Nf6 Rh6 36 Nxd5, which is
an easily drawn ending.
30 ... Rxg4+ 31 Kf1 Rc8 32 Rb7+ Kf6 33 Rd7 Rxc6 34 Rxd5

Note: This position would be an easy draw for White if only one pair of rooks
remained.
The second pair greatly complicates the issue and offers Black winning chances.

34 ... h5 35 Red1 Rcc4 36 R5d3 h4 37 Rf3+ Kg7 38 Re1 Kh6 39 Rd1 Ra4 40
Re1 Ra5 41 Rd1 Rag5
Now ... Rg1+ followed by ... Re5+ is threatened.
42 Rd8 Rg1+ 43 Ke2 Ra5!
Offer my dogs bones and they gnaw on them for hours. This is one of those positions
which is probably drawn/not-so-easy-to-draw, since White can play on for an eternity,
hoping to induce an error. Black’s best chance to win is to coordinate the following:
1. Push his passed h-pawn as far down the board as possible without actually losing it.
2. Keep White tied down to defence of f2.
3. Move his king forward without walking into tactics or perpetual check.
44 Rh8+ Kg5 45 Rd3 Re5+ 46 Kf3 Rf5+ 47 Ke2 Ra1
Exercise (critical decision): There is but a short distance between paradise and
hell.
Black threatens ... Ra2+, picking up the f2 pawn. Should White defend passively with
48 Rd2, or should he counterattack Black’s h4-pawn with 48 Rd4? The only thing
which separates White from the future he wants is a clear drawing plan. One path
offers decent drawing chances, while the other loses.

48 Rd4?
It’s unwise to pursue an object you don’t actually want to catch. There are some who
interpret the choosing of a lesser evil as a kind of micro-defeat. In this case the endgame
principle remain active at all costs in rook endings is the exception to the normal rule.
Answer: 48 Rd2! is a very difficult move to play, since to do so we must escape our
inborn prejudice for passivity. The move – a contradiction in rook endings – offers decent
chances to save the game after 48 ... Ra3 49 Kf1 Kg4 50 Kg2 when Black’s progress isn’t
such an easy matter.
48 ... Ra2+ 49 Ke3 Raxf2 50 Rdxh4 R5f3+ 51 Ke4
Exercise (planning): Come up with a winning plan for Black:

Answer: Simply swap one pair of rooks, after which Black achieves a won ending,
since White’s king is cut off.
51 ... Rf4+! 52 Rxf4 Rxf4+ 53 Ke3 Kg4 54 Rh1 Rf3+ 55 Ke2 g5! 56 Rg1+
Rg3 57 Ra1
Or 57 Rxg3+ Kxg3 58 Kf1 Kh2 59 Kf2 g4 and Black promotes.
57 ... Rg2+ 58 Kf1
If White’s king is allowed access to g1, then it’s a draw, but White has no way to
penetrate the black rook’s barrier.
58 ... Kh3!
I’m a bit depressed by the fact that Topalov plays this technical ending blindfolded at
least as accurately as I would sighted.
59 Ra3+ Kh2 60 Ra5 g4 61 Rh5+ Kg3 62 Ra5 Rb2 63 Ra3+ Kh2 64 Ra8
Rb1+ 65 Kf2 g3+ 66 Kf3 g2 67 Rh8+ Kg1 68 Ke2 Rb5
69 Re8

Tip: On 69 Rh7 Black wins with a technique known as building a bridge, with
69 ... Re5+ 70 Kd3 (if 70 Kf3 Kf1 and Black promotes) 70 ... Kf2 71 Rf7+ Kg3
72 Rg7+ Kf3 73 Rf7+ Kg4! 74 Rg7+ Rg5 75 Rxg5+ Kxg5 when White’s king
is too far away to stop promotion.

69 ... Rh5 0-1


70 Ra8 Kh1 with ... g1Q to follow.

Summary
If White reacts to our early ... c5 with the set-up g3, Nf3 and d5, without having played
Nc3, then we have the option to play ... b5, entering a line of the Benko Gambit. You
don’t need to study the entire Benko Gambit. All you need to know is this one particular
line.
Chapter Eleven
English Opening Set-ups
1 c4 g6 2 Nc3 Bg7 3 g3 d6 4 Bg2 e5

Let’s keep it simple in this chapter and go for ... e5 structures in all cases. White has
three main set-ups in the English:
1. White plays Nf3 and then pushes on the queenside with Rb1 and b4.
2. White plays the Botvinnik English set-up with e4 and Nge2.
3. White plays e3 and Nge2, keeping the structure flexible.
As mentioned above, we meet all three set-ups with ... e5. We can toss in ... f5, or we
can also avoid it and play ... Nf6, followed by a future ... c6 and ... d5 break. In all cases
Black should generate adequate counterplay.

Game 29
D.Aldama-B.Xia
San Diego (rapid) 2017

1 c4 g6
It isn’t easy teaching kids since they rarely get my jokes. In this game my 13-year-old
student Brandon takes out an experienced IM with the Modern, which is all the more
astounding, since we had only been working on the Modern for two weeks prior to this
game.
Here is an example of what can happen if White’s queenside initiative goes
unchallenged: 1 ... e5 2 Nc3 Nc6 3 g3 g6 4 Bg2 Bg7 5 Rb1 d6 6 b4 f5 7 b5 Nce7 8 d3 Nf6
9 Nf3 0-0 10 0-0 h6 11 c5.

Tip: This is a common queenside line-opening mechanism for White.

C.Lakdawala-B.Baker, San Diego (rapid) 2016, continued 11 ... Ne8?! (11 ... Be6 is
fine for Black) 12 cxd6 cxd6 13 Qb3+ Kh7 14 Nd2 Bf6 15 a4 Rb8 16 Ba3 b6 17 Rfc1 Bb7
(he wants to weaken my king, but he also weakens his own queenside and central light
squares with this swap) 18 Bxb7 Rxb7 19 Nd5 Nxd5 20 Qxd5 Rd7 21 Rc6 Be7 22 Rbc1 Nc7
23 Qa2 d5?! 24 Nf3 Bxa3 25 Qxa3 Re8 26 d4! exd4 (26 ... e4 27 Ne5 Rg7 28 Nxg6! is a
winning breakthrough for White) 27 Nxd4 Qe7 28 Qc3 Rc8 (this is the equivalent of
resignation, but 28 ... Na8 doesn’t look all that much better, since the knight will never
see the light of day again) 29 e3 h5 30 h4 Qe5 31 Nf3 Qxc3 32 R1xc3 Kg7 33 Ne5 and 1-
0; g6 falls.
2 g3 Bg7 3 Bg2 d6 4 Nc3 Nc6 5 Nf3
Alternatively, 5 d3 e5 6 Rb1 a5 7 a3 Nf6 8 b4 axb4 9 axb4 Rb8 (a novelty, which has
never been repeated, although there is nothing wrong with it, as Black prepares the way
for ... Ne7, ... c6 and an eventual ... d5; normal is 9 ... 0-0) 10 b5 Ne7 11 Nd5!? (11 Nf3 is
White’s main move; here play can go 11 ... 0-0 12 0-0 h6 13 Bb2 Be6 14 Qc2 c6 15 Ba3
Qc7 16 Nd2 Rfd8 and Black’s position looks fine) 11 ... Nfxd5 12 cxd5 0-0 13 Qb3 Bd7 14
Ba3?! (14 Nf3 c6! equalizes) 14 ... c5! 15 b6?! (unappetizing as it looks, White had to try
15 bxc6 bxc6 16 Qxb8 Qxb8 17 Rxb8 Rxb8 18 dxc6 Nxc6 19 Bxd6 Rb1+ 20 Kd2 Nd4,
although Black’s massive development lead easily outweighs White’s extra pawn) 15 ...
Ra8 16 Nf3 Ba4 17 Qa2 Nc8 18 Nd2 Nxb6 when Black won a clean pawn and (barely!)
converted, K.Shirazi-C.Lakdawala, Pasadena 1983.
5 ... e5 6 0-0 f5
Brandon, a former Leningrad Dutch Defence player, feels comfortable in the ... f5
lines. My personal preference is for 6 ... Nf6 playing for a future ... Ne7, ... c6 and ... d5
plan, after many further preparatory moves. Play might run: 7 d3 0-0 8 Rb1 a5 9 a3 h6 10
b4 axb4 11 axb4 Be6 12 b5 Ne7 13 Bb2 c6 14 Qc2 Qc7 15 Nd2 Rfc8. It would be difficult
for White to claim an edge here since Black, at some point, will achieve ... d5 and can
even play for a win with the extra central space.
7 d3 Nf6 8 Rb1

Note: There are two schools of thought in this structure: the play ... a5 camp,
and the avoid ... a5 camp.

My preference is for the latter, since when Black avoids ... a5, he often drops his a-
pawn, although the pawn grab costs White time.
8 ... 0-0
When I was a kid my Petrosianic, safety-first mother forced me to wear a super-geeky
plastic helmet when riding my bicycle. Of course, this was tantamount to an invitation to
all the other non-helmeted cool kids to crash into me and knock me off my bike.
Unfortunately my mother’s upbringing always makes me play the safest possible move,
so I generally go for 8 ... a5. In this way Black can’t later drop his a-pawn, as was the
case in Brandon’s game: 9 a3 0-0 10 b4 axb4 11 axb4 (this is virtually the same as our
main game, except Black has no weak a-pawn to lose) 11 ... h6 12 b5 Ne7 13 Bb2 g5 14
Ra1! Rxa1 15 Qxa1 Be6 16 Qa7 b6.

Note: It was former world champion Boris Spassky who realized (with colours
reversed, in the Closed Sicilian) that this is the optimal defensive queenside
structure for Black, since White can only target c7, which isn’t so easy to reach.
Meanwhile, Black can build for a kingside assault.

Now 17 Rc1 was seen in L.Van Wely-S.Movsesian, Wijk aan Zee 2009, when the
chances look approximately balanced after 17 ... Nd7.
9 b4 Rb8?!
9 ... a6 is correct.
10 Be3?!
I don’t particularly like this move, since it basically loses a tempo to Black’s coming ...
f4. Far more promising is 10 b5! Ne7 11 Qa4 with a clear advantage for White.
10 ... Ng4?!
Black should try to exploit the bishop’s position with the bold 10 ... f4!. Now if White
accepts with 11 gxf4 Ng4 12 Nd5 Nxe3 13 fxe3 exf4 14 exf4 Ne7 it feels to me like Black’s
dark-square power offers full compensation for the pawn.
11 Bg5! Qe8
11 ... Bf6 12 Bd2 leaves Black’s f6-bishop and g4-knight awkwardly placed.
12 Nd5 Qf7?!
12 ... e4 was necessary. Even then Black is in big trouble after 13 b5! exf3 14 exf3
when White regains the temporarily sacrificed piece with a strategic advantage.
13 b5 Nd8

Exercise (combination alert): White has no less than three ways to a winning
position. Find one of them.

14 Qa4
Answer: This is the least promising of White’s favourable moves. Even stronger is 14
Bxd8! Rxd8 15 Ng5! Qd7 16 Qa4 (threatening to trap Black’s rook with Qxa7) 16 ... a6 17
c5! dxc5 18 Qc4 (threatening horrendous discoveries) 18 ... Kh8 19 bxa6 Nh6 20 Qxc5
when White is a pawn up with a winning position.
Answer no.2: Simplest is 14 Be7! which picks up a clean exchange. If 14 ... Re8??
White traps Black’s queen with 15 Ng5.
14 ... Ne6
When we embark on such a sacrifice, we essentially hand over free will and allow the
position’s currents to carry us where they will. Brandon decides his a7 complaint is better
left unvoiced. Unfortunately he misplayed the opening and doesn’t get enough
compensation. However, 14 ... Ra8? is too slow and allows White 15 Bxd8! Rxd8 16 Ng5
Qd7 17 c5! threatening c6 and Nxc7. If 17 ... Rf8 (17 ... dxc5?? 18 Qc4 Kh8 19 Nxc7! wins,
since the check on f7 is deadly for Black) 18 h3 Nh6 19 cxd6 Qxd6 20 b6! cxb6 21 Nxb6
Rb8 22 Qxa7 Black is busted.
15 h3?!
Correct is 15 Be7! Re8 16 Qxa7 Bd7 17 Ng5 Nxg5 18 Bxg5 with a clean extra pawn for
White.
15 ... Nf6?!
Brandon misses 15 ... e4! seizing the initiative.
16 Qxa7
So White won Black’s a-pawn, but it did cost him time, so Black obtained some
compensation for it.
16 ... Bd7 17 Qe3!?
IM Aldama fishes for complications, but why, when he stands better? Better was to
reduce Black’s attacking potential with 17 Bxf6!.
17 ... Nxd5
After 17 ... f4! 18 Nxf6+ Bxf6 19 gxf4 exf4 20 Bxf4 Nxf4 21 Qxf4 Qe7 Black is two
pawns down, but gains dangerous attacking compensation.
18 cxd5 Nc5

18 ... Nxg5 19 Nxg5 Qe7 20 Ne6! Bxe6 21 dxe6 Qxe6 22 Rbc1 is clearly in White’s
favour.
19 d4?!
We sometimes make the wrong move for the right reason. White correctly realizes
that he must disturb the centre, but does it the wrong way, since his move loses both his
extra pawn and the initiative. White’s admonition takes effect best with 19 Nxe5!.
Brandon told me he saw White’s trick but underestimated its force: 19 ... Bxe5 20 d4 is
clearly in White’s favour.
19 ... exd4 20 Nxd4 Rbe8!
The correct rook.
21 Qd2
Pieces are not machines, which can be powered down when not in use. The IM reels
and the advantage begins to swing to Black. Perhaps better was the immediate 21 Ne6
Nxe6 22 dxe6 Bxe6 23 Rbc1 Bxa2 24 Qa7 with a messily even game.
21 ... Ne4!
These days many of the kids we face play with adult intensity.
22 Bxe4 fxe4!
Now the d5- and h3-pawns hang simultaneously.
23 Ne6!
White returns the pawn to eliminate Black’s dangerous light-squared bishop.
23 ... Bxe6 24 dxe6 Qxe6 25 Kg2 Ra8
Attacking a2.
26 Qc2?!
We must deal with the resources we have on the board, not with the ones we hope to
have. This is a temporary rage, the way public outrage over a political scandal tends to
die down at the end of a three-day news cycle. He had to go passive and brace himself
with 26 Rb3 d5.
26 ... Rxa2 27 Qxc7 Rxe2
Shockingly, Black is now up a pawn. There was a time earlier in the game when he
couldn’t even imagine such wealth.
28 Be3
Likewise, after 28 Rbe1 Rxe1 29 Rxe1 Bd4 30 Re2 Qd5! White is unlikely to survive.

Exercise (planning): We sense a storing up of energy in Black’s position, hoarding


it for future contention. Well, the future is right now. Find one powerful idea and
Black gets a strong attack.
Answer: Target the e3-bishop and, by extension, the f2-pawn.
28 ... Rf3!
Threat: ... Rfxe3. Time pressure has one of two effects on us:
1. It makes us play like a jackass.
2. It magically focuses our mind, dilating our awareness.
From this point on, we see a clear example of number two on the list in Black’s play.
29 Qd8+?!
This makes matters worse, but White was unlikely to survive 29 Rfc1 Be5! (threat: ...
Rxg3+; 29 ... Rfxe3? 30 Qc8+ Qxc8 31 Rxc8+ Kf7 32 Kf1! forces Black to hand over the
exchange) 30 Kg1! Bxg3 31 Qxb7 Rf8 32 Rc7 Qf6!.
29 ... Bf8 30 Bf4
A weak glue binds the white defensive barrier. If 30 Ba7 Qe5! (threat: ... Qxg3+) 31
Qh4 h5! (under the veneer of defensive necessity lies a poisonous idea; the coming ...
Be7 is deadly for White) 32 Rbc1 Be7 33 Rc8+ Kf7 34 Rc7 Ke8! and White is forced to
hand over the exchange and swap queens, which leaves him busted.
30 ... Qd5!
Threat: ... e3. Also deadly is 30 ... Qa2! 31 Qb6 g5! 32 Bxg5 Qd5 33 Bf4 e3!, forcing
mate.
31 Qg5
31 Rbd1 Qb3!! is treachery taken to a level of exquisite detail. Now if 32 Bxd6?
(marginally better, but still losing is 32 Qb6 Rfxf2+! 33 Rxf2 Qf3+) 32 ... Rfxf2+! 33 Rxf2
Qf3+! exploits the pin of White’s f2-rook to force mate in two moves.
31 ... Qd4!
31 ... Qa2! 32 Be3 Rexe3 also wins.
32 Kh1
After 32 Be3 the simplest is 32 ... Qxe3 and game over.
32 ... Rfxf2 0-1
33 g4 (or 33 Rxf2 Qxf2 34 Qd5+ Kh8 and the checks soon end) 33 ... e3! 34 Kg1 Qe4
forces mate.

Summary
In the Nf3 English versions, we must make two choices:
1. Do we play for ... f5, or do you prefer the ... Nf6, ... h6, ... Be6, ... Ne7, ... c6 and ...
d5 plan?
2. Do we play ... a5, which opens queenside lines? Or do we avoid ... a5 and risk
losing our a-pawn later, as was the case here? Keep in mind that in taking the a-pawn,
White loses time.

Game 30
L.McShane-Z.Efimenko
Wijk aan Zee 2011

1 g3
A more common move order would be 1 c4 g6 2 Nc3 Bg7 3 g3 d6 4 Bg2 e5 5 e4 Nc6 6
Nge2, reaching our position.
1 ... e5 2 c4 Nc6 3 Nc3 g6 4 Bg2 Bg7 5 e4

This is the Botvinnik English structure. White’s idea is to seize more central space than
he normally gets in the English, at the cost of handing Black a potential hole on d4.
5 ... d6 6 Nge2 h5!?
Duncan Suttles would approve! We are all guilty of mindlessly memorizing opening
lines, the purpose and meaning of which are long forgotten to us. It’s easy to sleepwalk
through the first 12 moves of a chess game. As such, it’s refreshing to see a player
deliberately attempt to contaminate mainstream theory with an interesting and
combative idea. Black immediately menaces White with the annoying ... h4. I like this
move, which exudes a clear Modern Defence feel to it, and often throws White’s normalcy
out of sync, rather than opt for the pure reversed Closed Sicilian lines like 6 ... Nge7, 6 ...
f5, and 6 ... Be6.
7 h3
The idea is to meet ... h4 with g4. Instead:
a) 7 h4 Bg4 (provoking f3) 8 f3 Be6 9 d3 Qd7 10 Nd5 Nce7! (the idea is to eject the d5
intruder with ... c6) 11 d4 0-0-0!? 12 Be3 f5 when I like Black’s chances and don’t feel he
stands any worse, G.McDonald-N.Rokitta, correspondence 2000.
b) It just feels wrong to allow us our intent with 7 d3?! h4. after which White would be
putting his life on the line if he castled short.
7 ... Be6 8 d3 Nh6!?
He plans ... f5 and possibly ... Nf7. Also interesting is 8 ... h4!? 9 g4 Qd7 10 Be3 0-0-
0!?.
9 Nd5 f5 10 h4!
White prevents ... h4 and clamps down on the g5-square.
10 ... Nf7
Keeping an eye on g5.
11 Be3 0-0 12 Qd2
On 12 exf5 Black must recapture with his bishop, since 12 ... gxf5? leaves his h-pawn
vulnerable after 13 Bg5! Nxg5 14 hxg5.
12 ... a6
This is a bit of a warning for White: castle long and I will come after you with ... Rb8
and ... b5!.
13 Rc1
More aggressive is 13 0-0-0!? b5 14 Qc2 Ne7 and the complications are certain to
increase.
13 ... Qd7 14 0-0 a5!?
I would play 14 ... Rae8.
15 b3
Note: One idea behind this is to slowly play a3 and b4. A secondary idea is that it
secures c4, which enables d4 in many lines.

Instead, 15 a3 allows 15 ... a4! when White is unable to play b4 without allowing an
en passant capture on b3.
15 ... Kh7 16 a3
Keeping b4 options open.
16 ... Nh6
He either provokes f3, or he slips his knight into g5.
17 Bg5 Ng4 18 Rce1 Nd8 19 f3
This isn’t much of a concession to eject Black’s knight.
19 ... Nh6 20 exf5!?
White decides to disturb the balance. It’s hard to come up with an alternative plan,
except just waiting.
20 ... gxf5
Black doesn’t want to give White’s pieces access to e4. A slightly safer alternative is to
go for 20 ... Bxf5 21 Nec3 Ne6 22 Be3 intending Ne4 next.
21 d4 Nc6 22 Rd1!?
McShane dares Black to grab a pawn. Much safer is 22 dxe5 dxe5 23 Rd1.
22 ... Bxd5!
22 ... Nxd4 is a long and forcing line which eventually leads to equality after 23 Nxd4
exd4 24 Nf4 Bf7 25 Bh3 a4 26 Qd3 axb3 27 Bxh6 Kxh6 28 Bxf5 Bxc4! (deflection). 29
Bxd7 Bxd3 30 Rxd3 Rxa3 31 Rb1 Be5 32 Rdxb3 Rxb3 33 Rxb3 Bxf4 34 gxf4 Rxf4 35 Rxb7
Rxf3 36 Kg2 Rf7!. The game is heading for a draw.
23 cxd5 Nxd4 24 Nxd4 exd4 25 Rfe1 Rae8!?
He doesn’t care about his a5-pawn.
26 Bf1!?
If 26 Qxa5 Nf7 27 f4 Nh6! (preparing to jump back into the g4hole) 28 Bxh6 Kxh6
when the position looks even.
26 ... Nf7 27 f4?
White maintains a tense equality with 27 Rxe8! Rxe8 28 a4 Ne5 29 Kg2 Qf7 30 Qxa5.

Exercise (planning): White logically denied his opponent use of e4, but in this case
he underestimated Black’s future use of the e4-square, which just opened up.
How did Black exploit this?

Answer: Offer an exchange sacrifice to earn to deadly central passed pawns.


27 ... Re4! 28 Bd3 Rfe8!

Tip: A sacrifice like this is one where we should trust our gut feeling over any
calculation. This is an example of a very promising strategic exchange sacrifice
which gives Black the advantage, whether White accepts or not.

29 Kf2
29 Bxe4? is virtually unplayable: 29 ... fxe4 (this instantly prettifies Black’s once
homely structure) 30 Qe2 Kg6 31 Rf1 Qf5 when Black’s central passers rule the board and
far outweigh White’s extra exchange.
29 ... Nh6 30 Bxh6
He can’t allow Black’s knight into g4.
30 ... Kxh6

A new imbalance arises of bishops of opposite colours, which principle states favours
the attacker, which in this case is Black.
31 Qxa5?!
At long last White grabs the loose a5-pawn, but in doing so he allows Black’s attack to
get out of control. He had to try 31 Qc2.
31 ... Bf6!
Once vague worry solidifies into concrete concern for White. Black’s multipurpose last
move achieves:
1. Black worries White about future sacrificial possibilities on both f4 and h4.
2. Black clears the g-file to potentially double major pieces to threaten g3. When
White meets the threat with Rg1, then ... Re3 follows, with an additional attacker on g3.
32 Rc1?!
He had to try 32 Qb5 c6 33 dxc6 bxc6 34 Qc4 d5 35 Qc1, but even here Black
dominates after 35 ... Re3.
32 ... Qg7!
Targeting g3.
33 Qd2
Avoiding 33 Bxe4 fxe4 34 Rxc7 Qg4! when the infiltration to f3 is decisive.
33 ... Re3!
White can no longer cover his base pawn on g3.
34 Rg1 Rg8
Warning: When the base pawn of our structure falls, our position tends to fall with it.

35 Bxf5 Rxg3 36 Bg6!


Our needs run deeper than our reservations. This inventive interpolation fails to help
White, since Black simply ignores it with his next move. Still, it’s White’s best shot to
confuse.
36 ... Bxh4!
There is absolutely no reason to go for 36 ... Rxg6?! which allows White some prayer
of saving the game after 37 f5+ Bg5! 38 hxg5+ Rxg5. Black is still winning, but not nearly
to the same degree as with the game continuation.
37 f5+ Re3+
It’s always disorienting when our check is met by the opponent’s block/check.
38 Kg2
38 Kf1 is met with the same idea, 38 ... Qe5.
38 ... Qe5!
White’s king begins to feel the full weight of Black’s reprisal. Worrisome checks are
menaced on both e4 and g3.
39 Kh1
Exercise (combination alert): We sense a kill here for Black, if only we can
overcome the barrier of our own confusion. Can you find the winning continuation
a Fide 2700 player missed?

39 ... Qf4
Still winning, but immediately decisive is:
Answer: 39 ... Qe4+! 40 Rg2 Bg5! (threat: ... Rh3+ and ... Bxd2) 41 Kg1 Rg3! and
White’s game collapses.
40 Qh2
Likewise, after 40 Rg2 Rh3+ 41 Kg1 Qxd2 42 Rxd2 Bg5 Black wins.
40 ... Qe4+! 41 Rg2 d3!
The push of the d-pawn disorganizes the defence.
42 Rcg1
If 42 Rd1 d2! 43 Rdxd2 Re1+ and White must hand over his queen.
42 ... d2 43 f6!
Desperation arises when we can no longer bear the agony of the present moment.
McShane’s British stiff-upper-lippedness is tested to the bounds of human patience, so he
tries one last desperate attempt to launch a counterattack on a heroic scale.
43 ... Rxg6 44 Qf4+!
A bargainer must have something of value, or there will be no deal. This deflection
shot which leaves White’s queen en prise, startles. Although creative, the idea still fails,
since it’s the same as attempting to reach a planet four light years away in a commercial
airliner. He simply lacks the necessary spaceship to reach his desired destination.
44 ... Bg5
Most certainly not 44 ... Qxf4?? 45 Rxg6+ Kh7 46 Rg7+ Kh8 47 Rg8+ Kh7 48 R1g7+
(“Be consoled that pain is transitory and ends the moment you die,” the rook consoles
Black’s king) 48 ... Kh6 49 Rh8 mate. However, 44 ... Rg5! forces mate with Black’s own
queen sacrifice: 45 Qxe4 Rh3+! 46 Rh2 Rxg1+ 47 Kxg1 d1Q+ 48 Kg2 Rg3+ 49 Kf2 Rf3+
50 Kg2 Qf1 mate.
45 Qxe4 Rxe4 46 f7 Rh4+ 47 Rh2 Rxh2+ 48 Kxh2 Rf6 0-1

Summary
Try the funky early ... h5!? idea, to throw Botvinnik English players out of their comfort
zones.

Game 31
A.Ipatov-G.Guseinov
World Rapid Championship, Dubai 2014

1 c4 g6 2 Nc3 Bg7 3 g3 d6 4 Bg2 e5 5 e3

This is a flexible plan where White can play d3 or d4, and also meet Black’s kingside
pawn storm of ... f5 and ... g5 with f4!.
5 ... f5
In this chapter we concentrate on ... e5 and ... f5 formations. Black can also go for a
reversed King’s Indian Attack set-up with 5 ... Nf6 6 Nge2 0-0 7 0-0 c6 8 d4 Nbd7 9 b3
Re8.
6 Nge2 Nf6 7 b4!?
White often pushes the b-pawn to the fourth rank, but usually not this early. More
normal is 7 0-0 0-0 8 d3 (after 8 d4 c6 9 b3 e4 Black can slowly prepare to attack White’s
king with his extra territorial advantage in that sector) 8 ... c6 9 Rb1 Be6 10 b4 d5 when
Black’s achievement of ... d5 offers dynamic equality.
7 ... 0-0
Black makes no attempt to refute White’s early insertion of b4. Worth a thought is 7 ...
a5 8 b5 e4!?, the idea of which is to allow a black knight use of the hole on c5.
8 0-0 c6!

Tip: Playing for ... c6, ... Be6 and ... d5 is a reliable equalizer for Black against this
system.

9 b5 Be6 10 d3
Instead, 10 bxc6 bxc6 11 d3 Qc7! (Black should avoid a too-early ... d5 and only play
it when ready: 11 ... d5!? 12 Ba3 Re8 13 cxd5 cxd5 14 Qa4 and I slightly prefer White’s
chances) 12 Ba3 Nbd7 is about even.

Note: Be careful about playing ... d5. If you feel uncomfortable about playing it early,
then you can also opt for the slower ... Qc7, and play it only later.

For example, 10 ... Qc7 11 a4 a5! 12 Ba3 Nbd7 and I think Black equalized.
10 ... d5!? 11 Ba3 Re8 12 cxd5
If 12 bxc6 Nxc6! 13 cxd5 Nxd5 Black looks just fine.
12 ... cxd5 13 d4!?
White doesn’t want ... d4 hovering over him, so he fixes the structure, giving Black a
kingside space advantage, to offset White’s queenside space. Chances look balanced
here.
13 ... e4
In such positions territory ceded by one side is unlikely to ever be regained.
14 Na4
He wants to provoke ... b6 to weaken c6.
14 ... b6 15 Nac3 Nbd7 16 Bd6 Bf8!?
It’s strange to swap away the opponent’s (technically) bad bishop for our good one,
but obviously here White’s ‘bad’ bishop was not such a bad piece, since it sat deep in
Black’s camp.

17 Bxf8 Nxf8 18 a4 g5
Black strives for an ... f4 break.
19 Na2!
Eyeing the c6 hole, via b4.
19 ... Qd6 20 Qd2 Ng6 21 Nb4 Bd7 22 Rac1 Rac8 23 Na6 Kf7
Black sees the ending coming and follows the principle to centralize his king.
24 Qb4 Qxb4 25 Nxb4 Ke7 26 Rxc8 Rxc8 27 Rc1 Rxc1+ 28 Nxc1 f4 29 Bf1?!
I think this is White’s first real mistake of the game. He should challenge and dissolve
Black’s kingside spatial advantage with 29 exf4 gxf4 30 f3! fxg3 31 hxg3 exf3 32 Bxf3
Kd6, with a likely draw to follow.
29 ... Kd6 30 Nca2 f3!
Black has a choke hold on the kingside and the only question is: can White fortress
the position, denying Black’s pieces entry?
31 h3 Ne7 32 Nc3 h5 33 Kh2 Ne8

Exercise (planning): White has a choice of three plans to reach his goal of a draw:
Plan 1: Get proactive with 34 h4. If Black plays ... g4, the position will probably be too
sealed for a win; if Black plays ... gxh4, then after gxh4, White’s king has a path into
play
with Kg3.
Plan 2: Take charge with a piece sacrifice for three pawns, starting with 34 Nbxd5,
followed by Nxe4+ and Nxg5, giving White three pawns for the piece.
Plan 3: Just shuffle a random piece and do nothing, asking Black to prove a win.
Two of the three plans should hold the draw, while the unlucky third plan loses.
Which one would you play?

Answer: 34 h4!
White should hold the draw with this active plan.
However, the secondary active plan 34 Nbxd5? is losing: 34 ... Nxd5 35 Nxe4+ Ke7 36
Nxg5 Nc3 37 Nxf3 Nxa4 38 Ne5 Nc3 39 Nxd7 Kxd7 40 f3 Nd6 41 g4 h4 42 Kg2 Ncxb5 and
Black can later sacrifice a piece on White’s kingside passers. Meanwhile Black’s queenside
passed pawns will surge. White is busted.

Tip: Some of us interpret patience as a kind of defeat, but we shouldn’t.

Answer no.2: White can also probably draw with 34 Nca2! and do absolutely nothing
but shuffle, forcing Black to come up with a plan to open the kingside. Our unwillingness
to make a decision is, in a strange way, itself a decision, since inaction can be interpreted
as a plan. White has good chances here to hold the game.
34 ... gxh4
I tried the sealing 34 ... g4 on Komodo and it couldn’t win with Black. I have a feeling
it’s a self-contained entity. I won’t bore you with the 100-move-plus analysis, so I deleted
it!
35 gxh4 Nf6 36 Bh3
36 Kg3 also works. If 36 ... Nf5+ 37 Kf4 Nxh4?? 38 Kg5 when White’s king double
attacks both knights, winning a piece.
36 ... Bxh3 37 Kxh3 Ng4 38 Nd1
38 Kg3?? fails miserably to 38 ... Nf5+ 39 Kf4 Nfxe3! (even stronger than 39 ... Nxh4
40 Nbxd5 Nxf2 41 Nf6) 40 Nxe4+ dxe4 41 Kxe4 Ng2 42 Kxf3 Nxh4+. Black’s extra piece,
coupled with his passed h-pawn, will win.
38 ... Ke6 39 Nc2
39 Kg3 Nf5+ 40 Kh3 Nxf2+ 41 Nxf2 Nxe3 42 Nc6 Nf5 43 Nd1 should be a draw.
39 ... Nf5 40 Nb4 Nxf2+!
The nature of argument is to force the opponent to submit to our view of the truth –
even when our viewpoint may not be the truth. This sacrifice of a piece for two pawns is
Black’s only hope for the full point, but shouldn’t be enough with correct play from White’s
side.
41 Nxf2 Nxe3 42 Nc6 Nf5 43 Nd1! Kd6
43 ... e3 44 Nxe3 Nxe3 45 Kg3 Nf5+ 46 Kxf3 Nxh4+ 47 Kf4 should be drawn.
Exercise (critical decision): The position is rife with ominous unknowns.
White holds the game with correct play. He can play 44 Nxa7, 44 Nb4 or 44 a5.
Once again, two lines hold the draw, while the third loses. What is your choice?

44 a5??
When we have multiple ways to draw, yet pick the single way to lose, we feel like a
person who looks for a flashlight to clearly see the daylight. This misadventure derails the
game from its logical conclusion.
Answer: White should hold a draw with 44 Nb4! e3 45 Nc2! e2 46 Nf2 (White’s army
is no longer one of disposable parts; he needs every one of his remaining pieces to work
at maximum efficiency) 46 ... Nxd4 47 Ne1 Kc5 48 Nfd3+ Kc4 49 Kg3 Kb3 50 Nxf3 Ne6 51
Kf2 Kxa4 52 Kxe2 Kxb5 53 Kd2 and he retains good chances of making a draw.
44 Nxa7!? should also draw, but with some difficulty, after 44 ... Nxd4 45 Ne3 Kc5 46
Kg3 Ne2+ 47 Kf2 Nc3 48 Nf1 Nd1+ 49 Ke1 Nb2 50 Nc6 Nxa4 51 Ng3 Nc3 52 Nxh5 Nxb5
53 Ne7 d4 54 Nf5 Nc3 55 Nf4 e3 56 Ne6+ Kd5 57 Nexd4 Ke5 58 Nxe3! f2+ 59 Kxf2 Kxd4,
with a drawn ending.
44 ... bxa5! 45 Nxa5 Nxd4 46 Ne3 Nxb5 47 Kg3 Nc3! 48 Kf4 d4
Black’s pawns are ants, labouring on a project greater than their individual selves.
49 Nf1 Kd5 50 Nb3 Ne2+
The knight gets behind White’s king and attracts his attention with a delicate cough.
51 Kg5 e3 52 Nh2
Taking Black’s h-pawn at this point is way too slow.
52 ... Ng1! 53 Nc1 Kc4 54 Kxh5 d3 55 Nxd3 Kxd3 56 Ng4 e2 0-1
White’s knight is out of reach.

Summary
Meet White’s flexible e3 system with ... c6, ... Be6 and ... d5, which I believe is a reliable
equalizer for Black.
Chapter Twelve
Anti-Queen’s Pawn Lines and Everything Else
This is the loose ends chapter, where we cover anti-queen’s pawn lines, like the London,
Trompowsky, Torre and Colle, as well as 1 f4, which is Bird’s Opening. Surprisingly, the
anti-queen’s pawn lines don’t work well against our Modern, mainly since we achieve our
... e5 break swifter and more efficiently than in traditional King’s Indian versus London,
Trompowsky et al lines.

Game 32
S.Belouadah-R.Rapport
Tromsø Olympiad 2014

1 d4 g6 2 Bf4
Your writer is one of a dying breed of players who long for the innocence of the pre-
comp days, when we didn’t know everything at the press of a button. Today’s shockingly
rapid alterations in opening theory have an embarrassing way of rendering our old
assumptions obsolete. This is the reason so many players are attracted to the London
System, whose inherent solidity makes the opening virtually prep-proof. However, the
London isn’t so great against a Modern move order. Hopefully this game shows why.
2 ... Bg7 3 e3 d6 4 Nf3 Nd7 5 h3
This is more accurate than 5 Be2. My 2100-rated student Ming is one of the top 10-
year-olds in the country. Here he dares to disobey his wise teacher, who advised him not
to play the London against Modern move order, yet he continues to play London against
Modern, with the barbed obstinacy of pre-teen self-assurance. To London purist students
(I teach many of them), the thought that the London can’t be played on all lines, borders
on the profane. 5 ... e5 6 Bg3 Nh6! (now White may lose another tempo with a coming ...
Nf5) 7 h3 0-0 8 c3 Qe7 9 0-0 f5.
Now ... g5 with a kingside pawn storm in the air. M.Lu-C.Lakdawala, San Diego (rapid)
2017, continued 10 dxe5!? dxe5 (now if Black later plays ... e5, he also controls the e5-
square) 11 Qd5+?! (a waste of time) 11 ... Nf7 12 Qb3 Nc5 13 Qc2 b6 14 Nbd2 Bb7 15 b4
Ne4 16 Bh2 c5 (I looked at the failed combination 16 ... Nxc3?? 17 Qxc3 e4 18 Nd4 c5 19
bxc5 bxc5 and then saw 20 Qa3!) 17 b5? (he had to play 17 Nxe4) 17 ... Nxc3! 18 Qxc3
e4 19 Nd4? (Ming had to live with a pawn-down position after 19 Ne5 Nxe5 20 Qb3+ Kh8
21 Rad1) 19 ... cxd4 20 exd4 Rac8 21 Qe3 Qb4! 22 Nb3 (this loses instantly, but obviously
hopeless was to drop his d4-pawn) 22 ... Rc3! 23 Qd2 (23 Qf4 Bh6 24 Qh4 Bg5 traps his
queen) 23 ... Rxb3! 0-1.
5 ... e5
Note: Here is the problem for White in the London versus the Modern: Black achieves
the freeing ... e5 without breaking sweat, when compared to the London versus the
King’s Indian lines, where Black blocks coverage of the e5-square with an early ...
Ngf6.

6 Bh2 Nh6!

Tip: Retain the central tension and remember the ... Nh6! idea against London.
Black can either play to f5 with the knight, or play for the plan ... f5 and ... Nf7 (as in
the above note versus Lu), where Black has the option of playing for a kingside
pawn avalanche with ... g5 if White castles short.

I think playing ... Nh6 is superior to 6 ... e4 7 Nfd2 f5 8 c4 Ngf6 9 Nc3 when White’s
queenside and Black’s kingside chances look about even.
7 dxe5
With 7 Be2 0-0 8 0-0 f5 White scores only 29% in my database from this position.
Black’s plan is easy: ... Nf7, ... g5 and start attacking White’s king with your pawn
avalanche.
7 ... dxe5 8 Nc3 0-0 9 Bc4 c6 10 0-0 Qe7 11 a4 Nf5
Tip: Don’t be in any rush to push your e-pawn, since this enhances the power
of White’s h2-bishop.

In this case it would be a blunder after 11 ... e4?? 12 Bd6.


12 Qe2 a5 13 Rad1 Nc5
White’s position is passive and he looks to be slightly less than equal.
14 Rd2?!
This inaccuracy allows Black to seize the initiative. 14 Qd2 looks correct.
14 ... Nd6! 15 e4
This concession hands Black the bishop-pair. White probably didn’t like 15 Ba2 Nde4
16 Nxe4 Nxe4 17 Rdd1 Nc5 when he finds difficulty in protecting a4.
15 ... Nxc4 16 Qxc4 Bh6
Simpler is 16 ... Be6 17 Qe2 f6 and White has no useful plan but to await Black’s
intent.
17 Rd5!
Clever, as accepting the exchange sacrifice with 17 ... cxd5?! allows White to seize the
initiative with 18 Nxd5 Qd8 (18 ... Qd6?? 19 Bxe5 is disastrous for Black) 19 Qxc5 f6 20
Rd1 with lots of activity and a pawn for the exchange.
17 ... Nd7
Even stronger was 17 ... Ne6! and White must retreat his rook, since 18 Rxe5? f6 wins
the exchange.
18 Rdd1 Bg7 19 Qa2 Nb6
I prefer 19 ... Nc5! keeping the knight more active. After 20 Qa3 b6 21 Rfe1 Ba6 White
has no useful ideas and can only wait.
20 b3
Now his queen looks rather forlorn on a2, but ... Be6 was coming.
20 ... f6 21 Ne1 f5
Black’s still mild initiative soon begins to sprout wings and fly.
22 Nd3
22 exf5?! violates the principle: don’t open the game when your opponent owns the
bishop-pair: 22 ... Bxf5 23 Qb1 h5 and Black’s bishops own the board.
22 ... f4
With the centre fixed, Black can attack with ... g5, ... h5 and eventually ... g4.
23 f3 Be6 24 Kh1 Nd7
He realizes his knight was misplaced on b6 and brings it closer to White’s king to
generate an attack.
25 Nb1!?
This looks cumbersome. Maybe he should just play 25 Rd2.
25 ... Rfe8 26 Bg1 Qh4
Rapport at first attempts a piece-play attack on White’s king and then later switches
to a pawn avalanche. 26 ... g5 at once begins the pawn avalanche plan.
27 c4
White frees his queen at the cost of creating potential queenside weakness on b4 and
c5.
27 ... g5 28 Qf2 Qh5
Rapport plays for mate. A technical player like me would be tempted into 28 ... Qxf2!?
and then grind away with the bishop-pair plus White’s queenside pawn weaknesses.
29 Qe2 Nf6 30 Nf2
White does all he can to suppress Black’s ... g4 intent.
30 ... Qh4 31 Ng4 Nh5
Threatening a cheapo on g3 and ensuring that White will hand over his remaining
bishop for Black’s knight.
32 Bf2 Ng3+ 33 Bxg3 Qxg3
Eliminating White’s key defender of his dark squares.
34 Rd3 Qh4 35 Rfd1 Qh5
Rapport decides to recentralize his queen. Also possible was to keep her where she
stood, play 35 ... h5 36 Nh2 Bf8 and at some point engineer a ... g4 break.
36 Nd2 Qf7 37 Nf1!?
White braces for the coming kingside storm by feeding defenders to his king. I would
think about launching a desperate central/queenside counter with 37 c5 h5 38 Nf2 Bf8 39
Rc3 Rad8 40 Nd3 Qf6 41 Nc4, but even here Black looks like he is in charge with 41 ...
g4!, with a strong attack brewing.
37 ... h5
The kingside is the screen upon which Black’s ambitions are projected. Finally, he
goes for the pawn avalanche plan.
38 Nf2 Bf8!

The dark-squared bishop is transferred to a more potent diagonal, where Black’s


kingside attack may grow in strength.
39 g4!?
A move based on the philosophy: sentimental decisions don’t win wars. In such dreary
positions we have no option but to resign ourselves to a series of indignities. This act of
structural self-abasement is justified since White will lose if he goes passive and does
nothing. For example, 39 Kh2 Bc5 40 Kh1 b5! with mounting queenside pressure.
39 ... Bc5 40 gxh5 Qxh5 41 Nh2
He hopes to create a defensive barrier by posting a knight on g4.
41 ... Kf7!
Clearing the way for his rooks to attack on the h-file, but can this really be called an
attack, when it comes without cost and carries virtually zero risk? Isn’t it better described
as a build up of force? To me, the word ‘attack’ connotes a roll of the dice and real danger
to the attacking side, as well as to the defender.
I think Rapport’s move is far better than taking the materialistic route with 41 ... Bxh3.
Grabbing a pawn like this turns us into the investment banker who hires a team of
attorneys in order to sue the homeless person for sleeping on his front doorstep. In such
situations we should only pick up material when it’s a spoil of war, rather than waste
resources or energy to gain it. After 42 Nxh3 Qxh3 43 Qg2 Qxg2+ 44 Kxg2 Bd4 45 Ng4
Kg7 46 Rh1 White can continue to resist.
42 Nfg4 Bd4 43 c5
If 43 Rxd4 exd4 44 Rxd4 Qxh3 45 Ne5+ Kf6 46 Nd3 Rad8 and White is just down an
exchange without any compensation.
43 ... Qxh3 44 Rxd4!?
The price of freedom doesn’t come cheaply. White decides that indifference to a
growing menace has the power to destroy him as easily as an overt threat. As such, he
takes the nuclear option of sacrificing the exchange, hoping to somehow get at Black’s
king by weakening e5. The trouble is White is in the impossible situation of attempting to
cure an inherently incurable ailment. 44 R1d2 Rh8 is also completely hopeless for him.
44 ... exd4 45 Ne5+ Kf6 46 Nc4
After 46 Neg4+ Bxg4 White must recapture with his pawn, since his h2-knight is
pinned.
46 ... Bxc4 47 Qxc4 Rad8 48 Rxd4

Exercise (combination alert): It’s time to hold court and deliver judgement.
White’s last move is a blunder in an already lost position. Black to play and force
mate.
Answer: Overloaded defender. White’s queen is unable to simultaneously cover h4
and her d4-rook.
48 ... Rh8 49 Rd2
Clever, but not good enough.
49 ... Rxd2 50 Qc3+ Kg6 0-1
51 Qxd2 is met with 51 ... Qf1 mate “A small animal, wary of predators, is wise only if
it learns to blend into its surroundings,” whispers the queen into her startled brother’s
ear.

Summary
Surprisingly, the London System isn’t so hot against our Modern move order, since we
achieve an effortless ... e5, which puts us at least a tempo up on the London versus
King’s Indian set-ups, where Black’s ... e5 goal is more laboured.

Game 33
M.Roumegous-A.Skripchenko
French Women’s Championship, Besancon 2006

1 d4 g6 2 Bg5

This is the Trompowsky versus the Modern. White’s weirdly posted bishop hopes for
weakening pawn pushes from our side, like ... h6 and ... g5 (which we may later agree
to), and also prevents Black’s easy ... d6, ... Nd7 and ... e5 ideas, as seen in our London
versus Modern lines.
Note: To the Modern player, the Trompowsky and Torre Attack are basically the
same opening, since we reach a transposition after 2 Nf3 Bg7 3 Bg5, which
would be the Torre Attack versus Modern.

We would soon transpose to our game’s continuation after 3 ... d6 4 e3 Nd7 5 c4 Ngf6
6 Nc3 h6 7 Bh4 g5!.
2 ... Bg7 3 e3
I think it’s too late for White to switch back to king’s pawn lines with 3 e4?! since 3 ...
c5! is awkward; ... Qb6 may even follow.
3 ... d6 4 Nf3 Nd7 5 c4 Ngf6

Note: In the Modern Defence there is clearly ideological overlap with the Pirc,
King’s Indian, and Benoni, and sometimes even the Benko Gambit.

In this book don’t hesitate to transpose to any of the above when it suits us. Black’s
last move slips us back into a line of the King’s Indian considered dynamically balanced.
6 Nc3
The idea of 6 h3!? is to create a safe haven for the g5-bishop if Black goes for the ...
h6, ... g5! and ... Nh5 plan. Here we switch plans and play 6 ... Ne4!.

Tip: This ... Ne4! idea is a freeing manoeuvre, also seen in the London System,
when White plays d4, e3 and c4, without Nc3. Black will force White to swap
away the knight which frees our position after 7 Bh4 c5!.
Flexibility is the Modern’s hallmark. Here a ... c5 break is logical since White
weakened the diagonal by transferring his bishop away from the hot zone. After 8 Nbd2
Black can either exchange on d2, or play 8 ... Qa5 which allows White a draw with 9 b4!.
Without this move White would stand worse, but 9 ... Qxb4 10 Rb1 Qa5 11 Rb5 Qc3 12
Rb3 is a repetition draw.
6 ... h6 7 Bh4 g5!

Tip: Remember this idea of ... h6, ... g5! and ... Nh5, which picks up the bishop-pair.
While it’s true that we seize space and weaken our kingside, this factor is
diminished by the fact that we haven’t castled kingside.

8 Bg3 Nh5 9 Be2 Nxg3


There is no rush to swap on g3. I would delay it and provoke White into losing time
with a future Nd2, to force the swap. For example, 9 ... e6! 10 Qc2 a6 and now White
wanted to resolve the issue and made a concession with 11 Nd2 Ndf6!? (Black still
refuses to swap) 12 Nde4 Nxg3 13 Nxf6+ Qxf6 14 hxg3 and in M.Danelia-A.Shimanov,
Kenner 2016, I think Black obtained a slightly superior version to the one played by
Skripchenko.
10 hxg3
Should we worry about White’s potential future kingside attacking chances, due to the
open h-file? The answer is no. We utilize one of our Modern dirty tricks of delaying
castling. We can keep our king in the middle, go queenside, or even much later castle
kingside if it’s safer to do so at that point.
10 ... e6!
Warning: Be very careful about mixing in ... g5 with ... e5, since this seriously
weakens our f5- and e4-squares.

For example, 10 ... e5?! 11 Qc2! when White is prepared to castle long and then play
Bd3, dominating the central and kingside light squares.
11 Qc2
White prepares to castle long, but there is no law which requires us to castle short,
with opposite wing attacks.
11 ... Qe7 12 0-0-0

Tip: We castle kingside only if White does so first with 12 0-0 0-0. Here we don’t
have h-file worries and we can play to exploit our bishop-pair.

12 ... a6!
Multipurpose:
1. This frees Black from Nb5 annoyances.
2. It also worries White about our own future queenside attacking chances with ...
Rb8, ... c6 and ... b5.
13 Kb1 Nf6 14 Bd3
14 e4 can be met with 14 ... Bd7, and if 15 e5 Ng4 16 exd6 cxd6 17 Bd3. Now maybe
we can risk 17 ... 0-0!? since White’s king isn’t so safe with the open c-file.
14 ... Bd7 15 Qb3
White wants to induce queenside castling in order to resolve Black’s undecided king’s
position.
15 ... Rb8!?
An aggressive decision which places White’s king in danger, due to a future ... b5, but
also increases the danger to Black’s king, since now it must either remain in the centre or
go kingside – neither of which is safe. 15 ... 0-0-0 is about even, where White’s space is
counterbalanced by Black’s bishop-pair and potential for future dark-square control.

16 Rc1 Ng4
Inducing Qc2, which allows Black an immediate ... b5.
17 Qc2 b5
Black achieves her thematic break.
18 cxb5 axb5 19 Nd1!?
I wouldn’t move the knight until it’s attacked, and would play 19 Qe2.
19 ... c6 20 Nd2 Ra8
The rook seizes the open a-file.
21 Nb3 Qd8
Black’s forces creep kingward.
22 Be4 Ra6 23 Qd2 Qb6 24 Bd3
White decides to wait and see how Black will attack. I would think about a more
active plan like 24 f4.
24 ... Bf8!?
Rarely is a retreat a good move when we attempt to attack. Black wants it both ways.
She wants to attack, but at the same time keep her king safe. It may be better to just
take the plunge with 24 ... 0-0!? allowing her to feed her h8-rook into the attack. The
cost, of course, is that she divulges her most closely guarded secret: her king’s address,
which allows White to attack as well.
25 e4!
With the pressure released from d4, White can expand in the centre.
25 ... e5!?
Black increases her grip on the dark squares, at the cost of slightly weakening f5 and
her light squares. The safer option was to remain neutral with 25 ... Be7.
26 f3 exd4!
An idea can either be carefully cultivated or grow wild. This appears to be a case of
the latter. We can clutter our mind with a tangle of variations, or we can just sacrifice, as
in this case, based on a gut instinct. To the mind of a scientist, a subjective evaluation is
synonymous with the word ‘useless’, but not so to an artist, who trusts feelings and
instinct, sometimes above logic and data. I don’t think this is a position which relies upon
absolutes, and therefore give the sacrifice two thumbs up. If not objectively, then for its
practical chances. White seizes the initiative if Black backs down with 26 ... Nf6?! 27 dxe5
dxe5 28 Qc3 Bd6 29 f4!.
27 fxg4 c5

In chess, as in life, when we acquire the object of our desire, we also sometimes get
its opposite as an unwanted adjunct. White is up a piece, but for it, look what she gave
up:
1. One pawn.
2. Black gets a monster central/queenside pawn roller.
3. White’s king is gravely endangered.
4. White was forced to hideously deface her kingside, with tripled pawns on the g-file
and an isolani on e4.
Conclusion: I think Black achieved full compensation for her speculative sacrifice. Also
she earns practical chances, since White’s position is certainly harder to play than Black’s.
28 Be2?
Now the pendulum of power swings in Black’s direction. White side-steps the ... c4
threat, but in this case she sells her most precious asset for a trinket, since a passive
response allows Black to seize the initiative. When we are rich in material yet poor in
position, as in White’s case, maybe we should try to bribe our way into heaven, despite
having unrepentantly sinned in life.
Right or wrong, White had to fight back with 28 e5! where she plans for an Ebenezer
Scrooge-like transformation, from material-up miser to generous benefactor. After 28 ...
c4 29 exd6 cxb3 30 Re1+ Kd8 31 a3 Bxd6 your guess is as good as mine as to who
stands better or worse.
28 ... Bg7
The bishop returns triumphantly to its natural g7 home, White’s e5 break is
suppressed and Black’s pawns are ready to roll forward.
29 Rf1 Be6 30 Nf2?
White had to try 30 Rf3 c4 31 a3! and then pray she survives the coming attack.

Exercise: Black missed a wicked combination here. How would you proceed
with Black’s attack?

30 ... c4
Good enough, but far stronger was:
Answer: Annihilation of defensive barrier. After 30 ... Rxa2! Black’s rook and queen
view White’s king and a-pawn roughly equivalent to the way Bonnie and Clyde regarded
bank guards: 31 Kxa2 Qa7+ 32 Kb1 Bxb3 and White’s king is caught in a box.
31 e5 Bxe5 32 Ne4 Ke7?!
An example of ‘lucky’ geometry can be attributed to bad luck; when it happens twice,
as we see here, it’s a pattern. Once again Black misses a same idea with 32 ... Rxa2!
which is crushing.
33 Qb4?!
This move wastes a precious tempo which is better spent playing 33 Nxd4.
33 ... Ra4 34 Qd2 cxb3 35 a3 Qb7
Stronger is the natural prying mechanism 35 ... b4!.
36 Bf3 Qa7 37 Rfe1 f6
Low clocks coupled with such a high level of complications tend to thrust upon the
players a disorienting sensation of disconnection from the board’s reality. The players
must have been in time pressure. Once again Black should rip open the white king’s cover
with 37 ... b4! 38 Nc5 bxa3! 39 Nxa4 Qxa4 40 Rc7+ Kf6 41 Rf1 Kg7 42 Qf2 axb2 43 Qxb2
d3 and White’s position collapses.
38 Rc6 Rc4!
Eliminating White’s only active piece, while mending her own structure.
39 Rxc4
If 39 Rc1 Kd7! 40 Nf2 d5 and White must either take on c4 or on e6, both of which are
hopeless.
39 ... bxc4 40 Qf2 Qa5 41 Rf1
White’s hoped-for initiative is clearly unsustainable. Black simply ignores the ‘threats’
along the f-file and proceeds with her own agenda.
41 ... c3 42 Nxf6
This is sort of the mother of all Hail Marys.
42 ... c2+ 43 Kc1 Bxf6 44 Bd5 Rf8 0-1

Summary
The Trompowsky and Torre, for our purposes, usually lead to the same King’s Indian line,
where we can play for the ... h6, ... g5! and ... Nh5 plan. White’s space is compensated by
our bishop-pair and light-square control.

Game 34
D.Vasilev-N.Djukic
Plovdiv 2014

1 c4
White can set up with a Colle formation, with or without the insertion of c4, as with 1
d4 g6 2 Nf3 Bg7 3 e3 d6 4 Bd3.
Tip: Some Colle players open mechanically with d4, Nf3, e3 and Bd3 against
everything. In this instance White’s bishop pay be misplaced on d3, since Black
can play for an ... e4 tempo gain.

4 Be2 is better, yet many of my Colle-loving students absolutely insist on playing their
light-squared bishop to d3. After that move, 4 ... Nd7 5 0-0 e5 6 c3 Qe7 (threat: ... e4,
winning a piece) 7 Nbd2 Ngf6 renews the threat. Now if 8 e4 White enters a version of
Coward’s variation, but a tempo down, since it took two moves to play e4.
1 ... g6 2 e3
This move may lead to a Colle formation.
2 ... Bg7 3 d4 d6 4 Nf3
We can buy two different books on the same opening system and we may get parallel
universes of assessment. One writer may call White’s choice of Colle formation
“pathetically unambitious and a waste of the white pieces”, while another writer like me
(I work tirelessly to make the chessboard a safer place for the meek among us, and have
written two books on the Colle) may describe it as “solid” and “a playground for the
unbooked up of the chess world”. The humble Colle’s philosophical outlook can be
described this way: if modesty constitutes a virtue, then modesty in excess must be even
better! The opening essentially creates a geological barrier against Black’s ambition.
4 ... Nf6
This move transposes to a position which is considered a harmless (for Black) version
of a King’s Indian. So innocuous is White’s Colle formation, that we are presented with a
choice of set-ups:
a) 4 ... Nd7 5 Nc3 e5 6 Be2 Nh6 (we are already familiar with this set-up from the
London versus Modern) 7 b4 (White’s arena of operations will be the queenside) 7 ... 0-0
8 a4 f5 with mutual wing attacks.
b) 4 ... c5 5 Nc3 Nc6 and then:
b1) 6 Be2 Bg4!? (our single-minded goal is to pressure d4 to the point where White is
obliged to push to d5) 7 d5 Bxf3 8 Bxf3 Bxc3+! (by now, we are old hands at this swap) 9
bxc3 Ne5 10 Be2 and now we borrow the intellectual property of the Dzindzi-Indian with
10 ... f5! where our two knights boldly challenge White’s two bishops in the somewhat
closed position, C.Daly-V.Malakhov, Khanty-Mansiysk Olympiad 2010.
b2) 6 d5 can be met with 6 ... Na5 7 e4. We took a look at this position in a note in
the Averbakh section of the book. White lost a tempo to achieve e4, while Black’s a5-
knight may or may not be interpreted as being out of play. We all fear that rhyme: a
knight on the rim is dim (or grim), but here I’m not so sure. Black will play for ... a6, ...
Rb8 and ... b5, where the knight begins to pressure White’s c4-pawn.
5 Be2 0-0 6 Nc3 c5
Maybe it’s stylistic bias, but I don’t like the reversed King’s Indian Attack plan of 6 ...
Nbd7 7 0-0 e5 8 b4 Re8 9 a4. I play White’s position down a tempo as Black with French
Defence versus the King’s Indian Attack, and do rather well with it. As such, I wouldn’t
want to take Black’s side here, a move down from normal.
7 0-0
7 d5 e6 is wonderful for our side, since we reach a Benoni formation a move down for
White, where the e-pawn sits passively at e3, rather than e4.
7 ... b6
After 7 ... Nc6 8 b3 (or 8 d5 Na5 and once again we reach a set-up similar to the one
we discussed in the above note) 8 ... Bf5 9 Bb2 Ne4 Black reached a comfortable equality
with this freeing swap.
8 b3

White absolutely refuses to push his d-pawn to d5, and goes for a Zukertort Colle
structure.
8 ... Bb7 9 Bb2
Of course, 9 d5?? would be a rookie’s error. Black wins material by simply exploiting
the pin with 9 ... Ne4.
9 ... e6
9 ... Nbd7 10 Qc2 a6 11 Rad1 e6 12 dxc5 Nxc5 13 b4 Nce4 is also fine for Black, who
reached a decent Hedgehog structure.
10 Qc2 Nc6 11 dxc5!?
This move shifts the structure and makes the game more interesting. With 11 Rad1
Qe7 12 Rfe1 d5! Black stands no worse, as 13 cxd5 is met with 13 ... Nb4 14 Qb1 Nfxd5.
11 ... bxc5
Principle: capture toward the centre. Black’s b-pawn got promoted to a c-pawn,
earning greater central influence. Instead, 11 ... dxc5 is unambitiously equal and boring.
12 Rad1

Tip: Don’t fear taking on a backward d6-pawn. The pawn looks a lot weaker than
it actually is. Black gains central influence and can later play for three possible plans:
Plan a: Launch a minority attack on the queenside much later with an ... a5, ... a4 and
... axb3 plan. This leaves b3 weak and also we may be able to occupy the b4 hole.
Plan b: Play for a kingside attack with a future ... Ne8, ... f5 and even ... g5!?.
Plan c: Play for ... d5 later on, taking on hanging pawns, greatly sharpening the game,
as seen in this game.

12 ... Qe7 13 Rd2


This plan to pile up on the d6-pawn isn’t so scary, since Black can easily defend it as
many times as it’s attacked. Still, I don’t see any other ideas for White, except this one.
13 ... Rfd8 14 Rfd1 a6
A precaution against future Nb5 ideas.
15 Ne1
This move may seem strange, but White is actually in an odd sort of zugzwang, in that
all his pieces are perfectly placed, and any alteration actually decreases their potency.
Still, there is no reason to retreat. Maybe a better way to shuffle is 15 Qc1.
15 ... d5!
Of course. if Black is okay with a draw, then doing nothing is fine. But if you are
ambitious, then taking on hanging pawns is well justified, since White’s pieces are
passively placed. I think avoiding 15 ... d5 out of fear of creating weakness is the same as
the cautious caveman who by happy accident, stumbles upon fire and thinks to himself:
‘It’s probably a bad idea. I should douse it with water before it hurts someone.’
16 cxd5
Following 16 Bf3 d4 17 Bxc6 dxc3 18 Rxd8+ Rxd8 19 Rxd8+ Qxd8 20 Bxb7 cxb2 21
Nd3 (21 Bxa6?? Qa5 simultaneously hits e1 and a6) 21 ... Ne8 22 Bc6 Kf8 23 g3 Qa5 24
Ba4 Qc3 25 Qd1 Black’s fearsome b2-pawn ties White down in the ending.
16 ... exd5

We reach a hanging pawns structure. Are hanging pawns a strength or a weakness? It


all depends. If they seize space and disrupt, then they represent a strength; if too many
pieces are swapped and they turn into targets, then they represent a weakness.
17 Nf3?!
White loses the initiative with this move. He should immediately worry Black with the
aggressive 17 Na4! Ne4 18 Bxg7! Nxd2!? (your writer is the kind of player who always
believes that life goes a lot smoother when you carry plan ‘b’ in your back pocket; I would
probably opt out with the meek 18 ... Kxg7) 19 Ba1!/ This sacrifice is the corporation
which puts profits above public safety in their product. White’s monster dark-squared
bishop offers huge compensation for the exchange. Even a materialistic player like me,
who is usually unable to give up even a tiny material concession without a small tug of
sorrow in my heart, would be willing to take on White here.
17 ... Rac8 18 Qb1
The queen was uncomfortable with Black’s rook eyeing her on c8.
18 ... d4!
Here we see an example of the disruptive power of hanging pawns. White’s pieces are
sent into disarray.
19 exd4 cxd4 20 Na4?
When we are in trouble, it’s important to limit our defensive misery to manageable
levels. This meek move makes matters worse and now White is caught in a tide he is
unable to navigate. He had to try 20 Nxd4 Nxd4 21 Rxd4 Nd7! when Black threatens d4,
c3 and e2 and wins material. White must hand over an exchange with 22 Bf1 Bxd4 23
Rxd4 Ne5 24 Ne2 Rxd4 25 Bxd4. He doesn’t quite have enough for the exchange, but still
his position is infinitely superior to the one he reached in the game continuation.
20 ... Ne4 21 Rc2
From this point to its demise, White’s poor rook gets no rest. 21 Rd3 is met with 21 ...
Nb4!.
21 ... Nc3!
Here we see a secondary component of Black’s idea and White feels the
disillusionment anew. This clever overload shot creates a suffocating passed pawn.
22 Nxc3 dxc3 23 Bc1
This is the case of the overloaded defender. White drops a piece after 23 Bxc3?? Bxc3
24 Rxc3 Qxe2.
23 ... Rxd1+ 24 Bxd1

Exercise (combination alert): This is the ‘why me?’ moment for White.
Find one powerful move and White collapses.
Answer: Weak back rank. White’s rook is trapped.
24 ... Nd4! 25 Be3
Alternatively, 25 Nxd4?? walks into 25 ... Qe1 mate, and 25 Bg5 is easy to deal with:
25 ... Qe6! 26 Rc1 Bxf3 27 gxf3 (27 Bxf3 c2! is a killer) 27 ... c2! wins.
25 ... Nxc2 26 Bxc2
26 Qxc2 Bxf3 27 Bxf3 Bd4! 28 Qc1 Bxe3 29 fxe3 c2 is also totally hopeless for White.
26 ... Bxf3 27 gxf3 Qd6 28 Qc1 Bd4 0-1

Summary
The Colle formation isn’t so scary against our Modern. We have a choice of multiple
satisfactory set-ups, playing either for ... e5 or ... c5.

Game 35
H.Danielsen-G.Jones
Reykjavik 2016

1 f4
Some of us believe that we must be specialists, rather than generalists, when it
comes to opening preparation. This is Bird’s Opening. GM Henrik Danielsen plays 1 f4
with White and as Black he plays 1 ... f5 on most non-1 e4 openings. In this way he
remains within his Bird’s/Dutch realm at all times.
1 ... g6 2 g3
This is the Leningrad Dutch formation with an extra tempo, since it’s played as White.
We must be familiar with this one, as well as two others after 2 Nf3 Bg7 3 e3 d6:
a) 4 d3 is the reversed Classical Dutch formation. After 4 ... c5 5 Be2 Nc6 6 0-0 Nf6
play will be very similar to the English versus Modern chapter, but this time we are the
ones who take on the English Opening’s side.
b) 4 d4 is the reversed Stonewall Dutch formation, met with 4 ... Nd7 5 Bd3 e5!.
Tip: Stonewall formations are not so great for White when Black’s d-pawn is held
back and we can play for ... d6 and ... e5!.

With 6 0-0 Nh6(this move reinforces both f5 and f7) 7 fxe5 dxe5 Black easily
equalized, K.Okur-S.Sen, Konya 2016.
2 ... Bg7 3 Bg2 c5
By playing in reverse English style against our opponent’s reversed Dutch formation,
we embrace a union of opposites. White will most likely attack kingside, while we operate
on the queenside.
4 d3 Nc6 5 Nf3 Nf6 6 0-0 0-0 7 Nc3
In this game Danielsen plays for an f5 break without e4. White’s more normal sets
are:
a) 7 e4 d6 8 c3. This set-up is known as the Clamp. White plays for an eventual d4
central space advantage. We respond with our normal wing attack with ... Rb8 and ... b5:
8 ... Rb8 9 h3 (this allows Be3 without fear of ... Ng4) 9 ... b5 10 Be3 b4 11 Nbd2 (11 c4??
is met with 11 ... Nh5! 12 g4 Bxb2 13 Nbd2 Bxa1 14 Qxa1 Ng7; White doesn’t have
enough attack for having given up an exchange and a pawn) 11 ... bxc3 12 bxc3 Ba6 13
Qc2 Nd7 and Black’s queenside play fully compensates White’s extra space.
b) Against 7 c3 I suggest we stick to pure Reversed English lines with 7 ... d6, since
we are already familiar with them from the English versus Modern chapter of the book. If
White plays e4, then we will simply transpose to the Clamp, from line ‘a’.
c) With 7 Qe1 White can play for e4, and also a plan involving h3, g4 and Qh4, with a
kingside attack brewing. After 7 ... d6 8 h3 Rb8 9 g4 b5 I think Black’s queenside chances
compensate White’s potential on the kingside.
7 ... d6 8 Nh4!?
8 e4 gets us to lines we already looked at in the English versus Modern chapter,
except this time we are the ones taking on the English side a move down. Chances are
about even.
8 ... Rb8
Black continues with the normal English plan of queenside expansion.
9 a4 a6 10 f5 Nd4
Black seeks a resolution to the tension on f5.
11 e4
He won’t oblige, but now Black’s knight sits well on d4. Chances look approximately
balanced.
11 ... b5
The increasing tug of opposing ambitions grow clamorous, with each side demanding
attention to their agenda.

12 axb5 axb5 13 fxg6?!


In a sense this move violates the gravest of attacking sins: partial commitment.
Resolving the tension benefits Black. White should consider retaining the pawn tension
and challenging Black’s strongest piece with 13 Ne2.
13 ... fxg6!
Believe it or not, this move which looks like a violation of principle, actually follows it.
Principle: capture away from the centre if your opponent’s attackers endanger your king. I
think Black’s move is stronger than the rote 13 ... hxg6 14 Nd5 b4 15 Bg5 with chances
for both sides.
14 Bg5 Bg4 15 Qd2 b4 16 Nd1
16 Nd5? hangs material to 16 ... Nxd5 17 exd5 Ne2+ 18 Kh1 Bxb2.
16 ... Nh5
Jones unleashes his g7-bishop’s power along the a1-h8 diagonal.
17 Rxf8+ Qxf8 18 Ra7?
White should play 18 Kh1 – an action driven by the press of defensive necessity.
Exercise (combination alert): A weapon must be cast aside when it becomes
unstably dangerous for its owner. White’s last move, attacking e7, was a blunder
allowing Black a devastating combination. How would you continue as Black?

Answer: Step 1: Drive White’s king into the corner.


18 ... Ne2+!

Tip: Caution has no place in military planning when a time-sensitive opportunity


arises.
When it’s time to strike, don’t hesitate!

In a strange way, this combination has no beginning, since its ‘start’ is connected to
previous strategic ideas which brought it to its current fruition.
19 Kh1
Step 2: Sacrifice on g3, fatally exposing White’s king.
19 ... Nhxg3+! 20 hxg3 Nxg3+ 21 Kg1
The king blends in as unobtrusively as an unbathed homeless man in rags, carrying a
half-drunk bottle of gin, who bought a ticket to the opera. When we are hopelessly lost in
the wilderness, one direction serves as well as the others. 21 Kh2 is met with 21 ... Be5!
(threat: ... Nxe4+ and ... Nxd2; White’s queen lacks safe havens and must hand over a
piece to save herself) 22 Bxe7 (or 22 Qe1 Nxe4+ 23 Kh1 Nxg5) 22 ... Nf5+ 23 Kh1 Nxe7.
21 ... Ne2+ 22 Kh1 Ng3+ 23 Kg1
If 23 Kh2 Be5 and White’s king is caught in the crossfire.
23 ... Bd4+!
White’s porous dark squares slip into enemy hands.
24 Be3
24 Kh2 Nf1+ 25 Bxf1 Qxf1 leaves White’s king helpless.
24 ... Ne2+ 25 Kh1 Qf6!
Step 3: GM Jones is notably cruel in putting his wounded opponent away. His idea is
neatly encapsulated in this finishing touch, where the entry of Black’s queen into the
attack is decisive.
26 Qe1
26 Bg5 Ng3+ 27 Kh2 Nf1+ 28 Bxf1 Qxf1 and if 29 Ne3 Be5+ forces mate.
26 ... Be5! 0-1
Threat: ... Ng3+ and ... Qxh4 and further resistance is futile. Indeed, after 27 Bf2 Qf4!
(threatening mate on h2) 28 Bg1 Ng3+ 29 Kh2 Bxd1 30 Qxd1 Nxe4+ 31 Kh1 we reach a
situation where neither fight nor flight is available. For White’s boxed-in king, this is
one of those nightmares where our dream feet make ineffective fleeing motions, yet our
dream body doesn’t budge and refuses to obey. 31 ... Nf2+! (removal of the guard) 32 Bxf2
Qh2 is mate.

Summary
Meet all Bird’s lines with the English set-ups.
Index of Complete Games
Agrest.E-Morozevich.A, Mallorca Olympiad 2004
Aldama.D-Lakdawala.C, San Diego (rapid) 2017
Aldama.D-Xia.B, San Diego (rapid) 2017
Arutyunov.N-Lakdawala.C, San Diego (rapid) 2010
Baker.B-Lakdawala.C, San Diego (rapid) 2016
Barczay.L-Suttles.D, Sousse Interzonal 1967
Belouadah.S-Rapport.R, Tromsø Olympiad 2014
Danielsen.H-Jones.G, Reykjavik 2016
Darini.P-Mamedov.R, World Rapid Championship, Doha 2016
Dordjevic.S-Lakdawala.C, San Diego (rapid) 2015
Ginsburg.M-Lakdawala.C, American Open, Los Angeles 2002
Graves.P-Lakdawala.C, San Diego (rapid) 2010
Hepworth.P-Gharamian.T, London (rapid) 2015
Humphrey.J-Lakdawala.C, San Diego (rapid) 2012
Ipatov.A-Guseinov.G, World Rapid Championship, Dubai 2014
Jovanovic.Z-Rapport.R, Austrian League 2015
Lakdawala.C-Xia.B, San Diego (rapid) 2017
Loiseau.Q-Wojtaszek.R, French Team Championship 2013
Mammadov.Z-Guseinov.G, Nakhchivan 2015
McShane.L-Efimenko.Z, Wijk aan Zee 2011
Naiditsch.A-Svidler.P, Khanty-Mansiysk (rapid) 2009
Nikolic.P-Topalov.V, Monaco (blindfold) 1998
Peng.Z-Malakhov.V, European Club Cup, Bilbao 2014
Raetsky.A-Sakaev.K, St. Petersburg 1999
Roumegous.M-Skripchenko.A, French Women’s Championship, Besancon 2006
Salaun.Y-Seret.J, French Championship, Epinal 1989
So.W-Norwood.D, Bunratty 2015
Stevic.H-Nepomniachtchi.I, European Individual Championship, Aix-les-Bains
2011
Sussman.L-Lakdawala.C, San Diego (rapid) 2012
Toh.T-Rogers.I, Singapore 1997
Tsang.H-Lakdawala.C, San Diego (rapid) 2010
Vasilev.D-Djukic.N, Plovdiv 2014
Wei Yi-Carlsen.M, Bilbao 2016
Yang.K-Lakdawala.C, San Diego (rapid) 2015
Yudin.S-Smirnov.P, Moscow 2008

Вам также может понравиться